Module 3
Module 3
kilogram (kg) > hectogram (hg) > dekagram (dag) > gram Solution
(g) Desired quantity = (given quantity)(conversion factor)
4.598 kg 45. 98 hg 459.8 dag 4598 g
What is the Knowing the rate of the infusion, that means for every 1 min 500 mcg of
equivalent value of drugs is infused to the body. Therefore, in 1 hr, how much of the drug is
X= infused to the body in terms of mg.
62,500 mcg in g?
X = 0.0625 g c. Length of time for infusion to last
INTERSYSTEM CONVERSION
145 µL x 0.0145 cL = 1.45 x10-2 cL
o Remember, converting a smaller unit to a higher unit, you need to move the
CONVERSION EQUIVALENTS OF LENGTH
decimal to the left depending on the number of zeros or conversion factor. 1m 39.37 in
Now if you convert from a lower unit to a higher unit, the decimal pont will 1 in 2.54 cm
be shifted to the right.
CONVERSION EQUIVALENTS OF VOLUME
Convert 55 ng to equivalent values in micrograms and grams. 1 gal 3785 mL
Solution 1 qt 946 mL
55 ng x 0.055 mcg = 5.5x10-2mcg 1 pt 473 mL
1f 29.57 mL
55 ng x 5.5x10-8 g or 0.055 g 1f 3.69 mL
1 0.06 mL
Convert 1.04 m into millimeters and micrometers. 1 mL 16.23
Solution
CONVERSION EQUIVALENTS OF WEIGHT
1.04 m x = 1,040 mm = 1.04 x 103 mm
1 kg 2.20 lb (avoir.)
1 lb (avoir.) 454 g
1.04 m x = 1,040 mm/1.04 x 106 micrometer
1 lb (apoth.) 373 g
1g 15.432 gr
How many acetaminophen tablets can be prepared from 60 g
1 gr 0.065 g (65 mg)
of acetaminophen if each tablet is required to contain 150 mcg.
1 (apoth.) 31.1 g
Solution
1 oz (avoir.) 28.35 g
60 g x x = 400, 000 tablets Whether it is apothecary or the avoirdupois system, using 2.20 lb as the
conversion equivalent of 1kg is common practice.
Based on experience, whenever there is conversion of pound to gram, whether
Two Tylenol regular strength tablets, each containing 325 mg of it is apothecary or avoirdupois, they usually use 454 g as the conversion
the drug, are prescribed to be taken once daily for 3 weeks. equivalent.
Calculate the total amount of drug in milligrams taken during this
3-week period. PRACTICE EXERCISES ON INTERSYSTEM CONVERSION
How many milliliters are equivalent to 8 fluidounces of a cough
x x x 3 weeks = 13, 650 mg or 13.65 g syrup?
Solution
Equivalent volume in mL
Traditional system of pharmacy; used in compounding of
prescription
o In terms of fluid measurement, it is common to all.
o Involved in compounding prescriptions or in commerce/trading A pharmacist received a prescription calling for 30 capsules,
each to contain 1/200 gr of nitroglycerin. How many 0.4 mg
nitroglycerin tablets would supply the amount required?
60 minims ( ) 1 fluidrachm or fluidram (f ) Solution
8 fluidrachm (480 minims) 1 fluidounce (f ) Number of tablets needed to fill the prescription
16 fluidounce 1 pint (pt or )
2 pints (32 fluidounces) 1 quart (qt)
4 quarts (8 pints) 1 gallon (gal)
Solution
Amount of medication in grams in 3 days:
II. REDUCING AND ENLARGING You need to use all of these computed quantities, then this would be good
enough to prepare a 240 mL of calamine lotion.
FORMULAS
Ca(OH)2 SOLUTION
OBJECTIVES: Enough to make 240 mL
There is no specific amount, but you must add enough of this to make
To perform calculations by various methods to reduce or whatever is the needed quantity.
enlarge formulars for pharmaceutical preparations
o Formulas used for our reference will be found in the USP and National
Formulary FACTOR METHOD
o Generally, these preparations are based on the final volume of 1000
mL or 1000 g of the product. If your final formulation is 1000 mL
preparation and you only need 30 mL of the preparation, you have to
reduce the original formula, and everything must be reduced.
o You should be familiar on the amount of the ingredients to be able to
reduce the formula to be able to get the desired amount of the product
since 1000 mL would usually be the final product amount.
If you observe, you can always see the factors 240 and 1000 in all the steps
FORMULAS MAY BE REDUCED OR ENLARGED USING THE if you are solving by ratio and proportion. So since this is common to all the
steps, then we might solve it only once. Multiply the factor to each of the
FOLLOWING METHODS quantities of the given ingredients.
prepare the 7.5 g of white petrolatum-based on the amount of the other FACTOR METHOD
ingredients that were computed.
The ratio and proportion method can be made simpler using the factor method
wherein we just have to determine the factor needed to be multiplied to the amount
FACTOR METHOD of each of the ingredients. This can be done by getting the ratio of the desired final
Using the factor method, which is a more simplified method, all you have to do is: quantity (1000 g) and the original final quantity (65 parts).
The desired final quantity you must prepare is 7.5 g, but the original/given
formula calls for the preparation of 100 g. Both are in grams, so it can be
canceled out and take the ratio, which is 0.075. Coal tar 5 x 15.38 = 76.92 g
In using the factor method, we must multiply the factor to each of the amounts Zinc oxide 10 x 15.38 = 153.85 g
of the given ingredients. Hydrophilic ointment 50 x 15.38 = 769.23 g
These are the amounts needed.
Dexamethasone 55 x 0.075 = 4.125 mg
Lanolin 5 x 0.075 = 0.375 g
Mineral oil 10 x 0.075 = 0.75 g
White Sufficient quantity to make 7.5 g
Petrolatum If you need the exact amount of the white petrolatum,
you must deduct the quantities of the other ingredients
from 7.5 g. Take note that they should be expressed
into a common unit first.
PROBLEM 3
What if you are not given exact amounts and given proportional parts already?
This does not mean that you must add enough to make a total of 50 parts. In
short, 50 parts do not represent the totality of the preparation. In this case,
it will only represent the proportional part or the amount of one ingredient,
which is the hydrophilic ointment.
Knowing that you will be incorporating all these ingredients to prepare an
ointment, then what will be the representation of the totality of the ointment?
If you are incorporating 5, 10, and 50 parts, what will be the total amount?
The total amount of the ointment will be represented by the sum of all these
quantities, which is 65 parts.
COAL TAR
5 parts : 65 parts = x g : 1000 g
x = 76.92 g
For coal tar, 5 parts are needed to prepare 65 parts of the ointment. How
much is needed to prepare 1000 g? It would always be the ratio of the
amount needed in the entire ointment.
ZINC OXIDE
10 parts : 65 parts = x g : 1000 g
x = 153.85 g
HYDROPHILIC OINTMENT
50 parts : 65 parts = x g : 1000 g
x = 769.23
PHARMACEUTICAL CALCULATIONS| MODULE 3 7
OBJECTIVES:
Calculate doses based on factors: This is just a simple ratio and proportion. The dose of the drug for every 1
o Age kg body weight (dose per kg) as to what would be the dose of the drug for
a patient having such bodyweight.
o Body weight
o Body Surface Area
SAMPLE PROBLEMS
SAMPLE PROBLEM 1
The usual dose of piperazine is 500 mg. Calculate the dose for
A. BASED ON AGE
We can observe that majority of the commercially products are formulated and a 10 yr. old child.
labeled for adult use. So, when it is intended for pediatric use, we must calculate Based on the age the child is 10 years old. Therefore, we have to use either
for the appropriate dosage that may be given to the patient. There will be an . Since the child is
adjustment of the adult dose to make it suitable for children to take. already 10 years old, therefore, .
Adult dose: the amount of drug that would give the desirable therapeutic Solution
response.
o An average adult dose of an analgesic is 500 mg, which should not be
given to a child. It should be computed based on the given factors (age,
body weight, body surface area).
: (2-12 years old) As per agreement, we will round off our answers up to the second decimal
place.
SAMPLE PROBLEM 2
If the adult dose of a drug is 100 mg, calculate the approximate
: (Less than 2 years old) dose for a child with a BSA of 0.83 m2.
Solution
B. BASED ON WEIGHT
:
Recall your conversion factors [1 kg = 2.2 lbs] Whenever your quizzes or long exams are given multiple choices, you have
to find or choose the best answer. Your answer may not be exactly the same
as one of the choices, but you choose the one that is nearest because there
might only be a slight difference in terms of the conversion factors that we
are using.
C. BASED ON BODY SURFACE AREA
To find the body surface area of a child there are two methods. SAMPLE PROBLEM 3
One is using a table known as the monogram, however since the board exam
will not allow you to use references, we will not discuss about it. The usual initial dose of chlorambucil is 150 ug/kg of body
Therefore, we will stick to a formula which will allow you to determine the weight. How many milligrams should be administered to a
body surface area. The BSA would depend on two factors: the height and the person weighing 154 lbs?
weight of the patient. Solution
The basis of the dosing is body weight. In order to compute for the individual
dose all you have to do is multiply to this the body weight in kg.
Since the body weight is given as 154 pounds therefore you must divide this
Example: by 2.2 pounds in order to express it as kilogram. Then, multiply this directly
by 150 to get the final dose (10,500 ug). But in the problem, it needs
Calculate the BSA for a patient measuring 1.65m in height and milligrams, so you must convert the ug to mg, so divide the weight by 1000.
weighing 65 kg. Review again conversion factor.
SAMPLE PROBLEM 4
The usual pediatric dose of a drug is 0.2 mg/m2. Calculate in
micrograms the dose for a child weighing 15 kg and measuring
representing the body surface area of an average adult. An average adult is 90 cm in height.
defined as a person having a weight of 65 kilograms (kg) and a height of
1.65 meters. Solution
CALCULATION OF INDIVIDUAL'S DOSE
You can directly calculate the individual dose after getting the BSA by LOPINAVIR
multiplying it to 0.2mg/m2. If not, list at least 4 decimal places because this
is only an intermediate step so the answer will not be affected. Only round
off the final answer to two decimal places.
CASE IN POINT 8.4 (P.143) Based on the given oral solution, the concentration of lopinavir is 80 mg/mL.
A hospital pharmacist is consulted on the appropriate dose of If the calculated single dose for the patient is 101.25 mg, then this would
lopinavir/ritonavir (KALETRA) oral solution in the treatment of correspond to the volume of the oral solution.
HIV-1 infection in a 12-month-old pediatric patient. The oral
solution contains, in each milliliter, 80 mg of lopinavir and 20 mg RITONAVIR
greater
than 6 months of age, not receiving other concomitant therapy,
may be calculated based on either BSA or bodyweight as
follows: C) CALCULATE THE DAILY DOSE IN MG BASED ON THE
230/57.5 mg/m2, administered twice daily PATI
12/3 mg/kg for patients <15 kg, administered twice daily This time we compute for the daily dose. For A and B earlier, we only computed for
10/2.5 mg/kg for patients >15 kg, administered twice in kg.
daily
LOPINAVIR
The patient measures 28 inches in length and weighs 22 lb.
These are the bases for the computation. If you are going to compute the
dose of the patient based on body surface area, the first bullet would be the
usual dose (230/57.5 mg/m2, administered twice daily). This first value will
represent lopinavir while the second is ritonavir.
If it is computed based on body weight, then you have to check first: is the
bodyweight of the patient less (12/3 mg/kg for patients <15 kg,
administered twice daily) or more than 15kg (10/2.5 mg/kg for patients RITONAVIR
>15 kg, administered twice daily).
To be able for you to determine which among the two bodyweight formulas
to use, you need to convert first the weight in lbs to kg. For you to compute
for the weight in pounds to kilograms, you have to divide the body weight
(22 lbs) by 2.2 which will give you 10 kg. Therefore, the basis would be
12/3 mg/kg, administered twice daily. Based on this dose, you are to translate this in terms of the volume of solution
to be administered. What equivalent volume will contain these corresponding
A) CALCULATE THE SINGLE DOSE, IN MG, USING THE BSA amounts of drug?
EQUATION
D) TRANSLATE THE DAILY DOSE FROM (C) INTO
CORRESPONDING MILLILITERS OF THE ORAL SOLUTION
We go back to the original strength of the drugs in the oral solution (80 mg/20
mg lopinavir/ritonavir in each milliliter).
LOPINAVIR
Since it is given in 28 inches and every inch is equal to 2.54 cm, multiply 28
by 2.54 to express the height in centimeters. Meanwhile, the weight of 22 lbs
is divided by 2.2 to express in kg.
RITONAVIR
LOPINAVIR
Of course, the two should be of the same volume because both drugs are
combined within the solution. You cannot administer 3 mL of the two
separately, and another volume of ritonavir which is different from that of
RITONAVIR lopinavir. This means that when you administer 3 mL of the drug, it will
automatically contain 240 mg of lopinavir and 60 mg of ritonavir.
PHARMACEUTICAL CALCULATIONS| MODULE 3 9
IV. ALTERING PRODUCT STRENGTH AND How many milliliters of a 1:5000 w/v solution of the
preservative lauralkonium chloride can be made from 125 mL
USE OF STOCK SOLUTIONS of a 0.2 % solution?
OBJECTIVES
1. Perform calculations for altering product strength through Solution
dilution or fortification (concentration)
2. Perform calculations for the preparation and use of stock
solutions
CONCENTRATION/FORTIFICATION Solution:
Strengthening a preparation by:
o Addition of active ingredients
o Admixture with a like preparation of higher strength
o Reducing amount of the diluent through evaporation of its
vehicle in the case of liquid
How much drug should be used in preparing 50 mL of a solution
DILUTION AND CONCENTRATION such that 5 mL diluted to 500 mL will yield a 1:1000 of solution?
Solution:
80% v/v %v/v = (80/130)x100=61.5%
Solute 80 80
Solvent 20 50
Solution 100 130
80% v/v = 80 parts of solute in every 100 parts of solution
In this case, if we add 30 parts of the solvent, the amount of solvent would 0.5 g of the drug will come from the 5 mL portion of 500 mL stock solution
increase from 20 to 50, while the amount of solute remains unchanged
However, the concentration of the final solution will be altered because you
have increased the amount of solvent DILUTION OF ALCOHOL
In the process of dilution:
o Only the solvent will increase IN TERMS OF VOLUME (V/V)
o Solute remains unchanged
o There is a change in the total amount of solution When water and alcohol are mixed, physical contraction
In the process of dilution, you are reducing the strength of the solution from occurs, the volume of the mixture is less than the sum of the
80% to 61.5% individual volumes
The strength/concentration is inversely proportional to the total amount of Volume of water cannot be calculated; it is based on "q.s."
solution to the approximate volume
o After mixing, you have to check the volumetric flask if it will reach the
mark. If not, you have to add more
DILUTION OF LIQUIDS
It may involve dilution of generally liquid solutions, which may be prepared
from: 1) a solid dissolved in a liquid to form a solution, 2) a liquid mixed
with a liquid to form a liquid solution, 3) dilution of alcohol preparation, and
4) dilution of acid
PHARMACEUTICAL CALCULATIONS| MODULE 3 10
If the tax on alcohol is P1.25 per wine gallon, how much tax must
be paid on 16 proof gallons that contain 20% (v/v) alcohol?
Solution
DILUTION AND FORTIFICATION / CONCENTRATION
OF SOLIDS AND SEMISOLIDS
As in the case of the preparation of ointments, lotions, or dilution semisolids
in the form of extracts from plant material.
PROOF STRENGTH
Proof Spirit is an aqueous solution containing 50% (v/v) of
absolute alcohol (100% v/v Ethyl alcohol)
Concentration of solutions can be expressed in different ways. Specifically
for alcohol, it is expressed as percentage by volume. Other ways of
expressing strength of solution containing alcohol:
V. ALLIGATON
ALLIGATION ALTERNATE
INTRODUCTION TO ALLIGATION Calculation of parts of 2 or more components of a given
strength that are to be mixed to produce a desired strength
Arithmetical method of solving problems o E.g., determine how many parts of solution A to be mixed with parts of
Mixing of solution or mixtures of solids possessing different solution B to get a mixture with specified strength or specific gravity
percentage strengths or specific gravity Alternative method when solving problems dealing with dilution
o Determine volume of diluent
Types o When using concentration x quantity, it only involves the concentration
o Alligation medial and quantity of the initial stock solution and final solution. It does not
o Alligation alternate involve the direct determination of the amount of diluent.
Determine the amount of the components to be mixed
ALLIGATION MEDIAL
Combination of several simple problems together Percent Percent Proportional parts
weighted average given desired required
of a mixture of 2 or more substances of known quantity and Higher A B D=X
concentration D
Computing for the specific gravity of a mixture of 2 or more solutions Lower B A D=Y
Determine the final strength/concentration or specific gravity of a mixture Arrange the given strength (percentage or specific gravity) of solutions from
highest to lowest.
EXAMPLE The desired strength should always be within the range of the highest and
lowest strength.
A mixture of alcohol contains the following:
a. 3000 mL of 40% v/v alcohol EXAMPLES
b. 1000 mL of 60% v/v alcohol
In what proportion should alcohols of 95% and 50%
c. 1000 mL of 70% v/v alcohol
strengths be mixed to make 70% alcohol?
Find the total % by volume of alcohol in the mixture
What is the specific gravity of a mixture of: How many milliliters each of a 2% w/v solution and a 7%
o 1000 mL of syrup with a specific gravity of 1.300, w/v solution should be used in preparing 1 gallon of a 3.5%
o 400 mL of glycerin with a specific gravity of 1.250, w/v solution?
o and 1000 mL of an elixir with a specific gravity of
0.950?
END
UNIVERSITY OF SANTO TOMAS
FACULTY OF PHARMACY
DEPARTMENT OF PHARMACY
PHARMACEUTICAL CALCULATION
INSTRUCTION: Select the correct answer for each of the following questions. Mark only one answer for each item
by shading the box corresponding to the letter of your choice on the answer sheet provided. STRICTLY NO
ERASURES ALLOWED. Use pencil no. 2 only.
5. ______ is the only unit in apothecary and avoirdupois system which has the same value and abbreviation
A. Fluidounce D. Milliliter
B. Gram E. Minims
C. Grain
8. The inhalant dose of amyl nitrate is 0.18 mL. Express the dose in minims
A. 3 D. 50
B. 30 E. 1.5
C. 2
9. One teaspoon is
A. 15 mL D. 20 mL
B. 5 mL E. 25 mL
C. 10 mL
12. A metric system prefix which denotes one-thousandth of the basic unit:
A. Milli D. Centi hundredth
B. Deci tenth E. Hecto 100
C. Kilo 1000
15. If 250 mg units of antibiotic weighs 1 mg, how many units are there in 15 mg?
A. 37.50 units D. 3.750 units
B. 16.66 units E. 1.660 units
C. 2.750 units
16. An aspirin tablet contains 1 – 1/5 grains aspirin. How many grams of aspirin are needed to make 1000
tablets?
A. 77 g D. 68 g
B. 78 g E. 69 g
C. 80 g
17. A formula for a cough syrup contains 1/8 gr. Of codeine phosphate per teaspoonful. How many grams
of codeine phosphate should be used to prepare a quart of the cough syrup?
A. 2.54 g D. 1.84 g
B. 1.74 g E. 2.74 g
C. 1.54 g
18. In dosing the drug gentamicin in pediatric patients, for every 1 mg/kg of gentamicin administered, serum
drug concentrations are expected to increase by 2.5 µg/mL. What would be the expected serum
following an administration of a 2.5mg/kg dose of gentamicin?
A. 5 µg/mL D. 2.5 µg/mL
B. 6.25 µg/mL E. 8.75 µg/mL
C. 10 µg/mL
19. A pediatric product contains 100 mg of erythromycin ethylsuccinate in each dropperful (2.5 mL) of the
product. How many kilograms of erythromycin ethylsuccinate would be required to prepare 5000 pint-
size bottles?
A. 74.6 kg D. 94.6 kg
B. 84.6 kg E. 96.5 kg
C. 94.5 kg
80 units per mL of the preparation
20. A physician places a patient on a daily dose of 48 units of U-80 insulin. How many mL should the patient
inject each day?
A. 0.4 D. 0.25
B. 0.5 E. 0.35
C. 0.6
21. How many two fluidounce bottles can be packaged from a gallon bottle of cough syrup?
A. 16 D. 64
B. 128 E. 48
C. 32
22. How many f.oz. bottles of Lagundi syrup can prepared from 5 gallons of the syrup?
A. 639 D. 740
B. 640 E. 760
C. 540
23. A 20-mL vial of biologic solution is labeled “2.0 megaunits”. How many units of drug are present in every
mL of solution? 10^6
A. 2000 D. 10,000
B. 1000 E. 20,000
C. 100,00
25. An adult patient is prescribed a daily dose of 80 mg aspirin as a precaution against heart attack. The
patient decides to cut 5-gr. Tablet into dosage units. 2How many doses can be obtained from each 5-gr.
Tablet?
A. 6 D. 3
B. 5 E. 2
C. 4
26. A physician orders Meprobamate 0.2 g. How much is to be administered if the dose on hand is 400 mg.
in each tablet?
A. Do not dispense D. Give ½ tablet
B. Give 2 tablets E. Give 1 ½ tablets
C. Give 1 tablet
27. A patient is to receive 260 µg of a drug 4 times a day for 14 days. How many 1/250 grain tablets must be
dispensed?
A. 66 D. 36
B. 56 E. 26
C. 46
28. The dose of a drug for children based on body surface area based on:
A. Weight and age D. Width and height
B. Weight and height E. Width and age
C. Age and height
29. The usual initial dose of chlorambucil if 150 µg per kg of body weight once a day. How many milligrams
should be administered to a person weighing 154 lbs?
A. 10.5 mg D. 8 mg
B. 18 mg E. 20 mg
C. 15 mg
30. An initial heparin dose of not less than 150 units/kg of body weight has been recommended of open
heart surgery. How many mL of an injection containing 5000 heparin units per milliliter should be
administered to a 300 pound patient?
A. 5.1 µL D. 4.1 mL
B. 4.1 µL E. 4.5 mL
C. 5.1 mL
31. What would be the dose for a 5-yr. old child weighing 30 lbs. if the adult dose is 50 mg? Use Clark’s rule.
A. 20 mg D. 25 mg
B. 15 mg E. 30 mg
C. 10 mg
32. The pediatric dose of cefadroxil is 30 mg/kg/day. If a child is given a daily dose of 2 teaspoonful of
suspension containing 125 mg of cefadroxil per 5 mL, what is the weight in lb. of the child?
A. 18.5 lbs. D. 18.1lbs.
B. 18.8 lbs. E. 18.0 lbs.
C. 18.3 lbs.
33. Apply Young’s Rule to determine the minimum dose of a 6-year old child weighing 25 kg, if the usual
morphine sulfate adult dose is 8 to 20 mg prn for pain.
A. Give 2.6 mg D. Give 5 mg
B. Give 6.6 mg E. Give 3.5 mg
C. Give 4.6 mg
34. If the loading dose of Kanamycin is 7 mg/kg of body weight, how many grams should be administered to
a patient weighing 130 lbs.?
A. 0.492 g D. 0.485 g
B. 0.414 g E. 485 g
C. 414 g
35. The adult dose of a liquid medication is 0.1 mL/kg of body weight as single dose. How many teaspoonfuls
should be given to a patient weighing 220 lbs.?
A. 2 tsp. D. 2.5 tbsp.
B. 2.5 tsp. E. 3 tsp.
C. 2 tbsp.
36. If glycerin (specific gravity 1.25) costs 131.00 per pint, how much would 10 lb. cost?
A. 105.90 D. 1005.90
B. 1105.90 E. 125.90
C. 1025.90
37. If a prescription order requires 25 g of concentrated HCl (density 1.18 g/mL), what volume should the
pharmacist measure?
A. 29.50 mL D. 21.2 mL
B. 0.0212 mL E. 2.95 mL
C. 23.0 mL
38. A perfume oil has a specific gravity 0.960 and cost 776.23 per kg. How much would 5 f.oz. cost?
A. 111.15 D. 180.15
B. 109.15 E. 191.15
C. 110.15
40. What is the weight in kilograms, of four (4) quarts sulfuric acid with a specific gravity of 1.845?
A. 6.96 D. 6.98
B. 5.96 E. 8.95
C. 5.98
41. A prescription calls for 0.3 g of phosphoric acid with a specific gravity of 1.71. How many milliliter should
be used in compounding the prescription?
A. 0.5 D. 0.3
B. 0.7 E. 0.6
C. 0.18
43. To prepare 2 fl.oz. of 3.5% solution, how many grains solute is required? (455 gr./fl.oz.)
A. 10.5 gr. D. 36 gr.
B. 31.85 gr. E. 28.25 gr.
C. 51.2 gr.
44. How many mL of 0.9% (w/v) NaCl solution should be prepared from 250 mL of 25% (w/v) solution?
A. 3750 D. 9
B. 2500 E. 900
C. 6944.4
45. A patient is determined to have 0.8 mg of glucose in each milliliter of blood. Express the concentration of
glucose in the blood as mg%
A. 800 mg % D. 80 mg %
B. 0.8 mg % E. 0.08 mg %
C. 8 mg %
47. Calculate the number kg of a 20% (w/w) solution which can be made from a kg. solute
A. 5.0 D. 50.0
B. 5000 E. 500.0
C. 0.5
48. If a patient is determined to have 100 mg % of blood glucose, what is the equivalent concentration in
terms of mg/dL?
A. 1 D. 100
B. 10 E. 140
C. 40
49. Strong Iodine Solution USP contains 5% w/v iodine. How many mg of Iodine are consumed daily if the
usual dose is 0.3 mL t.i.d.?
A. 15 D. 45
B. 90 E. 75
C. 22.5
50. Express in percentage the fluoride concentration in drinking given in 0.6 ppm..
A. 0.06 % D. 0.006 %
B. 0.00006 % E. 0.6 %
C. 0.0006 %
52. A Pharmacy aid adds 75 mL of Strong Iodine Solution USP (5.0 % w/v) to 1 liter of sterile water for irrigation.
What is the % w/v of Iodine present?
A. 0.35 % D. 0.60 %
B. 0.375 % E. 0.675 %
C. 0.53 %
53. How many grams of potassium citrate are needed to prepare 1 liter of 10%?
A. 1000 g D. 10 g
B. 50 g E. 500 g
C. 100 g
54. How many grams of a drug are required to make 120 mL of a 25 % solution?
A. 30 g D. 12 g
B. 10 g E. 20 g
C. 12.0 g
55. Calcium hydroxide topical solution contains 170 mg of calcium hydroxide per 100 mL at 15°C. Express the
concentration as ratio strength .
A. 1:688 D. 1:788
B. 1:888 E. 1:488
C. 1:588
56. How many mg of isofluorophate are contained in 15 g of a 1:10,000 ophthalmic solution of isoflurophate
in peanut oil?
A. 1.7 mg D. 1.5 mg
B. 1.9 mg E. 1.4 mg
C. 1.8 mg
59. A cupric chloride injection (0.4 mg Cu/mL) is used as an additive to IV solution for TPN. What is the final
ratio strength of copper in the TPN solution if 2.5 mL of the injection is added to enough of the IV solution
to prepare 500 mL?
A. 1:500 D. 1:50,000
B. 1:5000 E. 1:50
C. 1:500,000
60. How many milliliters of a 1:400 (w/v) stock solution should be used to make 4 liters of a 1:2000 (w/v)
solution?
A. 1000 mL D. 200 mL
B. 800 mL E. 400 mL
C. 1600 mL
61. How many milliliters of a 23.5% (w/v) concentrate of Sodium Chloride solution should be used in preparing
650 mL of a stock solution such that 30 mL dilute to liter will yield a 1:5000 solution?
A. 0.2 mL D. 11.75 mL
B. 4.33 mL E. 5.43 mL
C. 18.44 mL
62. You have a stock solution of 50% Sodium citrate and you were asked to prepare 300 mL of a 10% solution.
How many mL is needed?
A. 20 D. 60
B. 15 E. 75
C. 30
63. How many milliliters of 1:16 solution of sodium hypochlorite should be used in preparing 5,000 mL of 5%
solution of sodium hypochlorite for irrigation?
A. 800 mL D. 300 mL
B. 2500 mL E. 1250 mL
C. 4000 mL
65. How many grams of 10% (w/w) ammonia solution can be made from 1800g of 28% (w/w) strong ammonia
solution?
A. 6428.57 g D. 642.86 g
B. 5040 g E. 1620 g
C. 50,400 g
66. If a gallon of a 30% (w/v) solution is to be evaporated so that the solution will have a strength of 60% (w/v),
what will be its volume in fluidounces?
A. 76.8 D. 74.8
B. 70.8 E. 60.8
C. 64.0
67. How many grams of petrolatum should be added to 250 g of a 25% sulfur ointment to make a 5% ointment
A. 1000 g D. 100 g
B. 1250 g E. 50 g
C. 500 g
69. A cough syrup is labeled as containing 20% alcohol by volume. Which of the following sgtatement is (are)
true?
A. Each 100 mL of syrup contains exactly 20 mL Alcohol USP
B. The proof strength of this product is 40
C. There is the equivalent of 20 mL of absolute alcohol present
D. All of these
E. None of these
70. The amount of 190 proof required to make 500 mL of 70% alcohol is
A. 350 cc D. 368 cc
B. 520 cc E. 37 cc
C. 184 cc
72. The formula of an elixir calls for 4 gallons of 95% alcohol. Alcohol (95%) cost P 877.50 per gallon and the
drawback is P 310.50 per proof gallon. Calculate the net cost of the alcohol in the formula
A. P 1,153.20 D. P 1,150.80
B. P 1,115.50 E. P 1,115.80
C. P 1,150.20
73. Calculate the volume, in wine gallons, represented by 175 proof gallons of 70% (v/v) alcohol
A. 245 D. 122.5
B. 125 E. 145
C. 250
74. A pharmacist had, on the first month, 54 gallons of 95% alcohol. During the months, 35 gallons were used
in the manufacture of pharmaceutical products. How many proof gallons of alcohol were on hand at
the end of the month?
A. 36.5 D. 36.4
B. 36.3 E. 36.2
C. 36.1
75. Determine the specific gravity of a mixture of 900 mL of syrup with a specific gravity of 1.1898, 700 mL of
elixir with a specific gravity of 0.975 and 1150 mL of glycerin with a sp. Gr. Of 1.240.
A. 1.1349 D. 1.1561
B. 1.1468 E. 1.1651
C. 1.1486
76. In what proportion should 10% and 2% coal tar ointment be mixed to prepare a 5% ointment?
A. 3:5 D. 3:7
B. 2:5 E. 2:7
C. 2:3
77. What is the precentage alcohol in a mixture of 2000 mL of 50% (v/v) alcohol, 500 mL of 70% (v/v) alcohol
and 2.5 L of 95% (v/v) alcohol?
A. 71.67 % D. 74.5 %
B. 73.25 % E. 75.25 %
C. 72.5 %
78. How many mL of a syrup having a specific gravity of 1.350 should be mixed with 3000 mL of a syrup having
a specific gravity of 1.250 to obtain a product having a specific gravity of 1.310?
A. 3500 mL D. 5500 mL
B. 4500 mL E. 5800 mL
C. 4600 mL
79. If 800 g of 5% coal tar ointment is mixed with 1200 g of a 10% coal tar ointment. What is the concentration
of coal tar in the finished product?
A. 8. 5% D. 9 %
B. 9.5 % E. 10 %
C. 8 % %w/w
w
80. How many mL, of 95% alcohol and 30% alcohol should be mixed to make 4000 mL of 50 % alcohol
A. 1230 & 2770 D. 2000 & 2000
B. 2500 & 1500 E. 1500 & 2500
C. 2770 & 1230
81. How many grams of pure hydrocortisone powder must be mixed with 60 g of 0.5 % hydrocortisone cream
if one wishes to prepare a 2.0 % w/w preparation?
A. 0.90 D. 1.2
B. 0.92 E. 1.91
C. 0.30
82. How many gallons of water are needed to dilute 1 gallon of 6% concentration of antiseptic solution to
make2 % final concentration?
A. 1 gallon D. 1.5 gallons
B. 3 gallons E. 2.5 gallons
C. 2 gallons
83. How much water should be added to 2500 mL of 83% (v/v) alcohol to prepare 50 % (v/v) alcohol?
A. 1650 D. 1560
B. 1660 E. 1450
C. 1550
84. What is the precentage of alcohol in a mixture of 400 mL of 95% (v/v) alcohol, 1000 mL of 70 % (v/v)
alcohol and 0.6 L of 50 % (v/v) alcohol?
A. 38 % D. 75 %
B. 69 % E. 88 %
C. 7.5 %
85. An order calls for 500 mL of a solution of potassium sulfate to be made dso that it contains 10 mEq of
potassium sulfate. How many grams of potassium sulfate is required to prepare the solution?
A. 0.495 D. 0.440
B. 0.870 E. 0.680
C. 4.44
86. How many mL of a 3% (w/v) solution of ammonium chloride (MW = 53.5) should be administered
intravenously to provide 50 mEq?
A. 89.2 D. 99.2
B. 79.2 E. 59.2
C. 69.2
87. How many mEq of Magnesium sulfate (MW = 120) are represented in a 5-mL dose of 5% (w/v) MgSO4
solution?
A. 4.2 D. 3.0
B. 2.5 E. 3.6
C. 2.1
89. A solution contains 10 mg % Mg2+ (at. Wt. = 24). How many mEq are there in 1 liter?
A. 4.33 D. 8.55
B. 8.33 E. 9.23
C. 6.21
90. How many milligrams of Magnesium sulfate (MW 120) should be added to an IV solution to provide 5 mEq
of Mg2+ per liter?
A. 300 mg D. 200 mg
B. 210 mg E. 320 mg
C. 307 mg
91. How much elemental iron is present in 300 mg ferrous sulfate? (MW: FeSO4 7H2O = 278; At. Wt = Fe= 55.9;
S = 32.1; O = 16; H = 1.0)
A. 120.6 mg D. 30.2 mg
B. 60.3 mg E. 130 mg
C. 110 mg
92. How many milliequivalents of potassium chloride are represented in a 15 mL dose of a 10% (w/v)
potassium chloride elixir?
A. 20.1 mEq D. 167 mEq
B. 60 mEq E. 248 mEq
C. 0.06 mEq
93. Calcium chloride (CaCl2 2H2O) has a formula weight of 147. What weight of the chemical is needed to
obtain 40 mEq of calcium? (Ca = 40; Cl = 35.5 ; H2O = 18)
A. 0.8 g D. 2.94 g
B. 2.22 g E. 3.75 g
C. 1.47 g
94. The normal potassium level in the blood plasma is 17 mg %. Express the concentration in terms of
milliequivalent per liter.
A. 1.7 D. 4.36
B. 2.29 E. 2.42
C. 3.90
95. How many mL of glycerin would be needed to prepare 1 lb. of an ointment containing 5% w/w glycerin?
The density of glycerin is 1.25 g/mL?
A. 1.2 D. 24
B. 22.7 E. 12
C. 18.2
96. Twelve (12) bottlles of 100 tablet multivitamins cost P3,000 when bought on a promotional deal. If the
deals sell for P 450 per bottle, what percent of gross profit is realized on the selling price?
A. 45 % D. 44.44 %
B. 40.44 % E. 34.5 %
C. 35.44 %
97. An antihypertensive drug list at P 180.00 per dozen tablets, less a discount of 33-1/3 % for a purchase of
100 dozens, plus an additional promotional discoubnt of 10%. Calculate the net cost per unit.
A. P 9.00 B. P 9.15
C. P 9.25 E. P 9.75
D. P 9.50
98. A pharmacist receives a bill of goods amounting to P 35,400 less a 5% discount for quantity buying and a
2% cash discount for paying the bill within 10 days. What is the net amount of the bill?
A. P 32,950.40 D. P 32,951.60
B. P 32,957.40 E. P 32,937.60
C. P 32,930.40
99. Find the difference in the net cost of a bill of goods amounting to P 100,00 if the bill is discounted at 45%
and if it is discounted at 33 % and 12%
A. P 3,960 D. P 3,750
B. P 3,860 E. P 3,650
C. P 3,550
100. A prescription item cost P 13.50. Using a mark-up of 590% on the cost, What would be the selling price
for the prescription?
A. P 79.65 D. P 9315
B. P 93.15 E. P 7350
C. P 7965
—-END—-
SNS OVERACTIVITY
Affects both PVR and CO, which is why some drugs affect the SNS to lower
blood pressure.
In the last few years (2017 or 2018), they used JNC-8 Guideline as the
FIRST LINE DRUGS FOR HYPERTENSION definition for hypertension, but since we now have different categories for
The strategy for the treatment of hypertension would be add-on. hypertension, you need to memorize them.
These are the first line drugs since they affect both factors (PVR and CO).
1. Calcium Channel Blockers (CCB) BASED ON OFFICE BLOOD PRESSURE (BP) MEASUREMENT
2. ACE Inhibitors (ACEi) or Angiotensin II Receptor Blockers (ARB)
International Society of Hypertension (ISH) Guidelines
3. Diuretics (particularly thiazides)
o Decreases cardiac output by promoting the excretion of sodium.
Systolic Diastolic
Category
(mmHg) (mmHg)
Normal BP <130 and <85
High-normal BP 130-139 and/or 85-89
Grade 1 hypertension 140-159 and/or 90-99
Grade 2 hypertension and/or
VARIATION IN GUIDELINES
SYMPATHETIC ACTIVATION Due to different population characteristics.
Affects both PVR and CO. Usually normal BP is <120/80 for most guidelines except ISH (<130/85)
Since beta blockers affect sympathetic activation, they are no longer included while <130/85 is already considered hypertensive.
in the first line of drugs for hypertension.
o Beta blockers are 4th in line for the treatment of hypertension.
CLINICAL PHARMACY | MODULE 3 2
STRATEGY
A Start one drug, titrate to maximum dose, and then add
a second drug.
B Start one drug, then add a second drug before
achieving max dose of the first drug.
In other guidelines they immediately start with dual therapy but since JNC
8 is not updated, dual therapy is not the initial step.
C Begin 2 drugs at same time, as separate pills or
combination pill. Initiate combination therapy is
recommended if BP is greater than 20/10 mmHg above
Similar to ISH treatment strategy goal. Most drugs in the market are already a combination therapy.
Initial therapy (dual combination)
LIFESTYLE CHANGES
ACEi or ARB + CCB or diuretic
Consider monotherapy in low-risk grade 1 hypertension or in very old Smoking cessation
Control blood glucose and lipids
Step 2 (triple combination)
Diet
ACEi or ARB + CCB + diuretic o Eat healthy (i.e., DASH diet)
Step 3 (triple combination + Spironolactone or other drug) o Moderate alcohol consumption
For resistant hypertension o Reduce sodium intake to no more than 2,400 mg/day
Add spironolactone (25-50 mg od) or other diuretic, Physical activity
alpha blocker, or beta blocker o Moderate to vigorous activity 3-4 days a week
Consider referral to a specialist center for further investigation averaging 40 min per session
CLINICAL PHARMACY | MODULE 3 3
WHAT ARE THE PREFERRED DRUGS FOR THE TREATMENT Previous Stroke RAS blockers
OF HYPERTENSION AMONG ADULT FILIPINOS FOR CCBs
PREVENTION OF CV DISEASES? Diuretics
Statements:
Heart Failure (HF) RAS blockers, beta blockers, and
4.2.1 Among persons with uncomplicated hypertension,
mineralocorticoid receptor
angiotensin-converting enzyme (ACE) inhibitors or
antagonists (e.g., Spironolactone) are
angiotensin-receptor blockers (ARBs), calcium channel
all effective in improving clinical
blockers, thiazide/thiazide-like diuretics are all suitable
outcome in patients with
first line antihypertensive drugs, either as monotherapy or
established HFrEF
combination.
o ACEi or ARB, CCB, thiazide like diuretic. CCBs are indicated on in case of
4.2.2 Ideal combination therapy includes renin- poor BP control
angiotensin-system (RAS) blocker with calcium channel- Angiotensin receptor-neprilysin
blocker (CCB) or thiazide/thiazide-like diuretics. Other inhibitor (e.g., ARNI, sacubitril-
combinations of the five major classes may also be used in valsartan) is indicated for the
patients with compelling indications for the use of specific treatment of HFrEF as an
drug classes. alternative to ACE inhibitors or
o RAS blocker with CCB or thiazide like diuretic. ARBs also in hypertensive
o Compelling indications = comorbidities. populations
4.2.3 ACE inhibitors & ARBs are not recommended to be Chronic Kidney RAS-inhibitors because they
used in combination. Likewise, combinations of ACE-I or Disease (CKD) reduce albuminuria in addition to
ARBs with direct renin inhibitors (e.g., Aliskiren) should not be BP control
used. CCBs and diuretics (loop-diuretics
4.2.4 The use of free combinations is recommended if if eGFR <30 mL/min/1.73m2) can
single-pill combination therapy is not available or not be added
affordable. Chronic The treatment strategy should
Obstructive include an angiotensin AT1 -
EXACERBATORS AND INDUCERS OF HYPERTENSION Pulmonary receptor blocker (ARB) and CCB
Most common medications that can increase BP: Disease (COPD) and/or diuretic
Non-selective or traditional NSAIDs Beta blockers (ß1-receptor
Combined oral contraceptive pill selective) may be used in selected
Select anti-depressant medications including tricyclic patients (e.g., CAD, HF)
antidepressants and SNRIs Diabetes The treatment strategy should
Acetaminophen when used almost daily and for prolonged include an RAS inhibitor (and a
periods CCB and/or thiazide-like diuretic)
The effect of anti-retroviral therapy is unclear as studies Populations From Single pill combination including a
demonstrate either no effect on BP or some increase African Descent thiazide-like diuretic plus CCB or
Alcohol raises BP regardless of the type of alcoholic drink CCB plus ARB
Limited evidence on herbal and other substances Among RAS-inhibitors, ARBs
Ma Huang, Ginseng at high doses and maybe preferred as angioedema
reported to increase BP is about 3 times more likely to
occur with ACE inhibitors among
HYPERTENSION-MEDIATED ORGAN DAMAGE (HMOD) black patients
This is what is being prevented in the timely management of hypertension.
Defined as the structural or functional alteration of the Summary
arterial vasculature and/or the organs it supplies that is ACEI or ARB For most patients with CKD, HF, diabetes.
caused by elevated BP. Except for pregnant and black patients.
Thiazide Not preferred for pregnant patients, CAD, heart failure,
End organs include the brain, the heart, the kidneys, central CKD, diabetes, gout, metabolic syndrome.
and peripheral arteries, and the eyes. diuretics CKD: Loop diuretic is recommended based on the GFR.
Mnemonic: HyPer KA BEH Diabetes: Still an option since it could prevent CV
disorders, but is not recommended since it could increase
insulin resistance.
FIRST LINE DRUG THERAPY OPTIONS Gout: Thiazides prevent the excretion of uric acid.
FOR HYPERTENSION WITH CONDITIONS Calcium For most patients even those with pregnancy, CAD,
Pregnancy Methyldopa Channel previous stroke, heart failure, kidney disorders, COPD,
diabetes, African descendants.
Beta blockers (labetalol) Blockers
Dihydropyridine-Calcium Channel Beta For patients with CAD and HF.
Blockers For pregnant patients: Labetalol.
Blockers (DHP-CCBs)
ACEI and ARB should not be given to
hypertensive pregnant patients. HYPERTENSION AND PSYCHIATRIC DISEASES
Coronary Artery RAS blockers The prevalence of hypertension is increased in patients with
Disease (CAD) Beta-blockers irrespective of BP psychiatric disorders and in particular depression
levels with or without calcium According to guidelines, psychosocial stress and major
channel blockers (CCBs) psychiatric disorders increase the cardiovascular risk
CLINICAL PHARMACY | MODULE 3 4
Main drugs affecting RAAS: Aliskiren, ACE inhibitors, and AT1 receptor blockers.
DIURETICS
VASODILATORS
Type Drugs
Hydralazine
Arterial vasodilators
Minoxidil
Nitroprusside
Nitroglycerin
For emergency hypertension affecting heart.
Verapamil
CCB block influx of calcium in vascular smooth muscle and
Amiodipine
cardiac muscle.
Arterial and venous Clevidipine
Intracellularly, there is decreased muscular contraction
vasodilators Felodipine leading to reduced blood pressure through:
Calcium channel
blockers Isradipine o Heart: Reduced cardiac contraction, cardiac output.
Nisoldipine o Blood vessels: Reduced vasoconstriction, peripheral
Nicardipine resistance.
Nifedipine
Ditiazem
CLINICAL PHARMACY | MODULE 3 7
HEART FAILURE
It is a syndrome of reduced cardiac output (CO) resulting
from impaired ventricular ejection, impaired filling or
components of both.
o In some cases, if you have reduced cardiac output, it could also be due Diastolic Heart Failure Systolic Heart Failure
to high demand, but not necessarily because of heart failure. Impaired filling Impaired contraction
Types of heart failure
1. HF with reduced ejection fraction (HfrEF) was REDUCED EJECTION FRACTION (<55%)
formerly known as systolic dysfunction Left ventricle does not eject sufficient blood
2. HF with preserved ejection fraction (HfpEF) was
formerly known as diastolic dysfunction Heart muscle is weak and not pumping properly
E.g., Instead of 50-60 mL, the heart is pumping less because it is week.
HF AND EJECTION FRACTION
NORMAL EJECTION FRACTION (>55%)
The amount of blood ejected (Left ventricular ejection
fraction)
ACCF/AHA NYHA
Stages Of HF Functional Classification
A At high risk for HF None
but without structural
heart disease or
symptoms of HF
ACCF/AHA STAGES B Structural heart I No limitation of physical
Based on: Risk of developing heart failure. disease but without activity.
Recognizes the risk and structural abnormalities of the cardiac muscle. signs or symptoms of Ordinary physical
Increase in letter = increase in severity.
HF activity does not cause
symptoms of HF.
Stage Risk C Structural heart I No limitation of physical
A High risk for developing HF disease with prior or activity.
No structural disease of the heart current symptoms of Ordinary physical
No symptoms of HF HF activity does not cause
B Structural disease symptoms of HF.
No symptoms of HF II Slight limitation of
C Structural disease physical activity.
Past or current symptoms of HF Comfortable at rest but
D Refractory HF ordinary physical
End stage of HF Requires specialized treatment strategies activity results in
symptoms of HF.
NYHA (NEW YORK HEART ASSOCIATION) III Marked limitation of
FUNCTIONAL CLASSIFICATION physical activity.
Based on: Exercise tolerance or the ability to perform physical activities.
Comfortable at rest, but
Subjective assessment by the clinician.
less than ordinary
physical activity causes
Annual
Class Risk symptoms of HF.
Mortality
IV Unable to carry on any
I No limitation of physical activity (no 5-10%
symptoms) physical activity without
Ordinary physical activity does not symptoms of HF, or
cause symptoms of HF symptoms of HF at rest.
II Slight limitation of physical activity 5-10% D Refractory HF IV Unable to carry on any
(mild) requiring specialized physical activity without
Comfortable at rest interventions symptoms of HF, or
symptoms of HF at rest.
CLINICAL PHARMACY | MODULE 3 9
FRANK-STARLING RELATIONSHIP
Described the compensatory mechanism for heart failure.
Increase in quantities of blood flow into the heart = the walls of the heart
stretches = increase in contractility/force.
Over time, it will not be healthy for the heart due to the thickening of walls.
HF WITH REDUCED EJECTION FRACTION
MAIN PURPOSE OF MANAGEMENT OF HEART FAILURE Start with Beta Blockers and ACE inhibitors together for heart failure with
Prevent cardiac remodeling or Frank-Starling relationship. reduced ejection fraction.
o Cardiac remodeling is a change in heart size, shape, structure, and While for preserved ejection fraction, there are no specific guidelines.
function of the heart. Therefore, treatment management is based on the reduced ejection fraction.
o This is because of the compensatory mechanism of our body over time.
o Hypertrophy is an increased volume of tissue causing the enlargement
of the heart. FOR SYMPTOMATIC HEART FAILURE
In checking for heart failure, first thing is to have a Chest X-ray. First line: ACE Inhibitor + Beta Blocker (can be started at the same time) then
o It will check if your heart has increased in size or not based on the titrate to maximum dose.
diameter of the heart from left to right. : add MRA (e.g., Spironolactone and
Eplerenone, which are potassium sparing diuretics).
CLINICAL PHARMACY | MODULE 3 10
For fluid congestion: Loop diuretics (e.g., Furosemide) are used to relieve the ANGIOTENSIN RECEPTOR NEPRILYSIN INHIBITOR (ARNI)
signs and symptoms of congestion.
o Loop diuretics can still be given even if another diuretic, Spironolactone,
is already being taken.
HEART FAILURE
Cornerstone: ACE + beta-blockers.
If there is congestion, loop diuretics would be given.
If still symptomatic, MRA will be given.
If ACEI is tolerated, shift the ACEI or ARB to ARNI (Sacubitril + Valsartan).
If the heart rate is high ( 70), give Ivabradine.
CARDIAC ENZYMES/MARKERS When the patient complains about chest pain, ECG & troponin is tested.
o If troponin is positive, there could be STEMI or Non-STEMI.
Biomarkers measured to determine is muscle damage is STEMI ECG shows ST-segment elevation.
occurring: NSTEMI ECG without ST-segment elevation.
o Creatinine Kinase (CK) o If troponin is negative, there could be Stable or Unstable Angina.
o CK-MD (more specific for heart) When there are ECG changes, that is Unstable Angina.
o Troponins (most specific for heart)
ANTICOAGULANT AGENTS
FONDAPARINUX
Fondaparinux is preferred if the patient is at an increased
risk of bleeding or has a history of heparin-induced
thrombocytopenia (HIT), but only if PCI is not planned.
Fondaparinux is contraindicated in patients with a
creatinine clearance (CrCl) less than 30 mL/min
BIVALIRUDIN
Bivalirudin is recommended if PCI is planned, as a bridge
to PCI
Given for invasive strategy for NSTEMI.
REVASCULARIZATION THERAPY
Invasive procedure for STEMI since the artery is totally occluded.
Fibrinolytics catalyzes the conversion of endogenous
plasminogen to plasmin, which degrades fibrin and results
in lysis of the fibrin meshwork
o Alteplase
o Reteplase
o Tenecteplase
It is vital that this vessel be revascularized in the shortest
time possible from the onset of symptoms to decrease
morbidity and mortality
Home medications that increase the risk for coronary events
should be discontinued at the time of ACS event (e.g.,
hormone replacement therapy and NSAIDs)
CLINICAL PHARMACY | MODULE 3 15
INTENSITY OF STATIN THERAPY BY DRUG AND DOSE FOUR MAJOR STATIN-BENEFIT GROUP
High-Intensity Moderate-Intensity Low-Intensity Clinical ASCVD
Therapy Therapy Therapy
Daily dose lowers LDL Primary elevations of LDL-
Daily dose lowers LDL Daily dose lowers LDL
on average by 30%
on average by <30% Diabetes 40-75 years of age and LDL-C between 70-189
to <50% mg/dL without clinical ASCVD
Atorvastatin 10-20 mg
Rosuvastatin 5-20 mg Simvastatin 10 mg Without clinical ASCVD or diabetes, 40-75 years of age,
Simvastatin 20-40 mg Pravastatin 10-20 mg with LDL-C 70-189 mg/dL and estimated 10-y ASCVD risk
Atorvastatin 40-80 mg Pravastatin 40-80 mg Lovastatin 20 mg
Rosuvastatin 20-40 mg Lovastatin 40 mg Fluvastatin XL 80 mg
Fluvastatin XL 80 mg Fluvastatin 20-40 mg
Fluvastatin 40 mg BID Pitavastatin 1 mg RECAP
Pitavastatin 2-4 mg Dyslipidemia is just a risk factor. The goal is to prevent cardiovascular
*Simvastatin is not recommended by the U.S. Food and Drug Administration
(FDA) to be started at 80 mg/day due to increase risk of myopathy and recommended level of cholesterol. Ideally,
rarely rhabdomyolysis. o LDL: 100 (if you have ASCVD) or 130 (optimal if you do not have
Moderate- or high-intensity statin therapy is ASCVD)
o LDL-C:100 or below
recommended depending on group classification. o Triglycerides: 150
High-intensity statin is preferred for most patients with o Total cholesterol: 200
clinical ASCVD, LDL-C more than 190 mg/dL or more, or o HDL: 60
diabetes with estimated 10-year risk of more than 7.5% or You want a better value for your HDL. Higher amount is better.
Statins are the first-line drug in any type. However, the main problem with
more. the use of statins is Statin-Associated Muscle Symptoms (SAMS)
Moderate-intensity statin is preferred for patients with o Myalgia: muscle pain
clinical ASCVD over 75 years of age (although reasonable o Rhabdomyolysis: breakdown of skeletal muscles; increase in creatinine
to continue high-intensity if tolerable), diabetes and 10- kinase
There are two (2) pathways: exogenous and endogenous. The main
year risk of less than 7.5%, or who are otherwise not mechanism of action of the drugs is to limit the cholesterol in the liver. There
candidates for high-intensity therapy. is cholesterol absorbed from food intake and the liver-produced endogenous
Either high- or moderate-intensity statin therapy is cholesterol. The goal is to limit the absorption from the food by the intestine,
so no cholesterol goes to the liver. When there is a low amount of cholesterol
reasonable for patients without clinical ASCVD or diabetes in the liver, it will increase the activity of the LDL receptor so it will repeatedly
who have estimated 10-year risk of more than 7.5% or collect LDL from the plasma.
more.
CLINICAL PHARMACY | MODULE 3 19
BILE ACID SEQUESTRANTS Its primary effect is modest reduction in LDL-C (15%-24%)
with higher reductions achievable in combination with statin
therapy.
It reduces LDL-C by inhibiting the NPC1L1 protein, an
important transporter of cholesterol absorption in the small
intestine and hepatocytes.
It is generally well tolerated and is associated with mild
gastrointestinal (GI) complaints (e.g., diarrhea), myalgia,
and ALT (alanine transaminase) elevations when used with
Because the body uses cholesterol to make bile acids, this statins.
reduces the amount of LDL cholesterol circulating in the It has no effects on the CYP450 enzyme system
blood.
In the GI tract, bile acid sequestrants will combine with the bile acids and form MECHANISM OF ACTION
a complex, which will then be excreted. 15-30% of bile salts are excreted in
the feces. With the help of bile acid sequestrants, more cholesterol will be
excreted and a lower amount will be transported back to the liver.
BILIARY SECRETION
Bile is alkaline, bitter-taste, contains: It would inhibit just the absorption of cholesterol from the intestine. When this
happens, there is no cholesterol that will go back deliberate to the liver. The
o Bile salts goal is always to reduce hepatic cholesterol.
o Cholesterol
o Bilirubin ACLY (ATP CITRATE LYASE) INHIBITORS
o Electrolytes
o Water
Functions are:
o Emulsification
o Neutralization of acids
o Excretion of drugs and toxins
One drug to lower your cholesterol is bile acid sequestrants
The biliary secretion is the largest source of cholesterol. It is also one of the
major pathways for elimination of cholesterol. Cholesterol is synthesized in
the liver, and is the precursor of bile acid. The bile acid is a component of
bile.
Cholesterol bile acid bile
Bile is stored in the gallbladder and will only be released in the duodenum
during digestion. The bile is supplied to duodenum to help lipid absorption.
The bile is then reabsorbed in the GI tract for recirculation back to the liver
but other significant amount of bile acids is also secreted in the feces (15-
30% of bile salts excreted in the feces).
Bempedoic acid, a new drug, has the same activity as statins, it will also inhibit
cholesterol synthesis but with different targets. Statins work on HMG-CoAR
MECHANISM OF ACTION and bempedoic acid works on ACLY.
1. Bind bile acids in the intestine-forming complex
2. Loss of bile acids in the stools PCSK9 (PROPROTEIN CONVERTASE SUBTILISIN/KEXIN TYPE
3. Increase in conversion of cholesterol into biles acids in the 9 (PCSK9) INHIBITORS
liver The FDA has only approved Alirocumab and Evolocumab
4. Decreased concentration of intrahepatic cholesterol for people with genetic high cholesterol or serious heart
5. Compensatory increase in LDL receptors disease including those who
6. Increase hepatic uptake of circulating LDL or stroke and who need their LDL cholesterol lower.
7. Decrease serum LDL cholesterol levels There are limited options for patients who are either
intolerant to statin therapy, develop CVD despite being on
EZETIMIBE maximally tolerated statin therapy, or have severe
It is a preferred adjunct therapy because it modestly hypercholesterolemia
reduces the risk of recurent CV events when used in PCSK9 inhibitors are given as a shot every 2 or 4 weeks,
combination with statin therapy. to be injected in the upper arm, stomach, or upper thigh
CLINICAL PHARMACY | MODULE 3 20
Alirocumab and Evolocumab are fully human monoclonal Despite these favorable changes, Niacin has not been
antibodies to PCSK9. shown to improve CV outcomes in patients already receiving
Inhibiting PCSK9 promotes intracellular degradation of statin therapy with relatively well-controlled lipids at
hepatic LDL-C, prevents LDL receptor recycling to the cell baseline.
surface, and reduces LDL-C clearance from the circulation.
They reduce LDL-C by as much as 60% when added to MECHANISM OF ACTION
statin therapy. It is a potent inhibitor of lipolysis in adipose tissues
The drugs are administered via subcutaneous injection either mobilization of FFAs (major precursor of TGs) to the liver
biweekly or once monthly. VLDL and eventually LDL (after few hours).
It serves as maintenance drug every month HDL levels (the most potent antihyperlipidemic)
MECHANISM OF ACTION ADVERSE EVENTS
Due to frequent Niacin use.
Cutaneous flushing and itching appear to be prostaglandin
mediated and can be reduced by taking Aspirin 325 mg
shortly before Niacin ingestion.
Flushing seems to be related to rising plasma Niacin
concentrations and use of immediate-release formulations;
taking the dose with meals and slowly titrating the dose
upward may also minimize these effects.
o Side effects of some statins (ezetimibe) are more on diarrhea
o Niacin side effect is cutaneous flushing and itching
Niacin therapy may be associated with elevated hepatic
enzyme hyperuricemia, and hyperglycemia
It is contraindicated in patients with active liver disease and
active peptic ulcer disease
o The dose must be decreased to prevent adverse effects.
The activity of PCSK9 inhibitors is on the LDL receptor. It can be taken on its FENOFIBRATES
own or as an additive to statins. However, it is expensive. It is only used when Gemfibrozil and fenofibrate are potent TG-lowering
the patient is intolerant to statins. It is a new drug and is usually injected once
a month, or 2 or 4 weeks. therapies (20%-50%) but may cause a modest reciprocal
PCSK9 binds to LDL receptors causing them to be rise in LDL-C in patients with severely elevated G levels.
o Avoid in patients with hypercholesterolemia.
degraded.
Plasma HDL concentrations may rise 10%-15% or more with
PCSK9 inhibitors: Alirocumab (Praluent) and fibrates.
Evolocumab (Repatha). These drugs bind to the PCSK9
receptors and inactivate it while LDL receptors remain intact. Gemfibrozil increases the activity of lipoprotein lipase (LPL)
and reduces secretion of VLDL from the liver into the plasma.
LDL is removed from the blood and levels increase
Fenofibrate increases LPL activity and reduces apoprotein
NIACIN C-III (an inhibitor of PL) by activating peroxisome-
proliferator-activated receptor (PPAR ), which regulates
the expression of genes involved in the regulation of lipids
and other metabolic processes.
Fibrates are primarily used in patients with TG levels of
>500mg/dL (5.65 mmol/L) to reduce the risk of
pancreatitis.
RECAP
Niacin is used for treatment of triglyceridemia
The niacin decreases the free fatty acid release. Lipoprotein lipase releases
free fatty acid where it will proceed to the adipose tissue then it will return to
the liver
If the free fatty acid is decreased, the triglyceride synthesis in the liver is also
decreased. This will result in:
o Decreased VLDL,
o Decreased LDL,
o Increased HDL
Niacin (nicotine acid) lowers TG levels (20%-50%) by
inhibiting lipolysis with a decrease in free fatty acids in
plasma and decreased hepatic esterification of TG.
It also significantly raises HDL-C (5%-30%) by reducing
its catabolism and decreasing hepatic removal.
Niacin reduces hepatic synthesis of VLDL, leading to a
modest dose-dependent decrease in LDL-C (5%-20%).
CLINICAL PHARMACY | MODULE 3 21
Statins and ACL inhibitors (bempedoic acid) reduce the synthesis of cholesterol Statin therapy is not recommended in pre-menopausal
in the liver. These are the rate-limiting step in the cholesterol synthesis, thereby
decrease the LDL activity.
patients with DM who are considering pregnancy or not
Niacin and nitrates will act on the VLDL. using adequate contraception.
Niacin and the fibrates will act on the lipoprotein lipase due to the free fatty For patients who present with an ACS, and whose LDL-C
acids to be used by the peripheral tissues, but no free fatty acids will return levels are not at goal despite already taking a maximally
to the liver. tolerated statin dose and ezetimibe, adding a PCSK9
Ezetimibe and the bile acids are in the GIT; this is why their side effects would
be on the GIT.
inhibitor early after the event (if possible, during
The monoclonal antibodies will act on the PCSK9 receptors. hospitalization for the ACS event) should be considered.
For secondary prevention, patients at very-high risk not
OMEGA-3 POLYUNSATURATED FATTY ACIDS (PUFA) achieving their goal on a maximum tolerated dose of statin
High doses of omega-3 PUFA (2-4 g/day of EPA/DHA) and ezetimibe, a combination with a PCSK9 inhibitor is
significantly reduce TG and VLDL cholesterol levels (20%- recommended.
50%), but PUFA supplementation has either no effect on TG If the LDL-C goal is not achieved after 4 - 6 weeks despite
and LDL-C or may cause slight elevations maximal tolerated statin therapy and ezetimibe, addition
Omega-3 PUFA reduces TG levels by increasing hepatic of a PCSK9 inhibitor is recommended.
oxidation free fatty acids, increasing LDL hydrolysis by Statin treatment is recommended as the first drug of choice
activating PPAR , an inhibiting apoprotein C-Ill. for reducing CVD risk in high-risk individuals with
hypertriglyceridaemia [TG >2.3 mmol/L (200 mg/dL)].
PHILIPPINE GUIDELINES FOR DYSLIPIDEMIA
For individuals without diabetes aged 45 years with LDL- TAKEAWAY POINTS
C 130mg/dL AND 2 risk factors*, without atherosclerotic Dyslipidemia, particularly elevated LDL-C, is strongly
cardiovascular disease, statins are RECOMMENDED for the associated with ASCVD.
prevention of cardiovascular event. Considered optimal:
For individuals with diabetes without evidence of ASCVD, o TC <200,
statins are RECOMMENDED for primary prevention of o LDL-C <100,
cardiovascular events. o HDL-
For individuals identified to have familial o TG <150 mg/dL
hypercholesterolemia, statin therapy is STRONGLY Therapeutic lifestyle changes are the foundation of care
RECOMMENDED for the prevention of cardiovascular for dyslipidemia.
events. Statins are first-line drug therapy for dyslipidemia,
Among individuals with chronic kidney disease who are not reducing both cardiovascular and all-cause mortality.
on dialysis, statins are RECOMMENDED for primary Niacin, fish oil, or gemfibrozil may be considered for
prevention of cardiovascular events. patients with TG>500 mg/dL.
For individuals with ACS, early high-intensity statin that is Combining statins with other lipid-lowering medications
maximally tolerated is RECOMMENDED and should not be increases risk of adverse effects.
continued Four major groups of patients benefit from statin therapy
Statins should be given to ACS patients immediately. with an acceptable margin of safety.
For individuals with documented ACS, and target LDL-C has High-intensity statin therapy is preferred for most patients
not been reached despite maximally tolerated high- with clinical ASCVD or with higher ASCVD risk.
intensity statin therapy, ezetimibe may be added on top of Fatal rhabdomyolysis with statins is rare. Myopathy is
the statin therapy to get to goal LDL-C. usually associated with preventable drug interactions or
Among individuals without diabetes not at goal LDL-C, underlying patient risk factors.
routinely adding fibrates on top of statin therapy is NOT Statins and BAS are approved for treatment of
RECOMMENDED for primary or secondary prevention of hyperlipidemia in children.
cardiovascular disease because statins have no activity on the LDL-C. PCSK9 inhibitors and ezetimibe may be considered in
Among individuals with diabetes, routinely adding fibrates high-risk patients intolerant of statins or with very high LDL-
on top of statin therapy is NOT RECOMMENDED for C on maximally tolerated therapy
primary or secondary prevention of cardiovascular disease.
However, adding fibrates to statins may be considered
among MEN with controlled diabetes, low LDL-C (<35
mg/dL) persistently high triglycerides (>200 mg/dL) for
prevention of CV disease.
Among individuals with ASCVD, omega fatty acids
(EPA+DHA) given on top of statin therapy is NOT
RECOMMENDED
Among individuals with ASCVD on statin therapy at goal
LDL-C, but with persistently high triglyceride levels of 150-
499 mg/dl, omega fatty acids (pure EPA) MAY be given.
Intensification of statin therapy should be considered before
the introduction of combination therapy.
If the goal is not reached, statin combination with
ezetimibe should be considered.
CLINICAL PHARMACY | MODULE 3 22
END
CLINICAL PHARMACY | MODULE 3 23
CREATININE CLEARANCE
Creatinine clearance is the amount of blood filtered by
kidneys in each minute in order to make blood free of
creatinine
In a healthy woman, the creatinine clearance is about 95 mL
per minute. In a health man, the creatinine clearance is 120
Example: if a patient is 64kg, he is at risk at what urine output? 0.5 mL/kg mL per minute
per hour x 6 hours.
0.5 mL x 64 kg x 6 = 192 mL for 6 hrs The main function of the creatinine clearance is to predict renal function
For Unlike with GFR that uses blood, the creatinine clearance on the other hand,
End-stage, because it is no longer acute renal injury. Usually, when it is severe, uses urine.
it would at the end-stage renal disease If the physician would require to determine the creatinine clearance, the urine
shall be collected for 24 hours, so that the levels of creatinine can be
CLINICAL PHARMACY | MODULE 3 24
(CKD-EPI) EQUATION
Creatinine
GFR
Clearance
Creatinine clearance Glomerular
measures the filtration rate is
amount of creatinine the rate at which
Definition
that is removed from glomerular
blood in the kidneys filtration takes
per minute place in kidneys
If you have a low clearance, it means that the drugs accumulate in the body, Urine test urine
so, you must adjust the dose. collected during 24 Blood creatinine
Type of Test
The Cockcroft and Gault used to adjust the dosing for renal function hours is used for levels
analysis
COCKCROFT-GAULT EQUATION
PATHOPHYSIOLOGY
AKI is classified into three types based on location and type of
injury:
PRERENAL
Where: Scr is in milligrams per deciliter (mg/dL) and W is Decreased renal blood flow; little blood goes to the kidney
weight Damage or defect occurring before the kidney and is the
Used for CrCl estimation for drug dosing adjustment most commonly encountered type of AKI
o
Equation depends on patient body weight Wt
For underweight patient: INTRINSIC
o Actual body weight is used Structural damage within the kidney
For obese patient (>30% of IBW) Intrinsic AKI is direct damage to one or more areas of the
Adjusted body weight is used kidney, including the glomerulus, the tubules, or the
Cockcroft-Gault equation: Used to adjust the dose interstitium. Examples include glomerulonephritis (GN),
acute tubular necrosis (ATN), and acute interstitial nephritis
MDRD STUDY EQUATION (AIN).
It is usually immune-mediated process; due to
Glomerulonephritis
infection or inflammation that would directly
(GN)
damage the glomerulus
Acute Tubular
most common; the usual drugs that causes this
necrosis (ATN),
would be the aminoglycosides, cisplatin,
CLINICAL PHARMACY | MODULE 3 25
supportive care and protecting the kidneys from additional Calcium polystyrene sulfonate (SPS) at 15 to 30 g every 2
insults to 4 times a day
Post Sudden obstruction of urine flow dur to enlarged (CRRT), intermittent hemodialysis (IHD), peritoneal dialysis
renal prostate, kidney stones, bladder tumor, or injury (PD), or kidney transplantation to maintain renal
homeostasis and avoid complications.
ACUTE KIDNEY INJURY End-stage renal disease (ESRD) includes patients treated by
dialysis or transplantation, irrespective of the level of GFR.
Hypovolemia o For CKD patients, treatment purpose is to delay the progression of the
disease and prevent death.
Decreased cardiac output
Decreased Effective circulation volume TREATMENT
o Congestive Heart Failure The purpose of nonpharmacologic and pharmacologic
Prerenal o Liver failure treatment is to slow progression of CKD and to prevent
Impaired autoregulation and/or treat complications of reduced kidney function.
o NSAIDS Ultimately, patients with advanced CKD/kidney failure will
o ACE/ARB require RRT such as a kidney transplant, IHD, or PD.
o Cyclosporine
The mainstay of pharmacologic interventions aim to prevent
Glomerular Ischemia and/or treat complications of impaired renal function such
acute as anemia, MBDs, CV complications, and vitamin
glomerular irregularities.
nephritis o MBD- Mineral Bone disorder
Tubules and Sepsis/Infection
interstitium TREATMENT OF ANEMIA
Vascular Nephrotoxins Erythropoietin is produced in the kidney. For patients with kidney disorder, there is
Intrinsic decreased/ halt in production of blood, hence there is anemia.
Vasculitis Exogenous: Iodinated
contrast, aminoglycoside, Iron, Folate supplements
Malignant o IV iron - given for severe anemia
hypertension cisplatin, amphotericin B o Oral Iron
TTP-HUS Exogenous: Hemolysis, Ferrous Fumarate 65 Elemental Iron
rhabdomyolysis Ferrous Gluconate 35 Elemental Iron
Myeloma, intratubular Ferrous Sulfate 60 Elemental Iron
crystals Elemental Iron - the total amount of iron in the supplement
available for absorption by your body. It is required for the
Bladder outlet obstruction production of hemoglobin, and eventually be incorporated to the
Postrenal Bilateral pelvoureteral obstruction (or unilateral red blood cells
obstruction of a s solitary functioning kidney) Recombinant Human Erythropoietin
o Epoietin alfa and beta
CHRONIC KIDNEY DISEASE (CKD) o SQ route is preferred
a progressive disease staged on the patient's glomerular Novel Erythropoiesis-Stimulating Protein (NESP)
filtration rate (GFR) and presence of albuminuria o Darbepoeitin alfa
ESA therapy has become an integral part of the care for
GLOMERULAR FILRTRATION RATE CATEGORIES ON patients with CKD
KODIGO CLASSIFICATION Management of anemia includes administration of
erythropoiesis-stimulating agents (ESAs) (eg, epoetin alfa,
GFR GFR (mL/min/1.73 epoetin alfa-epbx, darbepoetin alfa, methoxy
Terms
Category m2) polyethylene glycol-epoetin beta) and regular iron
1 > 90 Normal or high supplementation to maintain hemoglobin concentration and
2 60-89 Mildly decreased prevent the need for blood transfusions.
Mildly to moderately Pharmacologic therapy for anemia of CKD includes iron
3a 45-59
decreased supplementation to prevent and correct iron deficiency and
Moderately to severely ESA therapy to correct erythropoietin deficiency.
3b 30-44
decreased Iron supplementation is first-line therapy for anemia of CKD
4 18-29 Severely decreased if iron deficiency is present, and for some patients the target
5 <15 Kidney failure Hb may be achieved without concomitant ESA therapy.
For most individuals with advanced CKD, however,
STAGES OF CHRONIC KIDNEY DISEASE combined therapy with iron and an ESA will be necessary.
STAGES GFR* % KIDNEY FUNCTION TREATMENT OF METABOLIC DISTURBANCES
1 90 or higher 90-100 %
2 89 - 60 89 - 60 % ACIDOSIS
3a 59 - 45 59 - 45 % Oral doses of sodium bicarbonate of 1±6 g/day
3b 44 - 30 44 - 30 % o The main route of excreting hydrogen ion is through the kidney. If
4 29 - 15 29 - 15 % .
5 Less than 15 Less than 15 %
Sometimes, KD is already diagnosed at stages 3b to 5 HYPERPHOSPHATEMIA
Kidney failure is defined as: (1) a GFR less than 15 Restricting dietary phosphate, administration of a
mL/min/1.73 m2 or (2) a need for renal replacement phosphate binder, such as aluminum hydroxide, calcium
therapy (RRT) such as continuous renal replacement therapy acetate, calcium carbonate, sevelamer
CLINICAL PHARMACY | MODULE 3 28
HYPOCALCEMIA
Oral calcium salts, Vitamin D
o Vitamin D is given for the absorption of calcium (e.g., calcitriol)
SUMMARY
The overall goal of therapy in CKD patients is to delay or
prevent progression of the disease while minimizing the
development or severity of associated complications.
Planning for renal replacement therapy (transplantation,
HD or PD) should begin for patients deemed high risk for
progression to ESRD (eg, at stage 4 CKD).
Chronic kidney disease (CKD) is classified based on the
cause of kidney disease, assessment of glomerular filtration
rate, and extent of albuminuria over at least a 3-month
period.
The most common causes of CKD (requiring dialysis or
kidney transplantation), often called end-stage renal
disease (ESRD), are diabetes mellitus and hypertension.
TAKEAWAY POINTS
Anemia of CKD is multifactorial with loss of erythropoietin
synthesis by the kidney, iron deficiency, and chronic
inflammation all implicated.
Angiotensin-converting enzyme inhibitors (ACEIs) and
angiotensin receptor blockers (ARBs) are primary
pharmacologic treatments to delay progression of CKD
because of their effects on renal hemodynamics to reduce
intraglomerular pressure and proteinuria.
o The antiproteinuric effect of ACEIs and ARBs is a class
effect and not specific to any one agent.
o For patients with hypertension, the primary goal is to
achieve the target blood pressure while a secondary
goal is to control proteinuria.
Sodium-glucose co-transporter 2 (SGLT-2) inhibitors are
emerging as potential agents to prevent progression to
later stages of CKD and ESRD and these effects seem to be
independent of glucose lowering.
Metformin is still considered a first-line agent in individuals
with type 2 diabetes and CKD. As previously discussed,
SGLT-2 inhibitors are emerging as a potential new
treatment of DCKD.
o These agents may be used in patients with an eGFR
above 25 to 30 mL/min/1.73 m2.
An be initiated and/or continued in individuals with an
eGFR greater than or equal to 45 mL/min/1.73 m2.
It is not recommended to initiate metformin in patients with
an eGFR between 30 and 44 mL/min/1.73 m2, and the
decision whether to continue therapy in patients who reach
this level of kidney function should be made only after
weighing the risks and benefits.
Management of PTH, phosphorus, and calcium is important
in preventing CKD-MBD and CV and extravascular
calcifications. Patients with CKD-MBD usually require a
combination of dietary intervention, phosphate-binding
medications, vitamin D, and calcimimetic therapy (for ESRD
patients) to achieve these goals.
CLINICAL PHARMACY | MODULE 3 29
END
CLINICAL PHARMACY | MODULE 3 30
ASTHMA
Mucus secretions increase to cause further narrowing of airways and plugging
of some smaller airways
Global Initiative for Asthma (GINA), 2018: Reduced flow of air to the alveoli
heterogeneous disease, usually characterized by chronic Muscle in the bronchi constricts and narrows the airways
airway inflammation. It is defined by the history of respiratory
symptoms such as wheeze, shortness of breath, chest tightness PATHOLOGY OF ASTHMA
and cough that vary over time and in intensity, together with
variable expiratory airflow limitation
The first-line and mainstay treatment is Inhaled Corticosteroids (ICS) for
localized action. Do not give oral as first-line treatment.
PATHOPHYSIOLOGY OF ASTHMA
Air travels into the lungs via the windpipe (trachea). It goes down a series of
TRIGGER FACTOR
branching airways called bronchi. These branch into smaller bronchioles and The main cause of asthma is allergens.
then into millions of tiny air sacs (alveoli). A trigger factor is needed to start a cascade of activities that will lead to
Oxygen in the air passes through the thin walls of the alveoli into the tiny airway inflammation. At first, there is bronchoconstriction; but for chronic
blood vessels nearby. Oxygen attaches to red blood cells and is carried in asthma, there is airway inflammation. The trigger factor is usually allergens.
the blood vessels to the rest of the body.
Trigger factors cause airway inflammation. This airway inflammation will cause
hypersecretion of mucus, airway muscle constriction, and swelling bronchial
membranes causing narrow breathing passages. Eventually, the patient will
experience wheezing, cough, shortness of breath, and tightness in chest.
CLINICAL PHARMACY | MODULE 3 31
Asthma vs COPD
Episodic; airway limitation/inflammation (ICS is mainstay)
Asthma Reversible bronchospasm; shortness of breath
Usually allergic in nature, thus the bronchodilators
Progressive
COPD
Usually irreversible airway limitation/inflammation
o Some asthma drugs are effective for COPD, but not all.
Bronchodilators: used because of bronchospasms.
o Beta-2 agonists, Muscarinic antagonists, and Methylxanthines.
Anti-inflammatory agents: mainstay and first-line of treatment for asthma
o Release of inhibitors (e.g. mast cell stabilizer)
o Steroids (e.g. corticosteroids)
o Slow anti-inflammatory drugs
o Antibodies (e.g. anti-IgE)
Leukotriene antagonists: can be a bronchodilator
o Lipoxygenase inhibitors
o Receptor inhibitors (e.g. Montelukast)
Steroids
1. Antigen will cause an immune response. Exposure to antigen causes synthesis
of IgE. Bronchodilators: Mainstay and first-line for COPD
2. IgE binds to and sensitizes mast cells and other inflammatory cells. Antibiotics
o This is why we have drugs that target the mast cells Anti-inflammatory drugs
3. The sensitized mast cell will cause the release of various mediators that can o Steroid
account for most of the signs of the early bronchoconstrictor response in o Usually given at the end (last-line). It should not be given as mainstay
asthma. and first-line drug for COPD
Mediators released:
o LTC4, D4, leukotrienes C4 and D4
o ECF-A, eosinophil chemotactic factor-A PHARMACOLOGIC OPTIONS FOR TREATMENT OF ASTHMA
o PGD2, prostaglandin D2 Controller
o Histamine tryptase o Reduce airway inflammation, control symptoms, and reduce future risk.
Some drugs also act on these mediators (e.g. Leukotriene antagonists, Reliever medication/ rescue medication
antihistamine). o For episodic asthma and acute bronchospasms.
The early response is usually bronchoconstriction and an increase of o Usually given for breakthrough symptoms
secretion/mucus production. However, LT and IgE also attracts some o Used for short-term bronchoconstriction due to exercise.
inflammatory cells, causing inflammation.
Main manifestation of asthma: inflammation (treated by ICS)
Add-on therapies
CLINICAL PHARMACY | MODULE 3 32
STRATEGIES CORTICOSTEROID
Acute bronchospasm must be treated promptly and relieved
reliever INHALED CORTICOSTEROIDS (ICS)
o Beta-2 agonists, muscarinic antagonists, and Have become common first-line therapy for individuals with
theophylline moderate to severe asthma.
Long-term preventive treatment requires control of the Beclomethasone, budesonide, dexamethasone,
inflammatory process in the airways with controller drugs. flunisolide, fluticasone, mometasone
o The most important anti-inflammatory drugs in the Candidiasis can be controlled with a topical mouthwash
treatment of chronic asthma are the corticosteroids. containing clotrimazole or a similar antifungal agent.
o Long-acting beta-2 agonists (LABAs) can improve the o Oral disorder associated with inhaled corticosteroid
response to corticosteroids. o ICS may accumulate and cause deposition in the pharynx, changing the
Antibodies against IgE and certain interleukins are also oropharyngeal flora, leading to candidiasis. If the patient is using
inhaled corticosteroid, it is important to educate the patient to wash or
used in chronic therapy. gargle every after inhalation to prevent accumulation of steroids
The leukotriene antagonists have effects on both
bronchoconstriction and inflammation but are used only for SYSTEMIC CORTICOSTEROIDS
prophylaxis.
Not for rescue doses ORAL CORTICOSTEROIDS
Nasal oxygen is basic therapy for acute bronchospasm of o Usually prednisone
any cause. o Used chronically only when other therapies are
unsuccessful because of their toxicity (limited use)
BETA-2 SELECTIVE AGONIST INTRAVENOUS CORTICOSTEROIDS
Reverse asthmatic bronchoconstriction (airway limitation and o For status asthmaticus
Status asthmaticus is a respiratory failure; severe/worst asthma attack
inflammation)
o Include prednisolone (active metabolite of prednisone)
SABA and hydrocortisone (cortisol).
Durations of action of 4 hours or less. LEUKOTRIENE ANTAGONIST
Choice for acute episodes of bronchospasm (reliever) Can be used for acute bronchoconstriction (relieve bronchospasm), and
Not effective for prophylaxis. inflammatory effect
Recall: SABA should not be given as first-line and it should be used in Used only for prophylaxis
combination with ICS Montelukast and zafirlukast are antagonists at the LTD4
leukotriene receptor
LABA o Montelukast is given orally
For prophylaxis (can be used as reliever) Drugs are orally active and have been shown to be
12-hour duration of action effective in preventing exercise-, antigen-, and aspirin-
It should also be in combination with ICS
induced bronchospasm.
Not recommended for acute episodes of asthma
Short-acting (SABA) Long-acting (LABA)
Albuterol (salbutamol) Salmeterol (Asthma & COPD) ANTI-IgE ANTIBODIES
Terbutaline Formoterol (Asthma & COPD) Omalizumab is a humanized murine monoclonal antibody
Metaproterenol Indacaterol (COPD only) to human lgE.
Vilanterol (COPD only) Inhibits the binding of IgE but does not activate IgE already
bound to its receptor and thus does not provoke mast cell
METHYLXANTHINES degranulation.
Bronchodilator It prevents activation by asthma trigger antigens and
From plants: Caffeine (in coffee), theophylline (tea), and subsequent release of inflammatory mediators
theobromine (cocoa).
Theophylline is the only member of this group that has been GINA GUIDELINES
important in the treatment of asthma.
ADULT AND ADOLESCENTS 12+ YEARS
The major clinical use of methylxanthines is asthma and
COPD.
MUSCARINIC ANTAGONISTS
Bronchodilator
Ipratropium (SAMA; for asthma)
Tiotropium (LAMA), aclidinium, umeclidinium, and
glycopyrrolate (COPD).
Reverse bronchoconstriction in some asthma patients
(especially children) and in many patients with COPD.
o Beta-2 antagonists more effective and preferred in acute
bronchospasms in asthma
o Antimuscarinic agent more effective in acute bronchospasm in COPD
No effect on the chronic inflammatory aspects of asthma.
Asthma medication options: Adjust treatment up and down for individual patient
needs
CLINICAL PHARMACY | MODULE 3 33
REVIEW ASSESS
Diagnosis
RESPONSE Symptom control &
Symptoms risk factors (including
Exacerbations lung function) Avoidance
Side-effects Inhaler technique &
Patient satisfaction adherence
Lung function Patient preference
Cromolyn,
steroids,
zileuton,
ADJUST TREATMENT antibody
Asthma medications
Non-pharmacological
strategies Beta antagonists,
Treat modifiable risk factors theophylline, Steroids,
muscarinic cromolyn,
antagonists, leukotriene
leukotriene antagonists
antagonists
Other
Preferred Controller Other Controller Preferred
Reliever
Choice Options Reliever
Options
STEP 1 Low dose ICS
As-needed low dose taken whenever
ICS-formoterol* SABA is taken +
STEP 2 Leukotriene As-needed Bronchodilation and other drugs used in asthma
Daily low dose receptor low dose ICS- Exposure to antigen (dust, pollen) means there is antigen and IgE on mast
inhaled antagonists (LTRA) formoterol* cells so this must be avoided.
corticosteroids (ICS), or low dose ICS o Cromolyn is not usually given nowadays.
or as-needed low taken whenever For mediators (leukotrienes, cytokines), we have early response
dose ICS-formoterol* SABA is taken + bronchoconstriction leading to acute symptoms and late response
Medium dose ICS, As-needed inflammation leading to bronchial hyperactivity.
STEP 3 o Leukotriene antagonist is the only drug that can be given for both early
or low dose ICS + short-acting
Low Dose ICS-LABA and late response (both bronchoconstriction and inflammation)
LTRA # beta2 -
High dose ICS, agonist o Early response: we have beta agonists, theophylline, muscarinic
STEP 4 (SABA) antagonists, and leukotriene antagonists
add-on
Medium dose ICS- Beta agonists, theophylline, and muscarinic antagonist for
tiotropium, or As-needed
LABA bronchoconstriction
add-on LTRA # low dose ICS-
STEP 5 o Late response: steroids, cromolyn, and leukotriene antagonists
formoterol Steroids and cromolyn for inflammation
High dose ICS-LABA
Refer for phenotypic Add low dose
assessment ± add-on OCS, but consider STEPPING DOWN ASTHMA TREATMENT
therapy e.g., side-effects
tiotropium, anti-IgE,
Current Current medication and
anti-IL5/5R, anti-IL4R Step dose
Options for stepping down Evidence
* Off-label; data only with budesonide-formoterol (bud-form) D
+ Off-label; separate or combination ICS and SABA inhalers Continue high dose ICS-LABA and
reduce OCS dose
Low-dose ICS-form is the reliever for patient prescribed bud-form or BDP- High dose ICS-LABA plus
Use sputum-guided approach to
B
form maintenance and reliever therapy oral corticosteroids
reducing OCS
# Consider adding HDM SLIT for sensitized patients with allergic rhinitis and Step 5 (OCS) D
Alternate-day OCS treatment D
FEV >70% predicted Replace OCS with high dose ICS
High dose ICS-LABA plus
Refer for expert advice D
ADDITIONAL NOTES other add-on agents
SABA is given as-needed as a reliever but requires ICS as controller. Continue combination ICS-LABA B
with 50% reduction in ICS
The preferred controller, ICS + formoterol can also be given as a reliever Moderate to high dose
component, by using available
because of its immediate release. ICS-LABA maintenance
formulations
The mainstay treatment is ICS. treatment
Discontinuing LABA may lead to A
Leukotriene can t be given as reliever because beta agonist is the preferred Step 4
deterioration
drug for bronchoconstriction. It can be given as other controller option Medium dose ICS- Reduce maintenance ICS-
o Formoterol can be given since we prefer beta agonist as reliever for Formoterol* as Formoterol* to low dose, and D
asthma. For COPD, antimuscarinic drugs are preferred for maintenance and continue as-needed low dose ICS-
reliever Formoterol* reliever
bronchospasm (e.g., ipratropium, tiotropium)
High dose ICS plus Reduce ICS dose by 50% and
(Step 4) Tiotropium is a long acting antimuscarinic agent second controller continue second controller
B
Antimuscarinic agent, SAMA is not recommended as reliever Reduce ICS-LABA to once daily D
(Step 5) You can give anti-IgE (Omalizumab), antihistamine, or add low dose Low dose ICS-LABA
Discontinuing LABA may lead to A
maintenance
oral corticosteroid but not recommended because of side effects deterioration
o IV corticosteroid like prednisolone can be given for status asthmaticus Step 3 Medium dose ICS- Reduce maintenance ICS-
(worst form of acute asthma attack) Formoterol* as Formoterol* to once daily and C
o Oral corticosteroids can be given as other controller options maintenance and continue as-needed low dose ICS-
reliever Formoterol* reliever
THINGS TO REMEMBER FOR THE BOARDS
CLINICAL PHARMACY | MODULE 3 34
Moderate- or high-dose
Reduce ICS dose by 50% A EMPHYSEMA
ICS
Once-daily dosing (Budesonide, A Enlargement of the airway
Ciclesonide, Mometasone) Characterized by permanent destruction of the alveoli as a
Switch to as-needed low dose ICS- result of irreversible destruction of the elastin that maintains
A
Formoterol
Low dose ICS Adding LTRA may allow ICS dose the strength of the alveolar walls
B
to be stepped down
Insufficient evidence to support -
Step 2 step-down to as-needed ICS with
SABA
Switch to as-needed low dose ICS- A
Formoterol
Complete cessation of ICS in adults A
Low dose ICS or LTRA and adolescents is not advised as
the risk of exacerbations is
increased with SABA-only
treatment
BDP: Beclomethasone dipropionate
*ICS-Formoterol maintenance and reliever treatment can be prescribed with low
dose budesonide-formoterol or BDP-formoterol
Step 1 is just symptomatic; the management is usually as-needed doses
Each step is usually done for 4 weeks or one month
Please tick in the box that applies to you. One box only. Grades 0 Grade FEV1 (% predicted)
4. Assessment of GOLD 1
mMRC I only get breathless with strenuous airflow GOLD 2 50-79
limitation GOLD 3 30-49
Grade 0 exercise. GOLD 4 < 30
mMRC I get short of breath when hurrying on the
Grade 1 level or walking up a slight hill
I walk slower than people of the same age Moderate or Severe
mMRC on the level because of breathlessness, or I Exacerbation
History
Grade 2 have to stop for breath when walking on Assessment of
my own pace on the level symptoms/risk hospital admission C D
mMRC I stop for breath after walking about 100 of 0 to 1 (not leading to
A B
Grade 3 meters or after a few minutes on the level exacerbations hospital admission)
mMRC 0-1
mMRC I am too breathless to leave the house or I CAT<10
Grade 4 am breathless when dressing or undressing Symptoms
We ask our patient regarding their tolerance to exercise. The patient would now
answer and place a check on what type of tolerance they could take. We are going ABCD is the basis of treatment
to use this for the management of our patient.
TREATMENT GOALS
CAT ASSESSMENT
The CAT assessment or COPD Assessment Test. The patient would answer this by Prevent disease
placing an x mark on how they would describe their situation and there is going to Relieve symptoms and improve exercise tolerance
be a computation of the total scores. The total score would be used for the
determination of the management. Improve health status
Prevent & treat exacerbations
For each item below, place a mark (x) in the box that best describes you Prevent & treat complications
currently. Be sure to only select one response for each question.
EXAMPLE:
Reduce mortality
I am very sad SCORE
I am very happy
I never cough I cough all the time STRATEGIES FOR COPD TREATMENT
I have no phlegm My chest is COPD responds to bronchodilators, including beta-2
(mucus) in my completely full of
chest at all phlegm (mucus)
agonists and muscarinic antagonists, but is less responsive
My chest does to corticosteroids than asthma.
My chest feels Asthma Combination of ICS + bronchodilator; structural damage
not feel tight at
very tight Combination of bronchodilators (beta-2 agonist,
all
When I walk up When I walk up a COPD antimuscarinic agents, and methylxanthine)
a hill or one hill or one flight of Corticosteroids may be given, but usually in the end
flight of stairs, I stairs, I am very A combination long-acting beta-2 agonist with a long-
am not breathless breathless acting muscarinic blocker, given by inhalation, is often
I am not limited I am very limited
doing any doing activities at preferred for prophylaxis
activities at home home Antibiotic therapy is more important in COPD than in
I am not at all asthma.
I am confident
confident leaving
leaving my home
my home because
despite my lung DIFFERENTIAL DIAGNOSIS OF COPD
of my lung
condition
condition
I don't sleep DIAGNOSIS SUGGESTIVE FEATURES
soundly because
I sleep soundly
of my lung
Onset in mid-life
condition COPD Symptoms slowly progressive (irreversible)
I have lots of I have no energy History of tobacco smoking or exposure
energy at all to other types of smoke
TOTAL SCORE: Onset early in life (often childhood)
Symptoms vary widely from day to day
THE REFINED ABCD ASSESSMENT TOOL Symptoms worse at night/early morning
Combination of CAT and mMRC Asthma Allergy, rhinitis, and/or eczema also
Determines the type of management best for our patient. present
o Serves as the basis of COPD treatment. The patient must be classified Family history of asthma
first (e.g., what grade and what group), then the right treatment is
determined. Obesity coexistence
Chest X-ray shows dilated heart,
Congestive Heart pulmonary edema
Spirometrically confirmed Assessment of Assessment of Failure Pulmonary function tests indicate volume
diagnosis airflow limitation symptoms/risk of restriction, not airflow limitation
exacerbations
Large volumes of purulent sputum
Bronchiectasis Commonly associated with bacterial
infection
CLINICAL PHARMACY | MODULE 3 36
Chest X-ray/CT shows bronchial dilation, counseling, delivered by healthcare professionals improve
bronchial wall thickening quit rates.
Onset all ages The effectiveness and safety of e-cigarettes as smoking
Tuberculosis
Chest X-ray shows lung infiltrate cessation aid is uncertain at present.
Microbiological confirmation Pharmacological therapy can reduce COPD symptoms,
High local prevalence of tuberculosis reduce the frequency and severity of exacerbations, and
Onset at younger age, nonsmokers improve health status and exercise tolerance.
May have history of rheumatoid arthritis Each pharmacological treatment regimen should be
Obliterative or acute fume exposure individualized and guided by the severity of symptoms,
Bronchiolitis Seen after lung or bone marrow risk of exacerbations, side effects, comorbidities, drug
transplantation
CT on expiration shows hypodense areas preference, and ability to use various drug delivery devices
Predominantly seen in patients of Asian Inhaler technique needs to be assessed regularly
descent
Usually when the patient is categorized under Group A based on GOLD
Most patients are male and nonsmokers treatment guidelines, influenza vaccine and pneumococcal vaccines are given
Diffuse Almost all have chronic sinusitis Influenza vaccination decreases the incidence of lower
panbronchiolitis Chest X-ray & HRCT show diffuse small respiratory tract infections
centrilobular nodular opacities &
hyperinflation Pneumococcal vaccination decreases lower respiratory
o We can see in the chest x-ray, there are tract infections
portions that have black spots Pulmonary rehabilitation improves symptoms, quality of life,
These features tend to be characteristic of the respective diseases, and physical and emotional participation in everyday
but ore not mandatory. For example, a person who has never activities
smoked may develop COPD (especially in the developing world
where other risk factors may be more important than cigarette In patients with severe resting chronic hypoxemia, long-term
smoking); asthma may develop in adult and even in elderly patients oxygen therapy improves survival
In patients with stable COPD and resting or exercise-
EXAMPLE CASE induced moderate desaturation, long-term oxygen
treatment should not be prescribed routinely. However,
THE REFINED ABCD ASSESSMENT TOOL individual patient factors must be considered when
Consider two patients both patients with FEV1 < 30% of evaluating need for supplemental oxygen.
18 and one with no exacerbations In patients with severe chronic hypercapnia and a history of
in the past year and the other with three moderate hospitalization for acute respiratory failure, long-term non-
exacerbations in the past year. Both would have been labelled invasive ventilation may decrease mortality and prevent re-
GOLD D in the prior classification scheme. However, with the new hospitalization.
proposed scheme, the subject with three moderate In select patients with advanced emphysema refractory to
exacerbations in the past year would be labelled GOLD grade optimized medical care, surgical or bronchoscopic
4, group D. interventional treatments may be beneficial.
Spirometrically Palliative approaches are effective in controlling symptoms
confirmed Post-bronchodilator FEV1 < 0.7 in advanced COPD.
diagnosis
VACCINATION FOR STABLE COPD
Grade FEV1 (% predicted)
Assessment of GOLD 1 Influenza vaccination reduces serious illness and death in
airflow GOLD 2 50-79 COPD patients
limitation GOLD 3 30-49 The 23-valent pneumococcal polysaccharide vaccine
GOLD 4 < 30 (PPSV23) has been shown to reduce the incidence of
community-acquired pneumonia in COPD patients aged
Moderate or Severe <65 with an FEV1 < 40% predicted and in those with
Exacerbation comorbidities
History
Assessment of In the general population of adults 65 years, the 13-
hospital admission C D valent conjugated pneumococcal vaccine (PCV13) has
symptoms/risk 0 to 1 (not leading
of demonstrated significant efficacy in reducing bacteremia
to hospital A B and serious invasive pneumococcal disease
exacerbations admission)
mMRC 0-1
CAT<10
Symptoms
No exacerbation: Group 4B
Three exacerbations: Group 4D
COMMONLY USED MAINTENANCE MEDICATIONS IN COPD Blood eosinophils >300 100-300 History of
cells/ L cells/ L mycobacterial
History of, or infection
concomitant, asthma
#despite appropriate long-acting bronchodilator maintenance therapy
*note that blood eosinophils should be seen as a continuum; quoted values
represent eosinophil counts are likely to fluctuate.
Note: Asthma is more on eosinophilic, while COPD is more on neutrophilic
ICS are contraindicated for patients with bacterial infections because ICS
medications are immunosuppressants
D
moderate LAMA or
exacerbations or C LAMA + LABA* or
LAMA ICS + LABA**
hospital admission *Consider if highly symptomatic (e.g.,
CAT>20)
**Consider if eos
0 to 1 moderate
A B
exacerbations (not
A A long-acting bronchodilator
leading to hospital
bronchodilator (LABA or LAMA)
admission)
mMRC 0-1,
,
CAT<10
The most common among anticholinergics are the Ipratropium bromide Eos: blood eosinophil count in cells per microliter; mMRC: modified Medical
(SABA) and Tiotropium (LABA). Research Council dyspnea questionnaire; CATTM: COPD Assessment Test
Take note that methylxanthines, phosphodiesterase-4 inhibitors, and mucolytic In Group A, the treatment is a Bronchodilator. This may be SAMA or SABA.
agents (carbocisteine, n-acetylcysteine) are included in treatments for COPD. The mainstay treatment is bronchodilator, others are add-ons
PHARMACOLOGICAL THERAPY FOR STABLE COPD KEY POINTS FOR THE USE OF ANTI-INFLAMMATORY
AGENTS
BRONCHODILATORS
Long-term monotherapy with ICS is not recommended
Bronchodilators are medications that increase FEV 1 and/or
change other spirometric variables Long-term treatment with ICS may be considered in
association with LABAs for patients with a history of
Bronchodilator medications in COPD are most often given
exacerbations despite appropriate treatment with long-
on a regular basis to prevent or reduce symptoms
acting bronchodilators
Toxicity is also dose-related
Long-term therapy with oral corticosteroids is not
Use of short acting bronchodilators on a regular basis is not recommended
generally recommended
In patients with severe to very severe airflow limitation,
chronic bronchitis, and exacerbations the addition of a PDE4
COMBINATION BRONCHODILATOR THERAPY
inhibitor to a treatment with long-acting bronchodilators
Combining bronchodilators with different mechanisms and with/without ICS can be considered
durations of action may increase the degree of
Preferentially, but not only in former smokers with
bronchodilation with a lower risk of side effects compared
exacerbations despite appropriate therapy, macrolides, in
to increasing the dose of a single bronchodilator
particular azithromycin, can be considered
INHALED CORTICOSTEROIDS Statin therapy is not recommended for prevention of
Factors to consider when initiating ICS treatment in combination exacerbations
with one or two long-acting bronchodilators (note the scenario is Antioxidant mucolytics are recommended only in selected
different when considering ICS withdrawal): patients
STRONG SUPPORT CONSIDER USE AGAINST USE
History of 1 moderate Repeated
hospitalization(s) for exacerbation pneumonia
exacerbations of of COPD per events
COPD# year# Blood
Blood eosinophils
exacerbations of COPD eosinophils <100 cells/µL
per year#
CLINICAL PHARMACY | MODULE 3 38
PPI
GASTROESOPHAGEAL REFLUX DISEASE
PPIs irreversibly inhibit the final step in gastric acid
secretion. Gastroesophageal Reflux Disease (GERD) is a disorder
caused by abnormal reflux of gastric contents into the
These agents inhibit greater than 90% of gastric acid
esophagus and often results from a defect in lower
secreted in 24 hours
o This is usually given once a day because the effect is usually 24 hours. esophageal sphincter (LES) function.
there is still decreased gastric acid, The classic symptom of GERD is heartburn described as
thus dose pacing is not effective. substernal warmth or burning starting in the epigastric area
PPIs are effective for healing duodenal and gastric ulcers. radiating to the neck
o First line for peptic ulcer
Excessive reflux of acid and pepsin resulting in mucosal
They have a protective effect against NSAID-related damage and inflammation is termed reflux esophagitis
mucosal injury.
THERAPEUTIC LIFESTYLE CHANGES RECOMMENDED FOR
TREATMENT FOR HELICOBACTER PYLORI-INDUCED ULCERS TREATMENT OF GASTROESOPHAGEAL REFLUX DISEASE
Selection of first-line treatment for confirmed H. pylori Dietary and
Aggravating factors Lifestyle changes
should incorporate prior antibiotic exposure, particularly medication
exposure to macrolides, and history of allergy. Directly irritating Directly irritating General
foods medications Smoking cessation
REGIMEN DURATION DRUG #1 DRUG #2 DRUG #3 DRUG #4
Amoxicillin 1g
Citrus fruits Bisphosphonates Weight reduction
Proon pump
PPI once or
Clarithromycin twice daily or Carbonated Aspirin/ NSAIDs if overweight
inhibitor-based 14 d 500 mg twice Metronidazole -
triple therapy
twice daily
daily 500 mg twice beverages Iron Reduction of
daily Onions alcohol
Bismuth Potassium
Bismuth
quadruple 10-14 d
PPI or H2RA
once or twice
subsalicylate
Metronidazole
250-500 mg
Tetracycline
500 mg four Spicy food consumption
525 mg four
therapy daily
times daily
four times daily times daily Tomatoes Avoid
Non-bismuth PPI once or Clarithromycin
Amoxicillin 1g
Metronidazole aggravating
quadruple or twice daily 250-500 mg 250-500 mg
10-14 d
on days 1- twice daily on
twice daily on
twice daily on
factors
days 1-10
therapy 10 days 1-10 days 1-10 Foods which lower Medications which Night time
PPI once or Metronidazole Clarithromycin
Sequential twice daily
Amoxicillin 1g
250-500 mg 250-500 mg
esophageal lower esophageal symptoms
10 d twice daily on
therapy on days 1-
days 1-5
twice daily on twice daily on sphincter tone: sphincter tone Avoid eating
10 days 6-10 days 6-10
PPI once or Metronidazole Clarithromycin Caffeinated Anticholinergics within three hours
Amoxicillin 1g
Hybrid therapy 14 d
twice daily
twice daily on
250-500 mg 250-500 mg beverages Estrogen/ before bedtime
on days 1- twice daily on twice daily on
14
days 1-14
days 7-14 days 7-14 Chocolate progesterone Elevate head of
Levofloxacin 10-14 PPI twice Levofloxacin Amoxicillin 1g
- Fried or fatty Nicotine bed
triple days daily 500 mg OD twice daily
Levofloxacin
PPI twice Amoxicillin 1g
Levofloxacin Metronidazole
foods Nitrates Postprandial
sequential
10 days daily on
days 1-10
twice daily on
days 1-5
500 mg once 500 mg twice Mint Tetracycline symptoms
CLINICAL PHARMACY | MODULE 3 40
END
HOSPITAL PHARMACY | MODULE 3 1
o It is required that the medical/hospital director is a physician. Cater to the pediatric population
If you are the owner of the hospital and a physician at the same Medical Center (PCMC)
time, you can be both the CEO and the Hospital director.
If you are the CEO but is not a physician, you cannot be the REHABILITATION & CHRONIC DISEASES
medical/hospital director. There is no requirement for the CEO,
only for the medical/hospital director. For handicapped/disabled patients
Organized medical staff of hospital
HOSPITAL PHARMACY | MODULE 3 2
HOSPITAL PHARMACY
FUNCTIONS OF HOSPITAL Also known as Institutional Pharmacy
Mnemonic: P.E.R.P.
The first ever recognized practice of pharmacy
Patient care Started by Jonathan Roberts the 1st hospital pharmacist; worked at
Education or training Pennsylvania hospital
Research or innovation o John Morgan 2nd hospital pharmacist
Public health/community health Practice of pharmacy in a hospital setting
Department/ division of the hospital where in the
MEDICAL STAFF procurement, storage, compounding, dispensing, and
distribution of medications are performed by legally
Medical staffs are different from hospital staff. qualified, professionally competent pharmacists and
o Medical staffs are medical doctors. their assistants
Responsible for the appropriate use of medications:
MAIN TYPES OF MEDICAL STAFF o Rational selection
o Dosing
CLOSED STAFF o Monitoring
The services in the hospital will be provided exclusively by the doctors who
are affiliated with the hospital. MINIMUM STANDARD OF HOSPITAL PHARMACY (ASHP)
Doctors who are not affiliated with the hospital could not use the services of Set by ASPH
o ASHP American Society of Health System Pharmacists
admit to a patient in that hospital. o PSHP Philippine Society of Hospital Pharmacists
Exclusive First President of PSHP: Rosario Capistrano Tan
Affiliated with the hospital Leadership and Practice Management
All services of the hospital are provided and controlled by Drug information and education
the attending or active medical staff Optimizing Medication Therapy
Medication Distribution and Control
OPEN STAFF Facilities, Equipment, and Info Resources
Certain physicians, other than those attending or active Research
medical staff, are allowed to use the private facilities
o Hospitals allow other physicians who are not affiliated with the hospital PHARMACEUTICAL CARE
to practice or use their facilities as needed.
Responsible provision of drug therapy and other patient
Courtesy medical staffs services for the purpose of achieving outcomes that improve
o Are the doctors who are not affiliated with the hospital but are allowed
patient quality of life
Mo
o Example, if the hospital needs a neurosurgeon but none is available, a HOSPITAL PHARMACIST
neurosurgeon from another hospital shall be allowed to perform
surgery even if that surgeon is not affiliated.
Usually observed in tertiary or secondary hospitals
GENERAL RESPONSIBILITIES
Policies and procedures
o In accordance with the Guidelines set by the PSHP (Philippine Society
SPECIAL TYPES OF MEDICAL STAFF of Hospital Pharmacists) and ASHP (American Society of Health System
Honorary retired; outstanding contribution Pharmacists)
o E.g. Dr. Fe del Mundo, president of the Philippine Medical Association Competence
She invented the incubator o Must be a registered pharmacist
Consulting specialists; passed a specialty board; Training and Education
consultant
Documentation
Active/ Attending most active; involved in regular patient
Interdisciplinary interaction
care (the one who admits the patient)
Associate junior members of the active staff; Junior
consultants
HOSPITAL PHARMACY | MODULE 3 4
Nasal Airway Oral Airway o If the patient has Ventricular Tachycardia and is stable with a normal
Nasal airway are used for patients The Oral Airway has an adjunct, it BP, the patient is considered asymptomatic and is given with
who are conscious or have a gag is used for unconscious patients and Amiodarone.
reflex. These are airway adjuncts patients with no gag reflex. o If the patient has Ventricular Tachycardia and patient is pulseless, we
that would facilitate better Because, if the patient is conscious need to defibrillate the patient. One pad goes to the sternum, the other
and has a gag reflex, it will not go to the apex of the heart.
oxygenation of the lungs.
through Ventricular Tachycardia
Laryngoscope
could immediately lead to asystole and the patient will flat line. When
the patient flatlines, this no longer shockable so what you need to do is
to conduct CPR.
B. EDUCATIONAL/TRAINING DIVISION
Coordinate activities of undergraduate/graduate
programs of pharmacy students (internships)
o The pharmacist under the educational or training division are the ones
who would oriented us on the things we are supposed to. So, the clue
here is that if there is a student included, studies and research then the
Placed on top of the emergency cart. answer is education/training division.
Cardiac monitor is for monitoring the heart rhythm or the patient to know if Hospital-wide education program
the patient still has a normal sinus rhythm or if there is an arrhythmia. o For example, they publish a newsletter, announcement, etc.
o The arrhythmia of some patients could be fatal and if it is fatal, the Train newly employed personnel
solution is to defibrillate it. o For, new employees, it is the education/training division that orients,
The defibrillator shocks the heart to go back to its normal rhythm. However, rain, assess, and explain the needed information in the hospital
Not all arrhythmias are shockable. There are only have 2 shockable rhythms: pharmacy
Pulseless Ventricular Tachycardia (VTAC) and Ventricular Fibrillation.
Ventricular Tachycardia C. PHARMACEUTICAL RESEARCH DIVISION
Normal Sinus Rhythm
(VTAC)
Develop new formulations
o Similar to thesis making. There is continuous drug design and
development. It's either they do a new formulation or improve existing
formulation.
o For example, the physician requested a drug D5.3 NaCl and the one
available in the pharmacy is 0.9 NaCl and 0.45 NaCl so it is possible
to create another way to get what the physician needs.
Improve existing formulations
Cooperate in research
HOSPITAL PHARMACY | MODULE 3 6
Sadly, since it lacks pharmacist in the Philippines, in some hospital pharmacy G. DEPARTMENTAL SERVICES
usually there is no one in this division. Although it is a requirement, this division When we talk about departmental services, it is about dispensing
is put aside since it is more important to have a pharmacist for dispensing, IV (Departmental, Dispensing).
admixture, procurement, etc
Usually, level 3 or 4 hospitals have this in the categorization of DOH
Control and dispense IV fluids
hospital, level 4 hospitals have training and education institution that is why Control and dispense regulated drugs
it is connected to research. Coordinate and control all drug delivery and distribution
system
D. IN-PATIENT SERVICES DIVISION
In-patient or admitted/confined in wards)
H. PURCHASING AND INVENTORY CONTROL DIVISION
Provide medication for in-patients Another word for purchasing is procurement.
o Usually in the board exams they change the word in-patient to admitted o Procurement refers to getting drugs and stocks from a supplier or
or confined patient or sometimes non-ambulatory patient. distributor.
Inspection and control Maintain drug inventory
o Inspection and control of the dispensed drugs that are to be delivered o This means auditing, counting the stock, how many were delivered, how
in the ward many were dispensed, how many remain, and how many broke/expired.
Cooperative with research division Purchase of drugs
o The research division develop new formulation for those who will use it, o The drugs they purchase will depend on the decision of the Pharmacy
which are those patients in the ward. and Therapeutics Committee (PTC) and the drugs that are written in the
o For example, they saw that there's is a specific formulation that should Hospital Formulary.
be used for this certain patient then that will be the root of their
research. Receive, store, and distribute drugs
o It is said in the invention that necessity is the mother of invention. We Interview MedReps
can say that necessity is the mother of research. We do research o MedReps are those who bring possible drugs that the hospital could
because we know something is needed. That is why, that is the reason it carry out in the hospital pharmacy. They are the advertisers of drugs.
is cooperative with research division. o The hospital pharmacist that is in the purchasing division is the one who
will interview the MedReps
Board exam question: o E.g., Is the bioavailability good compared to Drug X? Is the price good
compared to Drug X? If it is too expensive, patients might not buy it
Which of the following division of the hospital pharmacy is and the pharmacy will be at a loss, Is the dosage form okay? Does it
in cooperation with the research division? have any adverse effects?, etc.
in-patient services division.
TURNOVER RATE
E. OUT-PATIENT SERVICES DIVISION
Out-patient or ambulatory patient
o Patients that are not confined to the hospital. Remember that in the TR =
hospital pharmacy we cater both in-patient and out-patient
Compound out-patient Rx Annual purchase refers to the number of stocks that the hospital pharmacy
Dispense out-patient Rx purchased/procured.
Annual inventory refers to the amount on the shelves or the amount left.
Inspection/control of drugs
Maintain Rx records Example:
o Rx records stored in 2 years. 5,000 units were procured and during inventory checking (which can be
Similar activity in the community pharmacy. They cater to out-patients or quarterly, monthly, or semi-annually, depending on the hospital), the amount
-patient services division. left on the shelves was 2,000 units. What is the turnover
o When you see same function in the community pharmacy = Out-patient
services division TR = = =2.5
Provide drug consultation/patient counseling
o The drug information is provided to the out-patient.
LOW TURNOVER CAUSES:
F. DRUG INFORMATION SERVICES Duplication of stocks
Provide drug info on drugs and drug therapy to hospital o If there is a duplication of stocks, one set may have a low turnover rate
staff because the other set similar to it is the one being dispensed.
o Counseling regarding drug information to the hospital staff like the Large purchase of slow-moving items
nurses, doctors, dietitians, med techs, etc. o
Maintain drug information center Ex. 5,000/4,000 = 1.25
o In big hospitals, they create a website or an application for this so that o From the 5,000 items procured, 4,000 were left. Since many were left
drug information can be one click away. on the shelves, only a few were dispensed, hence the low turnover rate.
o Not all doctors are pharmacists, so not all are well-equipped with
knowledge regarding the drugs. HIGH TURNOVER INDICATES:
Hospital pharmacy newsletter
o When we say newsletter, we refer to updates (e.g., new dosage form,
new route of admin, phased-out drug) Small volume purchasing
This could mean that the medication is either in demand or low in number
Maintain literature files (which made it appear to be in
Board exam question: Ex. 5,000/1,000 = 5
Which division of the hospital pharmacy is responsible for o From the 5,000 items procured, 1,000 were left. This means that many
provision of drug information to the hospital staff? were sold, hence the high turnover rate.
Drug Information Services
NOTE: SATISFACTORY TURNOVER RATE = 4 TIMES A YEAR
This means that you are changing your stocks 4 times a year or every 3
months.
HOSPITAL PHARMACY | MODULE 3 7
BLOOD PRODUCTS
turnover rate (slow moving goods). If it is has been there for 1 year, then it Contain components from the blood
has a very low turnover rate. You have to think about changing those Not stored in the hospital pharmacy; they are stored in the blood bank
stocks/products so that you can sell them the next time.
If your stocks are already gone in a week, they have a very high turnover EXAMPLES
rate. You have to check whether the product is really in demand or there are
just a few stocks procured.
1. FRESH WHOLE BLOOD (FWB)
I. CENTRAL SUPPLY SERVICES DIVISION Blood withdrawn from a donor that contains all components intact (e.g., RBC,
Coordinate the distribution of medical supplies and WBC, and platelets); used in transfusions to restore blood volume especially
irrigating fluids after traumatic blood loss and maintain adequate blood pressure
o Medical supplies syringes, needles/IV cannula, catheters, urine bag, o Trauma: any condition wherein there is breaching of the integrity of
basin (also called -like shape), human tissue e.g., when stabbed, hit by a car, struck by a boulder
micropore tape, band-aid, povidone iodine, tegaderm, etc. The patient can proceed into hypovolemic shock in case fresh
whole blood is not transfused immediately due to the decrease in
circulating blood volume. Due to massive blood loss and
J. ASSAY / QC DIVISION decreased BP, the patient may die.
Perform analyses
Develop and revise assay procedure 2. PACKED RED BLOOD CELLS (PRBC)
Assist in research contains pure RBC; usually transfused in cases of anemia
o Anemia: satisfy any of the following (1) low RBC count, (2) low
This division monitors the steps in dispensing and the conditions of tools and hematocrit/HCT count, or (3) low hemoglobin/Hgb count
equipment (e.g., if a weighing scale is still accurate, if the mortar and pestle
Not all anemic patients should be transfused with pRBC; there is a threshold
are still in good condition).
that should be met first:
Threshold for blood
K. MANUFACTURING AND PACKAGING DIVISION Normal Range
transfusion
Manufacture commonly used items in the hospital RBC 4-6 million -
o But this is not usually done by this division anymore HCT 35-45% or
The %RBC that is relatively -
Drug packaging present in the plasma 0.35-0.45
o Nowadays, this is the focus of this division 12-16 g/Dl or
Hgb
Drug development 120-160 g/L
o Hgb value and what to do:
Unit dose program Hgb value Action
o Refers to a pack of medications that are to be used by an individual
Below 7 g/dL Blood transfusion is necessary
patient for 24 hours
No need for transfusion. Treat the anemia by
Between 7 to 12
giving oral iron (ferrous sulfate/ gluconate/
L. STERILE PRODUCTS DIVISION g/dL
fumarate) or IV iron (iron sucrose/dextran)
Produce small volume parenterals Above 7 g/dL
Blood transfusion is necessary
but symptomatic
Manufacture sterile ophthalmic, irrigating solutions (IV fluids),
o 1 unit of pRBC (1 blood bag) will increase the following by:
etc. HCT 3%
o We do not want to inoculate bacteria in the human body so the products Hgb 1 g/dL or 10 g/L
must be sterile. If there is bacteria in the blood or in any body organ,
the patient might have infection, bacteremia, sepsis, septic shock, or The units of pRBC to be transfused will depend on the goal/how much you
death. want to increase the Hgb count of the patient (e.g., if 6 g/dL, transfuse 3
bag to achieve 9 g/dL)
Perform aseptic dilution
o Very clean since there are no bacteria
3. FRESH FROZEN PLASMA (FFP)
contains plasma contents such as clotting factors; usually transfused in
CLASSIFICATION bleeding patients with clotting factor deficiency/coagulation disorder like in:
o Hemophilia
IV Fluids o
o Patients with liver disorder/ cirrhosis/ cancer, chronic hepatitis B, or
patients presenting with liver failure
Blood and Non-Blood IV NOTE: Liver is the organ that produces clotting factors
Products Fluids
4. PLATELET CONCENTRATE
Crystalloids Colloids contains pure platelets; transfused in patients with thrombocytopenia (low
platelet count) such as in Dengue and Chikungunya
o d go along its course as
Glucose- Proteinous Non-proteinous a viral infection. Once you recover, the platelet count would normalize
containing o For severe dengue patients that present with massive bleeding, transfuse
platelet concentrate
Electrolyte Albumin Gelatins Starch Dextrans
solutions (20% and 5%)
BOARD EXAM QUESTIONS
HES A hemophilic patient is presented with bleeding and bruising, what blood product is
Mixed Haemoccel (Hydroxyethyl best suitable for him/her?
starch) Answer: Fresh frozen plasma (FFP)
A dengue patient is presented with mucosal bleeding and hematemesis (vomiting
blood), what blood product is best suitable for him/her?
Gelotusin Pentastarch
Answer: Platelet concentrate
In cases of trauma, there is massive blood loss, what is the blood product transfused?
Answer: Fresh whole blood
Tetrastarch What is the threshold to transfuse pRBC and what are the effects of transfusing 1
unit of pRBC?
IV Fluids: fluids to be administered intravenously Answer: 7 g/dL or 70 g/L; increase by 1 g/dL Hgb and 3% HCT
HOSPITAL PHARMACY | MODULE 3 8
WHAT TO WATCH OUT FOR DURING BLOOD o Used for fluid resuscitation, especially for patients with electrolyte loss
TRANSFUSION (e.g., vomiting, diarrhea)
o Not for patients with hyperkalemia
Blood transfusion reaction Dextrose-containing crystalloids
o REMEMBER: When transfusing blood to patients, make sure that the o 5% dextrose in 0.9% sodium chloride (D5NSS)
properly typed and cross-matched to avoid Contains 5% dextrose in NSS
blood transfusion reaction
Used in patients placed in NPO (non per orem, nothing by mouth)
o Could lead to hemolysis (bursting/agglutination of RBC) and causing or no food intake to avoid the risk of hypoglycemia. If the patient
vessel blockage
can eat by himself, use only PNSS.
Fluid overload o 5% dextrose in water
o May cause increase blood pressure, pulmonary edema (fluid in lungs),
or peripheral edema (fluid in periphery)
Hypotonic solution
o
once; it takes time. However, it still depends on the concentration, e.g., IV CANNULA
for hypovolemic hypotensive patients, blood needs to be transfused An example of a medical supply
within 30 minutes. Inserted to provide IV access and infuse the IV fluids
NON-BLOOD IV FLUIDS
Colloids and crystalloids
Used to resuscitate blood volume when blood products are not yet available;
e.g., when there is a car crash patient in the street and is suffering from blood
loss, blood products may not be available readily.
COLLOIDS The diameter of the needle hole is measured through gauges. The higher
Longer half-life than crystalloids the gauge, the smaller is the lumen of the needle.
o 24 gauge: for babies or infants
o 22 gauge: for children
NON- o 20 gauge: for teenagers or for adults with small veins
PROTEINOUS
PROTEINOUS o 18 gauge: for adults
Albumin a colloid IV fluid used in cases of Starch o 16 gauge: large-bore needle
hypoalbuminemia HES Not for long-term use (e.g., 5 days, 1 week) since it is prone to contamination
Important in preserving blood volume inside the (hydroxyethyl o A nidus (breeding place) for infection
blood vessel starch) o Re-inserted after 72 hours to prevent infection and inflammation of the
One of the most abundant plasma protein; Pentastarch vein (phlebitis)
produced by the liver Tetrastarch
Hypoalbuminemia: fluid that is supposed to be Dextrans HOW TO INSERT AN IV CANNULA
inside the blood vessel leaks out to tissues
causing edema. Causes of hypoalbuminemia 1. Place a torniquet so that the vein will bulge.
include liver failure secondary to 2. Insert the IV cannula at 15 to 30 degrees directly at the
o Hepatotoxic drugs/agents vein
o Liver cancer 3. Connect the cannula to the IV tubing, which is already
o Viral infection, e.g., hepatitis
Gelatins connected to the IV fluid bag
Haemocoel
Gelofusin WHAT TO WATCH OUT FOR DURING CANNULATION
Pain
CRYSTALLOIDS o If the IV cannula is correctly inserted, it should be painless. Otherwise,
Shorter half-life than colloids there is something wrong with your insertion.
More common than colloids; examples are plain normal saline solution Bulging
o
The use will depend on the type of crystalloid, e.g., for maintenance solution, the skin.
fluid replacement, electrolyte replenishment
Types: M. RADIOPHARMACEUTICAL DIVISION
o Glucose-containing
o Electrolyte solutions
Procurement, storage, and dispensing of radioisotopes or
o Mixed radiopharmaceuticals
o Non blood IV Fluids Involves proper handling and control
symptoms of hyperthyroidism)
A normal thyroid gland (A) is butterfly-shaped organ located at anterior
neck. It only has little black parts and some areas appear whitish.
If th
up all the iodine leading to black coloration.
In toxic multinodular goiter (C), the iodine is taken up by a lot of nodes.
In case of toxic adenoma (D), the black color (iodine) is concentrated in the
hyperactive area/lump only.
Alpha particle Stopped by paper or outer turnout gear
SESTAMIBI SCAN Stopped by aluminum foil
99mTc-MIBI
Beta particle May penetrate partially into gear
May penetrate exposed skin
For myocardial perfusion scan
Stopped by a few inches of lead
Check if the myocardium is receiving enough perfusion or enough blood. Gamma ray
Will penetrate gear and body
Neutron particle Stopped by concrete
HOSPITAL PHARMACY | MODULE 3 10
Face shield, respirator mask, and PPE with suitable materials are worn so the When food intake is undesirable, in case it is prudent to
alpha particle, beta particle, and gamma rays would not be able to penetrate
the skin or any body part
rest the bowel.
o e.g., Post GIT surgery, chronic inflammatory diseases,
intractable diarrhea.
After surgery, the anesthesia still has an effect on the patient.
Thus, some degree of paralytic ileus may be observed.
TYPES OF DUR
Collecting the data from the past
RETROSPECTIVE
Ex. Doing a DUR using data from last year
Current data
CONCURRENT Ideal type because it involves the latest data
from the present time
ADVANTAGES
Identification of drug-drug interaction
Prevention of therapeutic duplication
Prevention of underdosing/overdosing
Improvement of quality of care
Encourage physicians to use more formulary drugs or
generic drugs
o Remember that the hospital pharmacy is a source of income for the
hospital. Hindi siya pwedeng malugi.
END
HOSPITAL PHARMACY | MODULE 3 14
TYPE A - AUGMENTED
Pharmacologically
Yes
predictable
Dose-dependent Yes
Incidence Common
Detection Early in clinical development
Mortality Low
1. If he cell is resting, the usual resting potential is negative (-) 90 mv or
Management Reduce dose negative (-) 70 mv.
2. When the cell is awake, there is a Na influx (+). When the sodium, which
is positive, goes inside the cell, the cell will become more positive. There is
now a depolarization.
HOSPITAL PHARMACY | MODULE 3 16
3. If all the sodium channels are open, potassium channels will open are dilated, it can cause
afterwards. There is a potassium efflux, which means the potassium goes hypotension.
out of the cell. Since potassium is positive (+) and sodium is also positive. ACE-inhibitors Cough
(+), the charge of the cell will now go back to negative (-). This called
the repolarization. Drugs ending in the suffix -
4. It will go back until it reaches the resting membrane potential. However, pril
if there is a Chloride, which is negative enters the cell, the cell will be more a. HMG CoA reductase
negative, leading now to hyperpolarization. b. Adenylate cyclase
o Enter of chloride inside the cell is called Chloride influx c. Kininase II
5. If the cell hyperpolarized, it will be difficult to reach the threshold, hence d. None of the above
the cell is considered to be in deep sleep. That is the mechanism of the Renin will convert Angiotensinogen
benzodiazepines, barbiturates Answer: Kininase II aka ACE
to Angiotensin 1(AT1). Then Ace
Anticoagulants Bleeding will convert AT1 to AT2. AT2 will be
E.g., Warfarin, Heparin, Mucosal (gum bleeding) the one binding to AT2 receptors
Kininase II or ACE
Enoxaparin GI (vomiting blood) ACE has another activity. It
Extension effect is
NOTES: degrades the bradykinin. Bradykinin
hypotension
Aspirin is not an anticoagulant, it is antiplatelet is a known lung irritant. Since
o When you have vessel damage, the 1st step in physiological hypertensive patients are given
hemostasis is vasoconstriction, 2nd step is the formation of platelet ACE-inhibitors, the bradykinin will
plug forming the scab, 3rd step is coagulation ending in a clot that not be degraded, hence causing
is stronger than the platelet plug cough to the patient. Since the
o Anti-platelet medication goes to the site of the injury forming the cough is not infectious in origin, it is
platelet plug. It targets vasoconstriction. The platelet plug is fragile a dry cough.
and can be removed. Coagulation then occurs forming the clot, that Minoxidil (Rogaine) Hirsutism
is where the anticoagulants work, they target the coagulation or the Vasodilator for angina Hair growth
clot formation. Extension effect is
Anticoagulants and antiplatelet hypotension
o Can both cause bleeding since they inhibit hemostasis Marketed now for its side
o Both drugs are given to prevent thrombus formation that may lead effect- for patients with
to embolus. alopecia
Embolus in the cerebral blood vessel - stroke
Embolus in the coronary blood vessel myocardial infarction
Antihistamies Sedation
Embolus in the pulmonary vasculature pulmonary embolism Antihistamines are for allergy. Usually caused by 1st generation
Embolus in the lower area blood vessel deep vein thrombosis However, histamines are antihistamines, but 2nd and 3rd
neurotransmitters for generations are less drowsy
Choosing between anticoagulants vs. antiplatelet
wakefulness. Hence inhibiting o First gen example:
o Antiplatelet is given first especially for patients with coronary artery
the histamines will cause Diphenhydramine,
disease.
sedations. chlorpheniramine
o Anticoagulants are given when there is known to be a thrombus
formation or valvular problems. Statins Rhabdomyolysis/Hepatotoxicity
o If major surgery, stop anticoagulants 1 week before and Statins inhibit HMG CoA Breakdown of own skeletal muscles
antiplatelets 10 days before (lifespan of platelet) then resume 72 reductase causing myalgia (muscle pain)
hours after surgery HMG-CoA reductase is the The breakdown product of the
Patient under Aspirin are requested to have their bleeding time monitored. rate-limiting enzyme for muscle is creatinine. So when you
o Warfarin Monitor prothrombin time (PT) cholesterol synthesis have rhabdomyolysis creatinine
o Heparin Monitor activated partial thromboplastin time (aPTT) levels will increase, which can cause
Antihypertensives Hypotension (< 90/60) intrinsic kidney injury and if not
Orthostatic hypotension is postural treated can lead to end stage renal
At the supine position, the heart and head are at the same level, but when failure.
you stand up the heart is lower than the head so it has to pump faster. If Rhabdomyolysis Lab: serum
creatinine, CKMM
Hepatotoxicity Lab: AST, ALT
People who are physically abused,
SIDE EFFECTS such as victims of hazing can also
Not pharmacologically related have rhabdomyolysis
DRUG SIDE EFFECT Aminoglycosides Nephrotoxicity
Opiates Constipation Protein synthesis inhibitors Neomycin, Tobramycin, Gentamycin
E.g., morphine for patients in Managed by laxatives like lactulose at the 30S ribosomal
subunit
severe pain. However, it may Acts on the mu receptors. However, Ototoxicity
cause respiratory failure when these receptors are acted upon, If the source of the
amiglycoside is Neomycin, Amikacin, Kanamycin
acetylcholine release in the myenteric If you have ototoxicity, you cannot
plexus that causes peristalsis is Strepotomyces, it ends with
hear my knock (NAK) anymore
decreased which causes paresis or
paralysis of the GIT is Micromonospora, it ends
Nitroglycerin Headache Vestibulotoxicity
Most toxic aminoglycoside: Streptomycin, Gentamicin
Vasodilator for angina Nitroglycerin are vasodilators for Tobramycin kasi tobra
the management of angina. Si Sarah G, laging naka vest
(sobra) tobrang toxic siya
However, it is not selective. It can
also dilate the cerebral and
peripheral blood vessels. If there is
cerebral vasodilation, this can cause TYPE B-BIZARRE
headache. Since the brain is a rigid
structure, vasodilation in the brain Pharmacologically
increases intracranial pressure, No
causing headache. On the other predictable
hand, if the peripheral blood vessels Dose-dependent Not clearly
Incidence Uncommon
HOSPITAL PHARMACY | MODULE 3 17
2. GENETIC FACTORS
Glucose-6-phosphate dehydrogenase (G6PD) deficiency
o We cannot give any medication just like in normal patients because
patients with G6PD are prone to have hemolytic anemia
Patients cannot be given certain drugs such as penicillins, NSAIDs,
sulfa- containing drugs, etc., and even food like beans
2 TYPES
1. IDIOSYNCRASY
Genetically determined
Porphyria Examples:
o Disease condition where there is a damage on the metabolism of o Malignant hyperthermia
hemoglobin. Patients with porphyria, when given medications, can have o Hemolytic anemia (G6PD deficiency)
hemolytic anemia, severe skin reaction, or mucosal damage o Stevens- Johnson Syndrome
o This is genetic, so they present with anemia, photosensitivity, problems PCP LAPSE: Phenytoin, Carbamazepine, Phenobarbital, Lamo-
with bone and mineral formation (broken teeth), and short, stubby trigine, Allopurinol, Penicillin, Sulfa drugs, Eryhtromycin
hands, hirsutism
HOSPITAL PHARMACY | MODULE 3 18
2. HYPERSENSITIVITY REACTIONS
TYPE II - CYTOTOXIC
TYPE I - ANAPHYLACTIC/IMMEDIATE
Rh Antigen or D antigen corresponds to the plus (+) sign. Anti-Smith Body (Anti-Sm) test is specific test for lupus
While,No Rh Antigen or D antigen corresponds to the negative Anti-dsDNA is a marker if the patient with Lupus has a renal involvement
(-) sign Discoid rash coin rash
Rh (D antigen) incompatibility has no effect on the first pregnancy
Malar rash butterfly rash; this crosses the nose bridge
- RH (+) father + RH(-) mother = RH(+) baby = mother
develops RH antibody in first pregnancy
Second pregnancy may cause the RH antibody to attach RH(+)
fetus, called HDN (hemolytic disease of the newborn)/
erythroblastosis fetalis
- reaction only happens at birth/ delivery Example of a Malar
Most Asians are RH positive. Rh negative is usually for Caucasians Rash
Management for HDN is Rhogam (Rh antibody) usually at the 28
gestation and 72 hours after birth
Blood type of the baby is determined at birth from the umbilical
cord
o Autoimmune hemolytic anemia
Fifth Disease or
Erethyma infectiosum
caused by parvo
virus
Not a malar rash. It
does not cross the
nose bridge
TYPE IV - DELAYED
T cell mediated only the T cell mediated
Sensitized TH1 cells release cytokines that activate
macrophages or TC cells which mediate direct cellular
damage
Happens not on the exposure but 72 hours after the exposure
Ag-Ab complexes deposited in various tissues induce Additional example is TB skin testing
complement activation and an ensuing inflammatory Typical manifestations:
response mediated by massive infiltration of neutrophils o Contact dermatitis
o Antigen Antibody complex or immune complex is usually produced at fake jewelry, poison ivy
the site or reaction. In a type III, the immune complex goes to other o Tubercular lesions
area of the body. So for example, when the immune complex goes to
the kidney, then inflammatory reaction in that organ will occur.
Typical manifestations:
o Localized Arthus reaction
o Generalized reactions (serum sickness, necrotizing
vasculitis, glomerulonephritis, rheumatoid arthritis,
systemic lupus erythematosus)
Diagnostic criteria for Lupus
TYPE C: CONTINUOUS
Uncommon
Renal involvement is lupus nephritis Dose and time-related
Antinuclear antibody (ANA+) is the usual test for probable lupus diagnosis o There is already chronicity on the use of the drug
HOSPITAL PHARMACY | MODULE 3 20
Cumulative dose of the drug If you are brewing coffee, then 1 cup would be around 25-30
mg but for decaffeinated coffee, there is only 0.25% of the total
These are effects that require a prolonged period of caffeine content (Decaffeinated coffee does not mean that there
exposure to develop is no more caffeine)
MOA: Taken as a stimulant because it inhibits adenosine
(antagonist to the adenosine receptor). When adenosine binds to
3 TYPES: the adenosine receptor in the brain, it causes sleepiness. Caffeine
binds to the adenosine receptors so that adenosine cannot bind
1. ADDICTION 2 types:
Person takes the drug compulsively despite o Physical Dependence - body cannot function well
potential harm and desire to stop o Psychological Dependence - mind cannot function well
Compulsion is the urge to take the drug because the drug abuser obtains
pleasure from taking these drugs 3. TOLERANCE AND TACHYPHYLAXIS
Pleasure is different from happiness
PLEASURE HAPPINESS Normal dose to Less/no effect
Dopamine Serotonin Tolerance means giving repeated dosing with normal does that causes less
effect of the drug. It develops through time.
Pleasure is peripheral, not long term, and Happiness is central, would
visceral (felt by the body) create good memories Tachyphylaxis is tolerance that is developed rapidly (fast-onset tolerance)
There is a feeling of fulfillment and excitement Example: Nicotine
that would make you want to do more o First time smokers feel satisfied, but after smoking for a few years, they
When you are eating your favorite food, you would feel that 1 stick is no longer enough, and so on
DISPENSING
PARTS OF THE PRESCRIPTION
PHARMACEUTICAL CARE (ASHP)
DATE
The direct, responsible provision of medication-related
care for the purpose of achieving definite outcomes that Rx is dated at the time they were written and also when
(patient-centered) they are received and filled in pharmacy.
o Normal Rx: 2 weeks
The goal of pharmaceutical care is to improve an
o Special Rx: 60 days
definite (predefined), medication-related therapeutic Important in establishing medication record of patient
outcomes. The outcomes sought are: especially in filling Rx for controlled substances
o Keep track of fills and refills
o (ultimate outcome)
o Important in establishing treatment history for patient
symptomatology. Necessary for prevention of misuse of prescription,
o Arresting or slowing of a disease process (e.g., cancer, especially ones which contain narcotics and controlled
and HIV since they have no cure) drugs
o Prevention of a disease or symptomatology (e.g., o After it is fully filled, the Rx cannot be used anymore (the pharmacist
vaccination) should keep it)
o Prevent antimicrobial resistance
The validity of the prescription depends on the appropriateness of the
DISPENSING medications t
The act of giving the medication to the patient; the soul of pharmaceutical Gives an idea of when the patient visited his/her doctor and explain to them
care practice why they need to consult their physician first before buying medicines
Types of prescription based on inscription Allows patients to choose their preferred brand based on the generic name.
As pharmacists, we must NEVER recommend brands.
o Compounded Rx
Imposed in dispensing, labeling, and advertising
Contains lists of ingredients to be used to
compound or prepare a certain medication or
INCORRECT PRESCRIPTIONS
dosage form
Extemporaneous compounding
o Rx for pre-fabricated product
Contains name, dosage form, and quantity to be Three types of Prescriptions based on general prescribing:
dispensed that have been prepared by the o Erroneous Prescription
manufacturer o Violative Prescription
Take the medicine from the shelf and dispense o Impossible Prescription
Types of ingredients when compounding an Rx Standard: Generic name (Brand name)
o Basis
Active pharmaceutical ingredient ERRONEOUS PRESCRIPTION
Elicit pharmacological or therapeutic activity Brand name precedes the generic name
o Adjuvant Generic name is the one in parenthesis
Enhances the effect or aids the basis Brand name is not in parenthesis
Ex: Aspirin (basis) + Phenyl salicylate (adjuvant)
o Corrective ADMINISTRATIVE ORDER NO. 90, S. 1990
Correct undesirable physical properties, e.g., , SUBJECT: Amendment to A.O. 62 s. 1989 (Rules and
conceal spots, mask bitter taste Regulations To Implement Prescribing Requirements)
Lessen side effect more than one
Aspirin (cause drowsiness) + Caffeine (corrective) drug product in one prescription form
o Diluent / Bulking agent
A prescription form with more than one drug product is no
Increases the bulk of the preparation longer regarded as erroneous
Different from vehicle (delivers the basis, aids in
administration)
VIOLATIVE PRESCRIPTION
Q.s. ad. water (diluent) vs add water in powder
for reconstitution (vehicle) Generic name is not written
Generic name is not legible and a brand name which is
SUBSCRIPTION (DIRECTIONS TO PHARMACIST) legible is written
Preparation of drug which requires compounding Brand name is indicated and instructions added (such as
No Substitution
Commercially manufactured products (pre-fabricated Rx):
or prevent generic dispensing
subscription designates dosage form & number of dosage
units to be dispensed.
o E.g. #60 tablets IMPOSSIBLE PRESCRIPTION
Only the generic name is written but is not legible
SIGNATURA/TRANSCRIPTION (DIRECTIONS TO PATIENT) Generic name does not correspond to the brand name
o E.g., Paracetamol (Alaxan)
Direction to patient on how to use the medicine
Both generic name and the brand name are not legible
Repeat verbally and clearly to the patient
The drug product prescribed is not registered with BFAD
Instructions on: o Counterfeit products (e.g., failed to renew CPR, online selling)
o When to take the medicine (before or after meals)
o Precautions RECORD BOOKS
o Simple instructions on how to take the medicine Poison Book 5 years after last entry
after dispensing the medication, we should also dispense information (tidbits Prescription Book 2 years after last entry
info) like take before meals, drink milk Dangerous Drugs Book 1 year after last entry
drive Where we
do not wait for the interns to do it. It is recommended to update it immediately
after dispensing the medications. Make sure that the books and stocks are
balanced.
Frequently abbreviated and are interpreted by
pharmacist and conveyed verbally to patient. It is then CLASSIFICATION OF DRUGS
prescription label
PRESCRIPTION / ETHICAL DRUGS
R. A. NO. 6675 (SEPTEMBER 13,1988) (GENERICS LAW) Dispensed upon written order of a validly-registered
licensed physician, dentist, or veterinarian
d
medicines identified by their generic names: For the management or treatment of a condition or
o Production of an adequate supply, disease.
o Distribution,
o Use, and
o Acceptance
DISPENSING, INCOMPATIBILITIES, PATIENT COUNSELLING (MODULE 3) CABIGAS, LUGO, BARROZO | 4B-PH 3
NON-PRESCRIPTION / OTC DRUGS NEVER partially fill a yellow prescription. In case the patient insists on
partial filling due to emergency (e.g., lives on another province), keep the
Dispensed even without the written order of a validly- prescription and instruct the patient to secure another one.
registered licensed physician, dentist, or veterinarian
Prevention or symptomatic relief of minor or self-limiting PROHIBITED DRUGS
ailments. Opium, its active components, & derivatives (Papaver
The patients can take the medications easily from the somniferum)
shelves, pay it, then go home without the supervision of the o Heroin, morphine
pharmacist Cocoa leaf & its derivatives
o Cocaine, alpha/beta eucaine
BEYOND-THE-COUNTER DRUGS o Erythroxylum coca (cocaine)
Familiar with available strengths & dosage forms Name of the Pharmacist
of prefabricated drug products Batch number (in case of product recalls or complaints)
colleagues first (co-pharmacist) before asking the patient. You should ask In Partial Filling of the Prescription.
the physician last. Information required to be written on the prescription:
Date of partial filling
3. NUMBERING & DATING THE PRESCRIPTION Quantity served & balance of the prescription
Place the same number on the label and record book as Name & address of the drugstore
desired. Additional requirements:
o Identifies the bottle or package. The prescriber must have an S-2 license (for narcotics)
o Reference of the original medication order.
The special DDB Prescription form must be used
Dating of the Prescription
A recording system following pertinent DDB regulation
o On the date filled/compounded to establish identity.
o On the date of partial filling (always stamp it) must be covered.
In dispensing drugs in List A and List B:
4. LABELING o Done by the pharmacist.
Prepare the label before compounding to avoid mix-ups and confusion. o Follow the order & instructions of the doctor on the
Right after compounding, you can just stick the label to the container. prescription.
Prescription labels should be o Partial filling of Prescription for drugs in List A:
o Computerized or typewritten - neat, attractive, and Prescription must be retained by the pharmacist.
legible The patient must ask the prescriber for another
o With aesthetic and professional appearing label prescription to complete the total dose of the
o Size in conformance with the size of the prescription medication.
container o After the Prescription is filled the original copy of the
Imprinted on the label prescription shall be retained & kept for a period of
o Name, address, and telephone number of the 1 year by the pharmacist for inspection.
pharmacy
Label of Compounded Prescription should include: 5.1. PREPARING/COMPOUNDING
o Number of Prescription Organize the method
o Date of filling Information necessary:
o Name of Patient o Adjuvant used
o Quality (name) and quantity of ingredients o Order of mixing
o Directions for administration (from transcription/signa) Dissolve first the least soluble ingredient in your solvent to
o Name of Physician maximize its dissolving capacity. If you start with the most
o Initials of the pharmacist soluble, the solvent will be easily saturated (it will occupy some
of the solvent particles), so it will be harder to dissolve the least
AUXILIARY LABELS provide safety & warning soluble
o Proper use o Amount of each ingredients
o Handling o Capsule size
o Storage Capsule Size Amount (grains) Powder capacity (mg)
o Refill status 5 1 60 130
o Necessary warnings and precautions 4 2 95 260
Mode of administration 3 3 130 390
o 2 4 195 520
o 1 5 225 650
o 0 7.5 325 910
o 00 10 390 1,300
o 000 (veterinary) 15 650 2,000
o Instructions to the caregiver o Type and size of container
Completely write the full words and do not abbreviate (e.g., teaspoon, Compatibility in a plastic or glass container
tablet, one and not tsp., tab, or 1) o Name and product identification number of
manufacturer.
GUIDELINES ON DISPENSING TO IMPLEMENT THE GENERICS pre-fabricated product in compounding
ACT OF 1988 E.g., Digoxin papertabs require Lanoxin tablets in compounding.
Thus, take note of its batch number and brand name.
Label of Unit Dose Prescriptions or dispensing products o Partially filled prescription
which are not in their original containers should include: Returned to the buyer after recording in the
o small bottles; tin cans; boxes; plastic or paper Prescription book
envelopes o Partial filling of prohibited or List A drugs
Shall not be allowed.
Name of the patient
Generic name of the drug
Brand name, if any
Manufacturer
Dosage strength
Expiry Date
Directions for use
DISPENSING, INCOMPATIBILITIES, PATIENT COUNSELLING (MODULE 3) CABIGAS, LUGO, BARROZO | 4B-PH 6
5.2. PACKAGING
Use appropriate containers/closures Dispensing price (DP)
o Closure: anything that seals/corresponds to the container; depends on = cost of ingredients
the container opening/neck % Mark-up Method
+ (cost of ing. × % mark-up)
o Colored or plastic containers
% Mark-up + Minimum = cost of ingredients
Plastic containers: Fee/Compounding Fee + (cost of ing. × % mark-up)
Lightness of weight, resistance to Method + min. fee/compounding fee
Advantages
breakage, versatile design = cost of ingredients
Permeable to atmospheric gases & Professional Fee Method
+ professional fee
moisture vapor NOTE: capsule shells are starting materials, NOT containers. Whatever method
Disadvantages dients.
Subject to leaching*
Deformed with extreme temperature
*Leaching: chemical composition of container contaminates the content 8. DELIVERING THE PRESCRIPTION
Sorption: reverse of leaching; the content attacks the container either Personally present the Rx medication to the patient
through absorption or adsorption
If personal delivery is not possible (delivered to the
Child-Resistant Containers
o Prevent accidental poisoning resulted from ingestion o Appropriate instructions are provided.
of medication and other household chemicals by o Encourage the patient to call should there be any
children. questions.
o All legend drugs must be dispensed in containers
having safely closures. FINISHED PRESCRIPTION
o Exempted preparations:
Oral contraceptives Correctly and skillfully compounded
In compounding, there are no quality control tests. The only way to ensure
Cardiac drugs (antihypertensive, myocardial infarction) quality is to correctly follow the procedures and add the ingredients at the
Epileptic medications; Anti- correct amount.
Products for childless person, arthritic, and Physical appearance
debilitated patients o Indicate the pride and care the pharmacist has taken
Products used in institutionalized settings in his professional work.
6. RECHECKING
All details of the label against the prescription order
o T
number, date, and
o Quality assurance; you want to give the right drug to the right patient
at the right time. It is checked by the pharmacist, pharmacy assistant,
cashier, etc.
Verify ingredients & amount.
Avoid wrong dispensing, which can lead to license revocation and patient
harm.
7. PRESCRIPTION PRICING
Fair/equitable profit/pricing code
o Not too pricey, but profit must be considered. There should be
balance.
EXPLOSIVE COMBINATION o Due to removal of oxygen from the air in the bottle
Oxidizing agents are chemically incompatible with by oxidation of syrup
reducing agents (redox reactions) Example: Bottles of a syrup
o Serious explosions may result from certain
combinations. OTHER TYPES OF CHEMICAL CHANGE
All oxidizing agents such as potassium chlorate, chromic Incompatibilities such as polymerization, double
acid, potassium permanganate, silver oxide, hydrogen decomposition, substitution, addition, etc.
peroxide, nitric acid or sodium peroxide will explode with
organic matter or oxidizable inorganic matter like sulfur THERAPEUTIC INCOMPATIBILITIES
and carbon Occur when two drugs or more drugs, IV fluids or both are
Strong nitric acid produces effervescence or explosion with administered together to produce a response which differs
tannins or with oil of turpentine in nature or intensity from that which was intended.
Hypophosphites may explode heated above 100 C or Occur at the site of the drug action.
when combined with nitrates, chromates, or Maybe intentional or beneficial
permanganates o E.g. morphine (cause respiratory depression) + atropine
Iodine may explode when treated with ammonia or with (prevents respiratory depression)
oil of turpentine. o Opium (cause constipation) + calomel (soften stool)
May also increase toxicity
CEMENTATION o Emetine + antimony
Rx may set a mass of cement-like hardness. o Epinephrine + cocaine
Occurs when compounds from hydrates (ex. Plaster of o Castor oil or fixed oil + santonin or aspidium
Paris) polymerize or convert to new crystal forms. oleoresin (oil increases solubility of santonin)
Acacia + Bi salts
COMBINATIONS LIABLE TO PRODUCE THERAPEUTIC
SEPARATION OF IMMISCIBLE LIQUIDS INCOMPATIBILITIES
Immiscible liquids not soluble in the prescription Sedatives + stimulants Caffeine + chloral hydrate
(sedative; knockout drug)
E.g., decomposition of chloral hydrate by the alkali into Demulcents + irritants Atropine + morphine
chloroform Laxative + astringents Tannin + aloin
Strychnine + barbital derivatives
GELATINIZATION Antagonistic effects
Solutions form a gel when combined with certain
substances. CONSEQUENCES
Therapeutic effectiveness
DEVELOPMENT OF HEAT OR COLD Reduced or delayed
Chemical reactions with either:
Loss of activity (therapeutic failure)
Liberation of heat (compounds will decompose) or
Delay in the release or absorption of drug
Absorption (form stable compounds) of considerable
amounts of heat. RESPONSIBLE
Physician rather than the pharmacist but the pharmacist
HYDROLYTIC CHANGES
may inform the physician to eliminate:
Effects of water
o Errors in prescription writing/interpretation
Many substances hydrolyze in water & the change may be o Overdose
hastened by heat, catalyst, esters, amides, certain metals
excessive single dose
(Zn, Fe), glycosides
too frequent administration
o Contraindicated drugs (steroids/peptic ulcer)
INVISIBLE CHANGES o Synergistic/Antagonistic effects
Chemical changes occur without visible evidence of the Alteration of prescription order requires permission of
reaction. prescriber.
Risky since there is no manifestation. You can only base it on the o Always make sure to have a current, evidence-based reference
physicochemical properties of the drug
The more diluted the drugs are in a Prescribing error, dispensing error, and administration error
Degree of Pharmacovigilance
solution, lessens ion interaction leading to
dilution o Science and activities relating to the detection,
incompatibility
Incompatibility increases with the length of assessment, understanding and prevention of adverse
Length of time effects or any other drug-related problem.
time that drugs are in contact with each
in solution
other
Ex. Calcium phosphate, should not be o consists of reported information on a possible causal
added consecutively when an IV relationship between an adverse event and a drug,
admixture is being prepared. the relationship being unknown or incompletely
This keeps these substances from documented previously.
Order of pooling, or forming a layer on top of
mixing the IV fluid, decreasing the chance of DI: ADVERSE DRUG REACTIONS
incompatibility.
o Prevents formation of precipitate CLASSIFICATION BY SEVERITY (KARCH AND LASAGNA)
Thorough mixing after each addition is No antidote, therapy, or prolongation of
essential. Minor
hospitalization is required in response to the ADR.
The management of the ADR requires a change in
DRUG INTERACTIONS Moderate drug therapy, specific treatment, or an increase in
Frequently applied to those situations: hospitalization by at least 1 day.
o Effects of one drug are altered by the prior or The ADR is potentially life threatening, causing
concurrent administration of another (drug-drug Severe permanent damage, or requiring intensive medical
interaction) care.
o Dietary item influences the activity of a drug (e.g., The ADR directly or indirectly contributes to the
cheese & monoamine oxidase inhibitors) (drug-food Lethal
death of the patient.
interaction)
o A drug causes alterations of laboratory test values
(drug-lab interaction) RISK FACTORS FOR ADRs
o A drug essentially interacts with itself (e.g. by 1. Age
stimulating its own metabolism) o Pediatric and geriatric patients are more likely to experience ADRs
May either be: since their organs are underdeveloped or not functioning well
o Adverse Drug Interaction 2. Gender
o Not common, but there are certain drugs wherein women are more
o Beneficial Drug Interaction sensitive than men, vice versa
3. Comorbidities
TERMINOLOGIES o Certain drugs are contraindicated to certain diseases
Adverse Event 4. Polypharmacy/ concurrent medications
o Any undesirable experience associated with the use 5. Duration of therapy
of a medical product in a patient 6. Narrow therapeutic index (margin of safety)
o E.g., loss of balance after taking a medicine 7. Ethnicity and Genetics (Pharmacogenomics)
Adverse Drug Event o Drugs give a different response depending on the genetic composition
or ethnicity
o An injury resulting from the use of a drug
E.g., loss of balance and spraining yourself after taking a
o Genetic polymorphism [N-acetyltransferase (NAT)]
medicine The slow acetylator experiences toxicity from
o Includes harm caused by the drug (adverse drug isoniazid, sulfonamides, procainamide, and
reactions and overdoses) and harm from the use of hydralazine (HIPS).
the drug (including dose reductions and Fast acetylator may not respond to isoniazid and
discontinuations of drug therapy). hydralazine in the management of tuberculosis
o May result from medication errors but most do not. and hypertension. (Asians)
Adverse Drug Reaction o Idiosyncrasies (G6PD Deficiency)
o Response to a drug which is noxious and unintended Hemolytic anemia may be triggered by
and occurs at doses normally used in man for sulfonamides or fava beans. (G6PD is needed for
prophylaxis, diagnosis, or therapy of disease or for regeneration of reduced glutathione)
IMPLICATIONS: When doctors present the diagnosis, pharmacists should
suggest the best drug that would fit the patient considering their genetic
o Harm directly caused by the drug at normal doses, composition
during normal use.
Different from toxicity or overdose (beyond normal doses)
CLASSIFICATIONS OF ADR (ABC SYSTEM)
Side Effect
o An expected and known effect of a drug that is not Type A (Augmented)
the intended therapeutic outcome. o Actions related to the pharmacological activity of the
o Common, reproducible, predictable, dose-dependent, drug (extended or side effects)
rationalizable through drugs pharmacology. Dose related responses arising from
Medication Error an extension of therapeutic effect
o Any preventable event that may cause or lead to Extension Antihypertensive : hypotension
effects Sedative-hypnotics (taken at night) : daytime
inappropriate medication use or patient harm while somnolence
the medication is in the control of the health care Beta-blockers : hypotension or bradycardia
professional, patient, or consumer. Insulin/sulfonylurea : hypoglycemia
DISPENSING, INCOMPATIBILITIES, PATIENT COUNSELLING (MODULE 3) CABIGAS, LUGO, BARROZO | 4B-PH 11
DRUG INTERACTIONS In general, we can advise patients taking antacids with another drug that
there should be adequate spacing in dosage intake such that one drug is not
affected by the other.
DRUG-DRUG INTERACTIONS
Duplication COMPLEXATION
o potentiation of effect when 2 drugs with the same Complex formation (chelates) which are not absorbable by the body.
active ingredient or with the same action (synergistic Quinolones/tetracycline + Fe, Ca, Mg, Al, Bi, Zn
action) are taken at the same time.
Antagonism ALTERATION OF MOTILITY / GER
o reduction of efficacy when two drugs with opposing Increase in GI motility leads to faster transit time from the stomach to
intestine.
actions are taken together. o Site of absorption is stomach = increased motility = decreased
Alteration of Pharmacokinetic Actions (ADME) therapeutic effect
o a second drug may increase or decrease the rate o Site of absorption is stomach = decreased intestinal tone = stays
longer in the stomach = increased therapeutic effect or
during ADME of first drug. overdosage/toxicity
o Site of absorption is intestine = increased motility = increased
PHARMACODYNAMIC INTERACTIONS therapeutic effect or overdosage/toxicity
o Site of absorption is intestine = decreased intestinal tone = stays
longer in the stomach = decreased therapeutic effect since drug is
ADDITION: 1 + 1 = 2 disrupted/destroyed by gastric fluids
The effect of one drug is added to the effect of another drug (combined
effect).
Cathartic/laxatives increase motility
GER (Gastroesophageal reflux) tendency to have lesser absorption
Prazosin + Beta-blocker Orthostatic hypertension
Non-dihydropyridine calcium Blockade of Anticholinergic
channel blocker + Beta-blocker atrioventricular node
Antidepressant + Azithromycin Cardiac arrythmias ALTERATION OF GI FLORA
Mostly antibiotics can disrupt the GI flora.
SYNERGISM: 1 + 1 = 3 Antibiotic + Digoxin Increase in digoxin level
The effect of one drug combined with the effect of another drug results to a Antibiotic + Warfarin Increase anticoagulation
pharmacologic effect which is more than what was expected.
Alters enterohepatic circulation of
Sulfamethoxazole (SMZ) and trimethoprim (TMP) Antibiotic + OCP
(Oral Contraceptive Pills)
estrogen leading to the decrease
Sulfadoxine and Pyrimethamine in concentration of OCP
REMEDY: Have spacing and do not administer them concurrently.
ANTAGONISM: 1 + 1 = 0
Common; the effect of a drug is cancelled by the presence of another drug DISTRIBUTION
Propranolol + Albuterol One drug is enhancing or inhibiting/blocking the distribution of another drug.
OHA + Glycocorticoids
L-Dopa + Neuroleptics DISPLACEMENT FROM PROTEIN BINDING SITE
Tetracycline + Penicillin Some drugs are protein-bound first before it becomes a free drug.
However, if another drug is already occupying the intended receptor of the
drug, it free drug.
POTENTIATION: 1 + 0 = 2 o Free drug = Increased drug concentration in the body.
There is a drug with effect (1) and drug with no effect (0). The drug with no Increase in free Warfarin occupies
effect enhances the effect of the drug with pharmacologic activity. Phenytoin +
phenytoin leading to phenytoin receptors,
Amoxicillin (antibiotic) + Clavulanic acid (inhibits excretion of Warfarin gingival hyperplasia displacing phenytoin
amoxicillin) = Co-amoxiclav (amoxicillin stays longer in the body) Tolbutamide + Both hypoglycemic
Loop diuretic + Silver = ear fluid imbalance ototoxicity Hypoglycemia effects
Sulfonamide
First, epinephrine will
ELECTROLYTE CONCENTRATION constrict the blood
The drug depends on the presence of electrolytes. Vasoconstriction vessels around the
Epinephrine + Beneficial drug certain body area.
Digoxin + Non potassium sparing diuretics = digoxin toxicity Lidocaine interaction for local Then, inject lidocaine,
o Potassium controls the entry of digoxin in the cells through potassium anesthesia. which will exert its
channels. Lower concentration of potassium in the body may lead to effect locally instead
digoxin toxicity. of systemically.
Lithium + diuretics Increase in the Low albumin = less
proteins for drugs to
Hypoalbuminemia concentration of bind = more free
PHARMACOKINETIC INTERACTIONS free drug drugs
ABSORPTION METABOLISM
One drug is enhancing or inhibiting/blocking the absorption of another drug. One drug is enhancing or inhibiting/blocking the metabolism of another.
The most common enzyme responsible for metabolizing drugs is cytochrome
ALTERED PH P450.
Drugs are only soluble, stable, or absorbed at a certain pH. Change in CYP450 inducers = metabolized faster = decreased therapeutic effects
CYP450 inhibitors = stay longer in the body = increased therapeutic
effects; may lead to overdosage/toxicity
Ketoconazole + antacid = decreased drug bioavailability
Bisacodyl + antacid
Antacids make gastric fluids less acidic and more alkaline/basic.
DISPENSING, INCOMPATIBILITIES, PATIENT COUNSELLING (MODULE 3) CABIGAS, LUGO, BARROZO | 4B-PH 14
ENZYME INDUCER ENZYME INHIBITOR The ff. are effects of drugs on food:
Carbamazepine Cimetidine, Ciprofloxacin, o neutralize drug effects
Cigarette smoking Chloramphenicol o changes in gastric emptying
Chronic alcoholism Acute alcoholism o drug chelation
Griseofulvin Valproic acid, Verapamil o changes in the activity of drug metabolizing enzymes
Rifampicin Erythromycin (macrolides o changes in splanchnic blood flow and plasma protein
Omeprazole except Azithromycin) binding
Phenobarbital, Phenytoin Disulfiram, Diphenhydramine o food reduces bioavailability
S Grapefruit Pharmacist should give proper advice, to avoid the
Isoniazid (INH) potential adverse drug reaction on whether to:
Metronidazole o eliminate the interacting food altogether
Ketoconazole o adjust the time of intake to allow spacing between intake of
Disulfiram is used to treat chronic alcoholism. It inhibits aldehyde dehydrogenase, food and of the drug
increasing the serum acetaldehyde concentration, which is responsible for the
deleterious effect (flushing, tachycardia, hyperventilation, and hypotension). This
discourages the patient to take alcohol. EXAMPLES OF FOOD THAT SHOULD NOT BE TAKEN WITH
DRUGS AS A GENERAL RULE:
Increases the risk of liver damage,
Alcohol increase drowsiness &/or sedation, or
cause nausea
A CNS stimulant, alters the action of many
drugs affecting the CNS depending on
Caffeine
whether the drug is sympathetic or
parasympathetic
Causes enzyme induction whereby the
biotransformation of some drugs are
Grapefruit hastened
EXCRETION
Causes enzyme inhibition whereby the
One drug is enhancing or inhibiting/blocking the excretion of another drug. biotransformation of some drugs are altered/deterred
pH: ionized compounds (acidic) are more readily excreted Dairy product or any product containing
Amoxicillin (antibiotic) + Clavulanic acid (inhibits excretion of Ca, Fe, Mg, Al, and other heavy metals
amoxicillin) = Co-amoxiclav (amoxicillin stays longer in the body) (like antacids & multivitamins), forms a
Milk
Methamphetamine (weak base) + Vitamin C = faster chelate with the drug rendering both the
excretion of methamphetamine drug & the heavy metal non-usable by the
body
DRUG-MEDICINAL HERBS INTERACTIONS Drug that Patients are advised to take the
Food supplements may act as drugs, but they do not have approved causes gastric medication with milk or crackers or with a
therapeutic claims because they did not undergo clinical trials. irritation full stomach
o Should they undergo clinical trials and passed, they become drugs. Can alter/disrupt the GI flora
May intensify or reduce the efficacy of a drug or cause a Almost always taken with an empty
serious side effect. stomach unless the patient complains of
Antibiotics
Avoided by consulting the doctor before taking gastric irritation
supplements. If the patient complains of GI irritation, advise them to
take a light meal such as crackers/biscuits
DIETARY SUPPLEMENTS OR FOOD SUPPLEMENTS
Alternative therapy most commonly used, includes: Biphosphates + food Decrease BA
o Medicinal herbs or herbal drugs May decrease BA except
o Nutraceuticals Penicillin and Tetracycline
Anti-infectives + any food
Natural substances that include certain herbs, such which should be taken 2 hrs.
products as cholesterol-lowering margarines, a.c. (before meals)
psyllium-fortified products Erythromycin stearate Taken before meals
Therapeutic claims not scientifically studied & Erythromycin ethyl succinate Taken before or after meals
evaluated by the FDA Fiber interfere with its
Digoxin + oatmeal
absorption
+ cheese, tyramine,
DRUG-FOOD INTERACTIONS yogurt, sour cream, cured HTN due to increase in
May result in any of the following: meat, liver, caviar, dried fish, catecholamine
o Delayed/reduced absorption of the food nutrients or avocado, banana, red wine
the drug. Warfarin + broccoli, spinach, Decrease anticoagulant
o Enzyme inhibition or induction resulting in delayed or kale, vitamin K effect
hastened drug elimination. Metronidazole + alcohol Disulfiram-like effect
o Reduced plasma concentration of the food nutrient or Griseofulvin/Phenytoin + fatty
Increase BA
the drug resulting in decreased therapeutic effect. foods
o Increased or decreased action of the medication or BA = Bioavailability
inactivation of the medication.
DISPENSING, INCOMPATIBILITIES, PATIENT COUNSELLING (MODULE 3) CABIGAS, LUGO, BARROZO | 4B-PH 15
10 STAR PHARMACIST
1. Communicator Talking to patients, doctors, other professionals
Research does not only happen within the
2. Researcher laboratory. Data is rich in the Pharmacy practice,
especially in community pharmacy
3. Agent of positive -
change
4. Manager In some drugstores pharmacists act as managers
Pharmaceutical care is the direct, responsible
5. Pharmaceutical provision of medication-related care for the
care giver purpose of achieving definite outcomes that
improve a patient's quality of life.
6. Decision maker medication knowing the diagnosis of the doctor?
Pharmacy is not only about patient care but a
7. Entrepreneur business where you have to protect the interest of
the company
8. Leader When it comes to provision of drug information
Be updated on current journals, pharmaceutical
magazines, brochures. Always visit the FDA
9. Life-long learner website for updates on drugs.
Graduate studies (MD, PhD, MSc, etc.)
10. Teacher Teach patients, doctors, other professionals
CRAMP DELLT
DRUG-FOOD INTERACTIONS
DRUG-DISEASE INTERACTIONS
STORAGE AND LABELLING o Never pour any liquids into a pre-labelled container
as this risks spoiling the label with drips of the
REQUIREMENTS medicament.
SECURE
STORAGE o Ensure that the label is secure before dispensing the
All products dispensed extemporaneously require some product to the patient.
form of additional storage instructions to be detailed on o The main reason for labels not sticking to product
the label. containers is because of a dirty or greasy container.
This information can be the addition of just a product
expiry date through to a number of important additional INFORMATION ON THE LABEL
auxiliary labels. LEGIBLE
o Always check label print size and quality to ensure
GENERAL PRINCIPLES OF LABELLING that it can be read clearly.
o If there is too much information to place on one label,
Every extemporaneously prepared preparation will
consider placing the additional information on a
require a label to be produced before the product can be
secondary label, rather than reducing the size of the
dispensed or sold to the patient.
print or trying to include too much information on one
The accuracy of the label is paramount as it conveys label.
essential information to the patient on the use of the
CONCISE
preparation.
o Although it is important that sufficient information is
Although the pharmacist or other healthcare practitioner placed on the label, it must be remembered that it is
may counsel patients when the medication is handed over, important not to confuse the patient by placing too
it is unlikely that patients will be able to remember all the much information on the label.
information that they are given verbally. o If the label contains too much information, rather than
The label therefore acts as a permanent reminder of the assisting patients, they may feel overwhelmed and as
key points that patients need to know. a result they may read none of the information.
ADEQUATE
FUNCTIONS OF THE LABEL OF A PHARMACEUTICAL o Ensure that sufficient information is given. For
PRODUCT requir
To indicate clearly the contents of the container. how much? How often? When required for what?
To indicate clearly to patients how and when the medicinal INTELLIGIBLE
product should be taken or used. o The wording of the information on the label must be in
To indicate clearly to patients how the product should be plain English or the vernacular, be easily
stored and for how long. understandable and use unambiguous terms.
To indicate clearly to patients any warnings or cautions of o It must always be remembered that patients may feel
which they need to be made aware. embarrassed to ask for further clarification on the
meaning of complicated words used on the label.
APPEARANCE OF THE LABEL ACCURATE
CORRECT POSITION o It is important that the title is accurate, the instructions
o Medicine bottles: The label should be on the front of are accurate and that the patient name is complete
a medicine bottle about a third of the way down the and accurate.
container.
The front of an internal bottle is the curved side
and the front of a fluted bottle is the plain side.
o Cartons: The label should be placed on the large side
of the carton.
If there is not enough room on a single side of the
carton for the entire label, it should be placed
around the carton, ensuring that all the
information is visible.
o Ointment jars: The label should be placed on the side
of the jar, ensuring that the contents of the label are
visible when the top is placed on the jar.
Ensure that the patient can open the container
without destroying the label (e.g. when labelling
cartons).
Ensure the label is positioned with care and is
straight, not crooked.
CLEAN
o Ensure the container is clean before packing the
product, then clean the outside before affixing the
label.
DISPENSING, INCOMPATIBILITIES, PATIENT COUNSELLING (MODULE 3) CABIGAS, LUGO, BARROZO | 4B-PH 21
o Macromolecular solutes, if dissolved in water will form as possible the reverse may result in separation of any
aggregates, so use wetting agents or add dissolved components.
macromolecules increments.
5. Isotonicity: 0.9% NSS SUSPENSION
o For parenteral solutions Liquid preparation with solid particles dispersed through a
o If not isotonic, will cause cellulysis or irritation liquid phase
o Iso-osmotic= same osmotic pressure o Solid = insoluble
o PNSS, RBC, and Blood Plasma: o Suspending agent = facilitates dispersion
-0.52°C Freezing point Preparation where at least one of the active ingredients is
Isotonic, iso-osmotic suspended throughout the vehicle.
6. Toxicity At least one of the ingredients is not dissolved in the
o Primary consideration when selecting solvent vehicle and so the preparation will require shaking before
7. Stability of the Preparation a dose is administered.
o Shelf-life of the product must be taken into account
o Add preservative, antioxidant, antimicrobial agent Common pharmaceutical products that are suspensions
o If instability is due to polymerization, add include:
polymerization inhibitors. o Ear drops
o Enemas
o Inhalations
Solubility each drug must be dissolved in a suitable solvent. o Lotions
The salt form of the drug and not the free acid or base form o Mixtures for oral use.
is used.
Flavoring and sweetening agents must be prepared ahead GENERAL CHARACTERISTICS OF SUSPENSION
of time.
Some suspension should contain antimicrobial agents to
When adding a salt to a syrup, salt should be dissolved first serve as preservatives
in a minimum amount of water.
The particles settle in all suspension even when a
Proper vehicle must be selected. suspending agent is added. (Shake well).
Tight containers are necessary to ensure stability of the
ORAL SOLUTIONS product. Upon reconstitution, it can be stored for 7 days at
Oral liquids containing one or more active ingredients room temperature or 14 days when refrigerated.
dissolved in a suitable vehicle. Principles to keep in mind when compounding suspension
o The insoluble powder should be small and uniform in
SOLUTIONS AS DOSAGE FORMS size to decrease rate of settling.
o The suspension should be viscous.
ADVANTAGES o Topical suspensions should have a smooth impalpable
Drug available immediately for absorption texture.
Flexible dosing o Oral suspensions should have pleasant odor and taste.
May be designed for any route of administration
ORAL SUSPENSIONS
No need to shake container
Oral liquids containing one or more active ingredients
Facilitates swallowing in difficult cases
suspended in a suitable vehicle.
Suspended solids may slowly separate on standing but are
DISADVANTAGES
easily redispersed.
Drug stability often reduced in solution
Difficult to mask unpleasant tastes SUSPENSIONS AS DOSAGE FORMS
Bulky, difficult to transport and prone to container
breakages ADVANTAGES
Technical accuracy needed to measure dose on Insoluble drugs may be more palatable.
administration Insoluble drugs may be more stable.
Some drugs poorly soluble Suspended insoluble powders are easy to swallow.
Measuring device needed for administration The suspension format enables easy administration of bulk
insoluble powders.
FURTHER CONSIDERATIONS DURING THE PREPARATION
Absorption will be quicker than solid dosage forms.
OF A SOLUTION
1. To aid dissolution, high-viscosity liquid components should Lotions will leave a cooling layer of medicament on the
be added to those of lower viscosity. skin.
2. Completely dissolve salts in a small amount of water prior It is theoretically possible to formulate sustained-release
to the addition of other solvent elements. preparations.
3. In complex solutions, organic components should be
dissolved in alcoholic solvents and water-soluble DISADVANTAGES
components dissolved in aqueous solvents. Preparation requires shaking before use.
4. Aqueous solutions should be added to alcoholic solutions Accuracy of dose is likely to be less than with equivalent
with stirring to maintain the alcohol concentration as high solution.
DISPENSING, INCOMPATIBILITIES, PATIENT COUNSELLING (MODULE 3) CABIGAS, LUGO, BARROZO | 4B-PH 23
Storage conditions can affect disperse system. The emulsifying agent ensures that the oil phase is finely
Suspensions are bulky, difficult to transport and prone to dispersed throughout the water as minute globules.
container breakages. o T -in-
o The oily phase (disperse phase) is dispersed through
DIFFUSIBLE AND INDIFFUSIBLE SUSPENSIONS the aqueous phase (continuous phase).
Generally all oral dose emulsions tend to be oil-in-water
DIFFUSIBLE SUSPENSIONS as the oily phase is usually less pleasant to take and more
Suspensions containing light powders which are insoluble, difficult to flavor.
or only very slightly soluble in the vehicle, but which on -in-
shaking disperse evenly throughout the vehicle for long be those with external uses.
enough to allow an accurate dose to be poured.
2 KINDS
INDIFFUSIBLE SUSPENSIONS o/w oil is internal; water is external; creamy, white
Suspensions containing heavy powders which are insoluble w/o water is internal; oil is external; glassy translucent
in the vehicle and which on shaking do not disperse evenly *** if yellow emulsion, emulsion is coarse
throughout the vehicle long enough to allow an accurate
dose to be poured.
The two liquids in an emulsion are immiscible and require
In the preparation of indiffusible suspensions, the main
the use of emulgent.
difference when compared to diffusible suspensions is that
the vehicle must be thickened to slow down the rate at Emulsions are unstable and the following steps must be
which the powder settles. This is achieved by the addition taken to prevent separation into two layers
of a suspending agent. o The correct proportions of oil and water must be used
during preparation. The internal phase should
CHOICE OF SUSPENDING AGENT represent about 40-60% of the total volume.
o The emulgent is needed for emulsion formation.
The amount of suspending agent used in any given o Homogenizer may be used to reduce the size of the
formulation depends on the volume of vehicle being globules of the internal phase.
thickened. o Preservatives should be added if the preparation is
It does not vary with the amount of powder in the intended to last longer than a few days.
preparation. o Shake well label must be provided
A suspending agent is intended to increase the viscosity of o The product should be protected from the light and
the vehicle and therefore slow down sedimentation rates. extreme temperature. Avoid freezing and heating.
This outcome could also be achieved by decreasing the o If the addition of flavor is needed to mask the taste of
particle size of the powder in suspension. the oil phase, the flavor should be added to the
The most common suspending agents used in external phase before emulsification.
extemporaneous dispensing are
Tragacanth Internal or external suspensions ORAL EMULSIONS
Internal suspensions Oral liquids containing one or more active ingredients.
Compound
15% Tragacanth, 20% Acacia, They are stabilized oil-in-water dispersions, either or both
Tragacanth Powder
20% Starch and 45% Sucrose phases of which may contain dissolved solids.
Bentonite External suspensions Solids may also be suspended in oral emulsions.
When issued for use, oral emulsions should be supplied in
WAYS TO MINIMIZE STABILITY PROBLEMS wide-mouthed bottles.
Particle size of all powders used in the formulation must be
reduced. EMULSIONS AS DOSAGE FORMS
A thickening agent may be used to enhance viscosity
(Bentonite 6%, Veegum 6%, Acacia 1%, Tragacanth 1- ADVANTAGES
3%, Na alginate 1-2%, MC 1-7%)
Unpalatable oils can be administered in palatable form.
A levitating agent may aid in the initial dispersion or
insoluble particles. (glycerin, PG, alcohol, syrup, water) Unpalatable oil-soluble drugs can be administered in
palatable form.
Flavoring agents and preservatives should be selected
and added if the product is intended for oral use. The aqueous phase is easily flavored.
The source of active ingredients must be considered. (b? The oily sensation is easily removed.
powders/ tablets/ capsule) The rate of absorption is increased.
It is possible to include two incompatible ingredients, one
EMULSION in each phase of the emulsion.
Liquid preparation in which one liquid is dispersed in
another liquid in the form of small droplets DISADVANTAGES
o 2 immiscible liquids Preparation needs to be shaken well before use.
o Oil, water, and emulgent A measuring device is needed for administration.
Essentially a liquid preparation containing a mixture of oil A degree of technical accuracy is needed to measure a
and water that is rendered homogeneous by the addition dose.
of an emulsifying agent.
DISPENSING, INCOMPATIBILITIES, PATIENT COUNSELLING (MODULE 3) CABIGAS, LUGO, BARROZO | 4B-PH 24
A cream is always miscible with its continuous phase. be allowed to cool and triturated with the powder/cream
Formulated to provide preparations that are essentially mixture on the tile.
miscible with the skin secretion. Fine powders may be triturated into the otherwise finished
Intended to be applied to the skin or certain mucous cream on a glass tile. Small amounts of powder should be
membranes for protective, therapeutic, or prophylactic added to an equal amount of cream, i.e. using the
purposes, especially where an occlusive effect is not -
necessary.
THE INCORPORATION OF LIQUIDS INTO A CREAM BASE
WATER-IN-OIL CREAMS (OILY CREAMS) AS BASES Non-volatile, miscible liquids may be mixed with the
Produced by emulsifying agents of natural origin, e.g. molten cream in the evaporating basin. Alternatively, if a
beeswax, wool alcohols, or wool fat. pre-prepared base is used, then incorporate as for
These bases have good emollient properties. They are volatile or immiscible liquids.
creamy, white, or translucent and rather stiff. Volatile or immiscible liquids, e.g. coal tar solutions, should
be triturated with the cream on the glass tile. Always
OIL-IN-WATER CREAMS (AQUEOUS CREAMS) AS BASES remember that volatile ingredients should not be added to
molten bases.
Produced by synthetic waxes, e.g. macrogol and
cetomacrogol.
OINTMENTS
They are the best bases to use for rapid absorption and
penetration of drugs. Formulated to provide preparations that are immiscible,
miscible or emulsifiable with the skin secretion.
They are thin, white, and smooth in consistency.
HYDROPHOBIC OINTMENTS and water-emulsifying
ointments are intended to be applied to the skin or certain
GENERAL METHOD IN PREPARING CREAMS
mucous membranes for emollient, protective, therapeutic,
or prophylactic purposes where a degree of occlusion is
TRITURATION desired.
The term applied to the incorporation, into the base, of HYDROPHILIC OINTMENTS are miscible with the skin
finely divided insoluble powders or liquids. secretion and are less emollient as a consequence.
The powders are placed on the tile and the base is
incorporated using the - PASTES
Liquids are usually incorporated by placing a small Semi-solid preparations for external use.
amount of ointment base on a tile and makin They consist of finely powdered medicaments combined
the center. with White Soft Paraffin or Liquid Paraffin or with a non-
Small quantities of liquid are then added and mixed in. greasy base made from glycerol, mucilage, or soaps.
Trituration can be successfully achieved using a mortar but Pastes contain a high proportion of powdered ingredients
this method is usually reserved for large quantities. and therefore are normally very stiff.
Because pastes are stiff, they do not spread easily and
LEVIGATION therefore this localizes drug delivery.
The term applied to the incorporation into the base of This is particularly important if the ingredient to be
insoluble coarse powders. applied to the skin is corrosive such as dithranol, coal tar
or salicylic acid.
It is the process where the powder is rubbed down with It is easier to apply a paste to a discrete skin area such as
either the molten base or semi-solid base. a particular lesion or plaque and not therefore
A considerable shearing force is applied to avoid a gritty compromise the integrity of healthy skin.
product. Useful for absorbing harmful chemicals such as the
ammonia which is released by bacterial action on urine
THE INCORPORATION OF SOLIDS INTO A CREAM BASE and so are often used in nappy products.
Soluble solids should be added to the molten cream at the Because of their high powder content, they are often used
lowest possible temperature and the mixture stirred until to absorb wound exudates.
cold. Because pastes are so thick, they can form an unbroken
Insoluble solids should be incorporated using a glass tile layer over the skin which is opaque and can act as a sun
and Spatula. If there is more than one powder to be filter. This makes them suitable for use for skiers as they
added, these should be triturated together in a mortar prevent excessive dehydration of the skin (wind burn) in
using - addition to sun blocking.
glass tile. The principal use of pastes was traditionally as an
Coarse powders. A minimum quantity of cream should be antiseptic, protective or soothing dressing.
placed in the center of the glass tile and used to levigate Often before application the paste was applied to lint
the powders. A considerable lateral shearing force should and then applied as a dressing.
be applied to avoid a gritty product. The powder/fatty
base mixture may then either be returned to the GELS
evaporating basin with the remaining cream or stirred until Pharmaceutical gels are often simple phase, transparent
cold or the remaining cream in the evaporating basin may semi-solid systems that are being increasingly used as
pharmaceutical topical formulations.
DISPENSING, INCOMPATIBILITIES, PATIENT COUNSELLING (MODULE 3) CABIGAS, LUGO, BARROZO | 4B-PH 26
The liquid phase of the gel may be retained within a SUPPOSITORIES AS DOSAGE FORMS
three-dimensional polymer matrix.
Drugs can be suspended in the matrix or dissolved in the ADVANTAGES
liquid phase. Can exert local effect on rectal mucosa.
Used to promote evacuation of bowel.
ADVANTAGES OF GELS Avoid any gastrointestinal irritation.
Stable over long periods of time Can be used in unconscious patients (e.g. during fitting).
Good appearance Can be used for systemic absorption of drugs and avoid
Suitable vehicles for applying medicaments to skin and first-pass metabolism.
mucous membranes giving high rates of release of the
medicament and rapid absorption. DISADVANTAGES
Gels are usually translucent or transparent and have a
number of uses: May be unacceptable to certain patients.
Anesthetic gels May be difficult to self-administer by arthritic or
physically compromised patients.
Coal tar gels for use in treatment of psoriasis or eczema
Unpredictable and variable absorption in vivo.
Lubricant gels
Spermicidal gels.
POWDERS AND CAPSULES
GENERAL METHOD POWDERS
FUSION
Intimate mixtures of dry finely divided drugs and or
This involves melting together the bases over a water bath chemicals intended for internal or external administration
before incorporating any other ingredients. Problems in compounding powders
The ointment base may include a mixture of waxes, fats, o Formation of aggregates
and oils, of which some are solid at room temperature and o Stratification formation of two layers due to
others are liquid. difference particle size or densities of the powders.
Hard Beeswax, Cetostearyl alcohol, Paraffin
Soft Wool fat, Yellow and white soft paraffin METHODS OF COMMINUTION
Liquid Liquid paraffin, Vegetable oils
Trituration
SUPPOSITORIES Pulverization
Solid unit dosage forms suitably shaped for insertion into Grinding
the rectum. Levigation
The bases used either melt when warmed to body For vegetable or animal origin
temperature or dissolve or disperse when in contact with o Beating
mucous secretions. o Contusion
Suppositories may contain medicaments, dissolved, or o Grating
dispersed in the base, which are intended to exert a o Slicing
systemic effect. o Chopping
Alternatively the medicaments or the base itself may be
2 KINDS OF POWDERS
intended to exert a local action.
Suppositories are prepared extemporaneously by Bulk powders
incorporating the medicaments into the base and the Divided powders
molten mass is then poured at a suitable temperature into o How to measure powder
moulds and allowed to cool until set. Block and Divide
Weighing- most accurate
PESSARIES Powder measures
A type of suppository intended for vaginal use. o Use parchment paper/ glassine paper
The larger size moulds are usually used in the preparation o Characteristics of Powder Paper
of pessaries such as 4 g and 8 g moulds. Folds readily
Used almost exclusively for local medication, the exception No springing back
being prostaglandin pessaries that do exert a systemic Impermeable to atmospheric condition
effect. Water repellant
Common ingredients for inclusion in pessaries for local Pharmaceutically elegant
action include: Remains clean during handling
Antiseptics
PROBLEMS WITH POWDERS
Contraceptive agents
Local anesthetics Hygroscopic absorbs moisture
Various therapeutic agents to treat trichomonal, bacterial Deliquescent absorbs moisture then liquefy
and monilial infections. Efflorescent crystalline substances, which when liberated,
the water of crystallization or hydration are converted to
powder-like substances.
DISPENSING, INCOMPATIBILITIES, PATIENT COUNSELLING (MODULE 3) CABIGAS, LUGO, BARROZO | 4B-PH 27
DISADVANTAGES
May block pores causing irritation
Possibility of contamination
Light fluffy powders may be inhaled by infants leading to
breathing difficulties
Not suitable for application to broken skin
ADVANTAGES
May be more stable than liquid equivalent
Administered with relative ease
Absorption quicker than capsules or tablets
DISADVANTAGES
Variable dose accuracy
Bulky and inconvenient to carry
Difficult to mask unpleasant tastes
ADVANTAGES
More stable than liquid dosage forms
Accurate dosing
DISPENSING, INCOMPATIBILITIES, PATIENT COUNSELLING (MODULE 3) CABIGAS, LUGO, BARROZO | 4B-PH 28
THE PHILIPPINE NATIONAL DRUG POLICY o It is widely acknowledged that the government is the
single largest purchaser of drugs in the country,
allocating major part of its health budget for drugs
PHILIPPINE NATIONAL DRUG POLICY (PNDP) and medicines.
The government's response to the problem of inadequate
provision of good quality essential drugs to the people. 5. The fifth pillar is on PEOPLE EMPOWERMENT.
Part of the problem is the high-cost drugs, which renders o This cuts across all the four pillars and aims to assist
them inaccessible to the majority of the population. people in exercising an informed choice in the
Stands on five pillars designed to eventually bring about purchase of cost-effective medicines.
the availability and affordability of safe, effective, and
good quality drugs for all sectors of the country, MEDICINES IN PNDF WITH ABUSE POTENTIAL (LIST A)
especially for the poor who need them most, but who can
least afford them. DANGEROUS DRUG PREPARATIONS (A1)
(drugs requiring S-2 License and DDB Prescription Form)
FIVE PILLARS OF PNDP
Form an integral unit, mutually complementary and Total = 19
supportive of each other. 1. Alprazolam
2. Bromazepam
1. The assurance of the safety, efficacy, and usefulness of 3. Butorphanol (as tartrate)
pharmaceutical products through QUALITY CONTROL. 4. Clonazepam
o Involve the regulation of the importation, manufacture, 5. Codeine (as phosphate)
marketing, and consumer utilization of all drugs and
6. Diazepam
their intermediates.
7. Fentanyl (as citrate)
2. The promotion of the RATIONAL USE OF DRUGS by 8. Flurazepam
both health professionals and the general public. 9. Ketamine
o Rational use of drugs refer to a carefully considered 10. Lorazepam
pattern of behavior on the part of the prescriber and 11. Methylphenidate
the consumer. 12. Midazolam
o Limit the use of medicines to situations where there are 13. Morphine (as sulfate)
clear valid indications for them. 14. Nalbuphine
o Only the most necessary and scientifically proven 15. Oxycodone
efficacious drugs should be used. 16. Phenobarbital
o Key Strategy: 17. Pethidine (meperidine)
Development and implementation of a Philippine 18. Thiopental sodium
National Drug Formulary (PNDF) which shall list 19. Zolpidem
those drugs which are most essential for the
diseases and conditions encountered in the
Philippines, and describe the appropriate use of CONTROLLED CHEMICALS (A2)
these essential drugs. (drugs requiring S-2 License using Ordinary Prescription Form)
Rules and regulations governing the promotion
and advertising of pharmaceutical products shall Total = 3
be reviewed and amended in order to contribute 1. Ephedrine (as sulfate)
towards the promotion of rational use of drugs. 2. Ergotamine (as tartrate)
o With these twin moves, consumers will now be 3. Methylergometrine (methylergonovine)
properly guided as to which drugs to use for their
particular needs and conditions. LIST B MEDICINES
Include immediate release, solid oral dosage forms of
3. The third pillar is the development of SELF-RELIANCE in multisource (generic) pharmaceutical products that require in-
the local pharmaceutical industry. vivo bioequivalence studies as proposed by the World Health
o Seeks to strengthen Filipino capabilities for the Organization (WHO):
manufacture of basic and intermediate ingredients for
drugs and medicines. ACTIVE PHARMACEUTICAL INGREDIENTS (APIs) ARE
o By developing a capability to produce strategic CLASSIFIED ACCORDING TO THE BIOPHARMACEUTICS
essential drugs locally, the country's dependence on Classification System (BCS) as follows:
imported drugs can be greatly reduced.
o Enable local drug manufacturers to be competitive BCS Class I: "high" solubility - "high" permeability
with the transnational drug firms. BCS Class II: "low" solubility - "high" permeability
BCS Class III: "high" solubility - "low" permeability
4. The fourth pillar relates to the TAILORED OR TARGETED BCS Class IV: "low" solubility - "low" permeability
PROCUREMENT OF DRUGS by government
o with the objective of making available to its own
clientele, particularly the lower-income sectors of the
society, the best drugs at the lowest possible cost.
DISPENSING, INCOMPATIBILITIES, PATIENT COUNSELLING (MODULE 3) CABIGAS, LUGO, BARROZO | 4B-PH 29
8. Cefixime 400 mg
9. Ciclosporin 25 mg
10. Clofazimine 100 mg
11. Dapsone 100 mg
12. Diloxanide furoate 500 mg
13. Efavirenz 200 mg
14. Erythromycin stearate and ethylsuccinate 250 mg
15. Etoposide 100 mg
16. Furosemide 40 mg
17. Glibenclamide 5 mg
18. Griseofulvin 250 mg
19. Haloperidol 2 mg
20. Indinavir sulfate 400 mg
21. Ivermectin 6 mg
22. Lopinavir 133.3 mg + ritonavir 33.3 mg
23. Mefloquine hydrochloride 250 mg
Depending on the classification, the oral bioavailability of 24. Mercaptopurine 50 mg
the API may be expected to range from being heavily 25. Nelfinavir mesylate 250 mg
dependent on the formulation and manufacturing method 26. Nevirapine 200 mg
(e.g. Class II APIs: poorly soluble yet highly permeable), to 27. Nifedipine 10 mg
being mostly dependent on the APIs permeability 28. Nitrofurantoin 100 mg
properties (e.g. Class III APIs: highly soluble yet poorly 29. Phenytoin sodium
permeable). 30. Praziquantel 600 mg
Pharmaceutical formulations that can be eligible for a 31. Pyrimethamine 25 mg
biowaiver procedure NOT requiring in vivo bioequivalent 32. Retinol palmitate 110 mg (200,000 IU)
studies Show the following characteristics: 33. Rifampicin 300 mg and rifampicin FDC with other anti-TB
1. Should contain a Class I API medicine
2. Should be rapidly dissolving (should release at least 34. Ritonavir 100 mg
85% of its content in 30 minutes in media with ph 1.2, 35. Saquinavir 200 mg
ph 4.5 and ph 6.8 at 37° Celsius) 36. Spironolactone 25 mg
3. Should not contain excipients which could influence the
37. Sulfadoxine 500 mg + pyrimethamine 25 mg
absorption of the API.
4. Should not contain API with narrow therapeutic index. 38. Sulfamethoxazole + trimethoprim
5. Should not be designed to be absorbed from the oral o 400 mg + 80 mg
cavity. o 800 mg + 160 mg
High permeability ensures complete uptake of 85% or 39. Theophylline anhydrous
more of the API during its passage in the small intestines. 40. Verapamil hydrochloride 80 mg
The decision to allow a biowaiver based on the BCS
should take into consideration the:
o solubility and permeability characteristics
o therapeutic use and therapeutic index of the API
o pharmacokinetic properties of the API
o similarity of the dissolution profiles of the multisource
and comparator products in standard buffers with a
pH of 1.2, pH 4.5 and pH 6.8 at 37° Celsius
o Data related to the excipients composition in the
multisource product are also required.
The WHO noted that in some countries, products may be
available at doses exceeding the highest dose on the
WHO Essential Medicines List (EML). In such cases, the
WHO tables on biowaivers may no longer be appropriate
and the dose solubility ratio and permeability will have to
be reassessed at the product dose.
o * Per DDB Regulation No. 3 s. 2005 to be prescribed thru Ordinary Rx (Personalized Rx) with S2, I DDP per Rx. Partial
Filling allowed. No Refill.
KETAMINE (Vial : Ketamax, Ketazol, Ketram, Uniket)
o * Per DDB Regulation No. 4 s. 2005 preparations not in injectable form i.e. capsule, tablet, or syrup, to be prescribed thru
Ordinary Rx (Personalized Rx) with 52. 1 DDP per Rx, Partial Filling allowed, No Refill.
PSEUDOEPHEDRINE HYDROCHLORIDE (Rhinos SR tablet)
PSEUDOEPHEDRINE SULFATE (Clarinase tablet; Clarinase tyrup)
o * Per DDB Resolution No. 8 s. 2004 preparations not in injectable form i.e. capsule, tablet, or syrup, to be prescribed thru
Ordinary Rx (Personalized RX) with S2, 1 DDP per Rx, Partial Filling allowed, No Refill.
ALPRAZOLAM (Tablet : Alprazolam, Altrox, Atrest, Praz, Xanor, Xanor XR)
BROMAZEPAM (Lexotan tablet)
CLONAZEPAM (Tablet : Clonotil, Rivotril)
CLORAZEPATE DIPOTASSIUM (Tranxene capsule)
DIAZEPAM (Tablet : Diazepam, Nixtensyn, Solina, Valium)
ESTAZOLAM (Esilgan tablet)
FLURAZEPAM (Dalmane capsulet)
MAZINDOL (Mazzol tablet)
MIDAZOLAM (Dormicum tablet)
NITRAZEPAM (Mozepam tablet)
PHENOBARBITAL SODIUM (Phenobarbital tablet)
PHENTERMINE RESIN (Duromine capsule)
ZOLPIDEM (Tablet : Niben, Pidezol, Stilnox, Stilnox MR, Ziohex, Zoldem, Zulnap)
o Per DDB Regulation No. 3 s. 2003 to be prescribed thru Ordinary Rx (Personalized Rx) with S2, Partial Filling allowed, No
Refill.
ERGOTAMINE TARTRATE (Avamigran Tablet)
METHYLERGOMETRINE MALEATE (Ampul : Cethergo, Lerin, Methylergometrine maleate, Medisyl, Mertgotrex,
Methergin, Myometril, Uterine, Utermet ; Usamena tablet)
DISPENSING, INCOMPATIBILITIES, PATIENT COUNSELLING (MODULE 3) CABIGAS, LUGO, BARROZO | 4B-PH 31
PRESCRIPTION LIMITS
Section 32 (6), DDB Regulation No. 3 s. 2003 The quantities that may be prescribed in a single applicable prescription by a
licensed practitioner should not exceed the specified quantities as follows:
Ordinary circumstances:
- 30 tablets or capsules
10 ampuls x 1 mL
03 ampuls x 2 mL
i. Benzodiazepines as anxiolytic or hypnotic or both 02 ampuls x 3 mL
02 ampuls x 5 mL
01 ampul x 10 mL
for muscle spasm/dystonia/tetanus 90 tablets (5 mg)
- 2 weeks supply
ii. Phenobarbital preparations
for epilepsy patients 2 bottles x 100 tablets
iii. Pethidine Hydrochloride - 03 ampuls
iv. Other Dangerous Drugs (hospital use) - 01 vial
A prescription may not be issued in order for an individual practitioner to obtain controlled substances for the purpose of general
dispensing to patients.
A prescription may not be issued to a drug dependent person for the purpose of continuing his dependence upon such drugs.
DISPENSING, INCOMPATIBILITIES, PATIENT COUNSELLING (MODULE 3) CABIGAS, LUGO, BARROZO | 4B-PH 32
REVIEW QUESTIONS Technique: Look at the question well but don t spend too much time on it. Look for
keywords. In here, the keyword is .
1. The method of blending which is particularly useful for solid
substances that liquefy or form eutectic mixture when in close or 7. This auxiliary label should be placed on the label when
prolonged contact with one another. dispensing suspension, emulsion, and lotion
A. Spatulation C. Trituration
B. Sifting D. Levigation
Spatulation cannot be used with drugs with potent substances because it 8. If an aromatic water is part of a Rx, it should be
does not offer intimate and uniform mixing.
A. Added at last when the product is already in the bottle
Geometric dilution is the method of blending for potent substances, wherein
a small amount of potent drug is mixed with a large quantity of diluent. and stopper quickly
o Add an equal amount of diluent to the API, combine, then add another B. If heat is used in compounding, allow the solution to cool
equal amount of diluent to the resulting mixture until all diluent is before dropping the volatile ingredient
consumed (1 mg API + 1 mg diluent = 2 mg + 2 mg diluent = 4 mg).
Trituration is used for intimate mixing. (Similar to geometric dilution) C. Both A & B
Sifting is a method of mixing that results to a light, fluffy product. D. None of these
Levigation is the method of blending for ointments.
Tumbling is used when the substances are enclosed in a rotating container. This is to avoid the evaporation of the volatile oils in the aromatic water.
Waxed paper waterproof and transparent so it can be used with 12. The most convenient and safest way for extemporaneous
hygroscopic and highly deliquescent drugs. To prevent atmospheric effect, compounding is in the preparation of
drugs wrapped in waxed paper may be double wrapped with bond paper. A. TPN C. Dermatological
Bond paper not for volatile substances because it is not moisture resistant.
B. Ophthalmic product D. Anti-hypertensive
Glassine glazed but with limited moisture-resistant properties.
Vegetable parchment thin, semi-opaque papers with limited moisture-
resistant properties. TPN and ophthalmic products should be compounded in a sterile environment,
while antihypertensive drugs require several steps. On the other hand, galenical
and dermatological products can be usually compounded using mortar and pestle.
6. A common Rx chemical which liquefy when mixed include:
A. Camphor C. Chloral hydrate
B. Chloral hydrate and camphor D. Phenol
DISPENSING, INCOMPATIBILITIES, PATIENT COUNSELLING (MODULE 3) CABIGAS, LUGO, BARROZO | 4B-PH 33
13. Tetracycline are not recommended to be taken 20. In compounding Rx order, the portion added to dilute
A. active constituents to a reasonable dose size is
B. With antacid A. Basis C. Corrective
C. By Children B. Vehicle D. Adjuvant
D. All of these
21. Which of the following is a plasma volume expander?
Tetracycline can cause photosensitivity and can complex with metals present in A. Dextran 40 C. Dextran 60
antacids and children (rich in calcium; drinking milk).
B. Dextran 80 D. Dextran 70
When dealing with multiple choices, you can cancel out what you think is wrong,
especially in the Morse type of questions/ look for keywords on what makes the Plasma expanders (aka plasma extenders) are nontoxic substances, not of human
statement wrong. origin, that are being used to restore blood volume in case of emergency blood
loss (to replace the amount/volume of blood lost) as rapidly as possible to
prevent hypovolemic shock. They are inferior to plasma but do not need
14. All of the following are useful in the treatment of cough refrigeration and are not required to be freshly prepared. They can be stored in
except; emergency vehicles like ambulance/fire trucks, unlike plasma.
A. Noscapine C. Syrup of ipecac (commonly induces nausea) Plasma is still preferred but plasma expanders are given in emergency cases.
B. Potassium iodide D. Iodoquinol (amebicide)
22. How many mL of 0.9% W/V NaCl solution can be
15. The lowering of melting point is called prepared from 250 mL of 25% W/V solution?
A. Evolution C. Hydrolysis A. 3750 C. 6944.44
B. Eutexia D. None of these B. 2500 D. 9
18. A signal to the pharmacist that the patient is overusing a 25. Type of container used in dispensing viscous liquids
certain product A. Collapsible tubes C. Wide mouth bottle (also for bulk powder)
A. Discharged patient C. Health patient B. Standard Rx bottle D. Dropper bottle
B. Not purchasing (missed dose) D. Frequent refills
26. How is 50% glucose solution administered when the Rx
19. The role of the pharmacist includes all except: calls for Glucose 25 g PO TID today
A. Recommended OTC drugs A. Administer 50 mL of 50% glucose solution orally 3x today
B. Advice patients on expected drug effect B. Administer 50 mL of 50% glucose solution orally 3x a day
C. Diagnose and prescribed drugs (role of doctor) C. Administer 100 mL of 50% glucose solution orally 3x a
D. All of these day
D. Administer 50 mL of 50% glucose solution IV
DISPENSING, INCOMPATIBILITIES, PATIENT COUNSELLING (MODULE 3) CABIGAS, LUGO, BARROZO | 4B-PH 34
28. Prescription bottle are used for dispensing 33. A medication order calls for 1L of D5W to be
A. Bulk powder C. Capsules and tablets administered over an 8-hour period. Using IV administration set
B. Liquids of low viscosity D. Viscous liquids which delivers 10 drops per mL, how many drops per minute
should be delivered to the patient?
CONTAINERS A. 25 C. 21
Round Vials solid dosage forms (capsules, tablets) B. 15 D. 20
Prescription Bottles liquids of low viscosity
bulk powders, large quantities of
Wide-mouth bottles
tablets, capsules, viscous liquids.
ophthalmic, nasal, otic, oral liquids to
Dropper bottles 34. The adult dose of a drug is 250 mg. What would be the
be administered by drop.
applying liquid medication to wound approximate dose for an eight-year-old child? Use
Applicator bottle
or skin surface rule.
Ointment jars and A. 100 mg C. 200 mg
ointments, creams
collapsible tubes B. 150 mg D. 175 mg
Sifter-top containers powders applied by sprinkling
Hinged-lid/slide
suppositories, powder packets
boxes
Aerosol containers pharmaceutical aerosol products
Dusting powder locally applied nontoxic preparations that are intended to have
no systemic action (applied externally)
Oral powder supplied as finely divided powders or effervescent granules (taken
orally)
Insufflations finely divided powders introduced into body cavities such as the
ears, nose, throat, tooth sockets and vagina. (inhaled)
Douche powder completely soluble and are intended to be dissolved in water
prior to use as antiseptics or cleansing agents for a body cavity
41. Process of breaking down the cell membrane such that 48. The form of sulfur that a pharmacist should use when
extracting fluid will be in contact with greater surface area extemporaneously compounding a lotion is
A. Comminution C. Trituration A. Flower of sulfur C. Precipitated sulfur (milk of sulfur)
B. Grinding D. Pulverization B. Sublime sulfur D. Washed sulfur (not commonly used)
53. The pediatric dose of cefadroxil is 30 mg/kg/day. If a 60. The composition of Oral Rehydration Solution are:
child is given a daily dose of 2 teaspoon of a suspension A. Amino acid, Glucose, Water
containing 125 mg of cefadroxil per 5 mL, what is the weight B. Glucose, Sodium, Potassium, Chloride, Water
in lb. of the child? C. Glucose, Sodium, Potassium, Bicarbonate, Water
A. 18.5 lbs. C. 18.3 lbs. D. Citric acid, Glucose, Calcium, Water
B. 18.8 lbs D. 18.1 lbs.
61. When multivitamins in solution are prescribed with
1 tsp : 5 mL = 2 tsp : 10 mL antibiotics like cefazolin an infusion fluid precipitation occurs.
This incompatibility can be corrected by:
A. Injecting the drug separately
B. Using
54. An order for 500 mL of a solution of potassium sulfate to C. Administering the drug separately after 30 mins
be made so that it contains 10 mEq of potassium sulfate. How D. Not administer the drug
many grams of potassium sulfate is required to prepare the
solution (MW= 174) Piggy back is an intermittent IV administration by which the solution from two
A. 0.495 C. 4.44 containers flow into a one patient vein to a common tube and a common injection
site.
B. 0.870 D. 0.440
62. A pharmacist can help in IV admixture program by
A. Check the IV container for particular matter of cloudiness
B.
C. Double checking any order for incompatibility and
stability problem
55. The amount of suspending agent to be added in a D. All of these
suspension depends on the:
A. Volume of the solvent used C. Volume of the Rx 63. The form of water most commonly used as a solvent during
B. The amount to be suspended D. None of these the preparation of parenteral is
A. Sterile Water for Injection USP (complete absence of pyrogen)
56. The best way to approach a physician to point out B. Bacteriostatic Water for Injection USP
medication or prescription error: C. Distilled Water (not advisable for parenterals)
A. Show competence by informing the physician your clinical D. Water for Injection USP (most commonly used)
pharmacy training
B. Bring along your reference books when approaching the Bacteriostatic water for Injection USP contains an antimicrobial agent. It is not
used for infants
physician
C. Develop friendly and professional relations with your
64. In the unit dose dispensing:
physicians
A. The pharmacist issues container of medication to nursing
D. Present information in the form of directive
unit where the nurse must prepare the medication prior to
administration
57. Aspirin will dissolve faster in
B. The patient has the opportunity to be in possession of his
A. Basic C. Neutral medium
medication and make him familiar with its attribute prior
B. Acidic D. All of these
to discharge
Aspirin will dissolve in solutions of alkali acetates an citrates.
C. The pharmacist prepares every dose of medication ready
Dissolves with decomposition in water and solutions of alkali hydroxide and to be administered by the nurse
carbonates D. None of these
Forms damp mass when triturated with other deliquescent or hygroscopic
substances like acetanilide, antipyrine, phenol, salol, phenyl salicylate,
aminopyrine 65. These are example of incompatibility, except:
Forms gummy consistency upon contact with atmospheric moisture or sodium A. Precipitation C. Hydrolysis
bicarbonate B. Enterotoxin D. Decomposition
58. Compound whose absorption may be reduced by food, 66. Which of the following dosage form does not contain
except: alcohol:
A. Tetracycline C. Isoniazid A. Alcohol C. Tincture
B. Cephalexin D. Metronidazole B. Enema D. Spirit
Benzoic acid is also right, but the best answer is retinoic acid or Vitamin A
DISPENSING, INCOMPATIBILITIES, PATIENT COUNSELLING (MODULE 3) CABIGAS, LUGO, BARROZO | 4B-PH 38
82. This will prevent stratification of powders and ensure 90. Determine the dose of a 6-year-old child weighing 50
uniform blending pounds when the usual adult dose of an opioid analgesic is 15
A. Tumbling C. Sifting
B. Sieving D. All of the above A. 5 mg C. 10 mg
B. 1.5 mg D. 3 mg
83. ______ result from hydrolysis, oxidation, reduction, or
complexation of parenteral dosage form which can be
detected only by suitable analytical method
A. Therapeutic parenteral incompatibility
B. Physical incompatibility
C. Chemical incompatibility
D. All of the above 91. A suspension is NOT a suitable dosage form for what type
of injection?
84. An anti-hypertensive drug lists at P 180.00 per dozen A. Intradermal C. Subcutaneous
tablet, less discount of 33 1/3% for a purchase of 100 dozen, B. Intramuscular D. Intravenous
plus an additional promotional discount of 10%. Calculate the
net cost per unit. IV suspension is painful and have risk of emboli, which may block the capillaries
A. P 9.00 C. P 9.25
B. P 9.15 D. P 9.50 92. Property of a colloidal solution
A. Particle pass through semipermeable membrane
B. High partition coefficient
C. Brownian motion
D. All of these
85. In weighing the ingredients to be used in compounding the
prescription, the pharmacist should: 93. U-100 insulin is
A. Weigh each ingredient and place back the container in A. Orange C. Red
the shelf B. White D. Brown
B. Place all ingredients on the left side of the balance and
Before, U- insulin is colorless
transfer each to the right side as it is weighed (avoid mix ups)
C. Weigh each ingredient without interruption 94. Relevant pharmaceutical service in hospital are
D. All of these A. Extemporaneous compounding
B. QC, IV admixture and drug information, education, and
86. When compounding an isotonic solution of silver nitrate, the training
pharmacist should avoid using tap water because it contains C. Quality control, IV admixture
_______ that will cause white precipitate D. Drug information, education, and training
A. Carbonates C. Fluoride
B. Chloride D. Sulfate 95. Filling of capsules include recording of the following,
except
87. Salting out is an example of _____ incompatibility A. Price of capsule C. Weighing of each capsule
A. Physical C. Therapeutic B. Dosage D. None of these
B. Chemical D. Decomposition
96. Aspirin with antacid will cause
88. A physician orders aminophylline suppositories, gr. Viis stat A. Alteration of pH C. Complexation and absorption
and non-rep. How much drug should each suppository contain? B. Alteration of GIT flora D. All of these
A. 7 gr C. 2 pcs sup
B. 7 ½ gr D. 1 pc supp 97. What element interfere with the activity of tetracycline?
A. Ca and Bi C. Mg and Ca
89. A prescription is Penicillin VK 300,000 U q 4h PO for a B. Mg and Na D. Na and K
child. How much is administered per dose of the drug if what is
available is Potassium Phenoxymethyl Penicillin 125 mg or 98. When 1 part of solute dissolves in 1-10 parts solvent, it is?
200,000 U per 5 mL? A. Very soluble C. Freely soluble
A. Administer 5 mL C. Do not dispense B. Soluble D. Sparingly soluble
B. Administer 7.5 mL D. Administer 10 mL
DISPENSING, INCOMPATIBILITIES, PATIENT COUNSELLING (MODULE 3) CABIGAS, LUGO, BARROZO | 4B-PH 39
Very soluble <1 Wide mouth jar can be considered, however, it is prone to contamination due to
Freely soluble 1 10 its higher surface area exposed.
Soluble 10 30
Sparingly Soluble 30 100 107. Use of paraben in syrup is as:
Slightly soluble 100 1,000
Very slightly soluble 1,000 10,000 A. Preservative C. Solubilizer
Practically insoluble >10,000 B. Thickener D. Buffer
99. Which of the following drug is an over-the-counter drug? 108. Normal saline solution contains ____ sodium chloride:
A. Dimeticone (composed of silicon dioxide) A. 0.1%
B. Warfarin (anticoagulant) B. 1.5%
C. Innovar (narcotic-analgesic) C. 0.9% (isotonic solution, physiological salt solution)
D. Mogadon (short-acting drug for insomnia; generic name is Nitrazepam) D. 1%
100. The usual storage condition specified for biologicals is 109. When a patient is taking corticosteroids, he should avoid:
A. Room temperature C. 2 to 8°C A. Aspirin and acetaminophen
B. Below 2°C D. A cool place B. Acetaminophen and ibuprofen
C. Amoxicillin
Freezing Below 2°C or -10 to -20°C D. Aspirin and ibuprofen
Cold 2 to 8°C
Cool 8 to 15°C
Room Temperature 15 to 30°C Corticosteroids stimulate peptic ulcer. Aspirin can induce gastric bleeding.
Ibuprofen has higher chance to induce gastric bleeding/upset than
Warm 20 to 40°C
acetaminophen, which has light tendencies.
Excessive heat or extremely hot > 40°C
101. Collodion is made flexible through addition of 110. This mechanical process of reducing the size of particles
A. Castor oil, gelatin, and camphor of a compound is called
B. Liquid pyroxylin and camphor A. Spatulation C. Levigation
C. Castor oil and camphor B. Comminution D. Sifting
D. Olive oil, resin, and camphor
Spatulation, comminution, levigation are all methods of mixing or blending.
116. Plastic parenteral bottles and bags differ from glass units
in that the plastic units have:
A. Two entry points C. vacuum
B. An air tube in the unit D. All of these
Weight range is 0.9 g to 1.1 g (used in assays, monograph, etc.), but the exact
is 1.0 g.
ADDITIONAL QUESTION
How much of cocoa butter are you going to use if the density
factor is 1.2 and the average weight of a suppository is 0.98
g?
Rx
gr ½
q.s.
PHENOBARBITAL
1 grain : 65 mg = 0.5 grain : 32.5 mg or 0.0325 g
TEN SUPPOSITORIES
0.0325 g x 10 supp = 0.325 g Phenobarbital
0.98 g x 10 supp = 9.8 g Total weight
119. The capacity of the syringe commonly known as tuberculin syringe is:
A. 1.0 mL C. 0.5 mL
B. 0.1 mL D. 2.0 mL
121. The process of wetting and smoothing zinc oxide with mineral oil in preparing ointment is
A. Trituration C. Spatulation
B. Levigation D. Pulverization
128. How many chloramphenicol capsule, each containing 250 mg of chloramphenicol are needed to
provide 25 mg/kg of body weight per day for 1 week for a person weighing 132 pounds?
A. 42 C. 6
B. 24 D. 10
130. If the loading dose of Kanamycin is 7 mg/kg of body weight, how many g should be administered to a
petient weighing 130 lbs.
A. 0.492 g C. 414 g
B. 0.414 g D. 0.485 g
131. How much sodium chloride is needed to adjust the following prescription to isotonicity? (E value of Zinc
sulfate = 0.15)
Rx
Zinc sulfate 1%
Sodium chloride q.s
Purified water q.s 60 mL
A. 0.09 g C. 45 g
B. 0.45 g D. 4.5 g
133. A patient is administered dextrose 5 % in water, 1 liter to run for 10 hours. How many drops is used per
minute?
A. 18.675 drops/minute C. 37.35 drops/minute
B. 49.8 drops/minute D. 24.9 drops/minute
134. The magnesium salt used in the preparation of magensium citrate solution is
A. Magnesium chloride C. Magnesium carbonate
B. Magnesium citrate D. None of these
135. A signal to the pharmacist that the patient is overusing a certain product
A. Discharged patient C. Health patient
B. Not purchasing D. Frequency of refills
138. The dose of drug for children based on body surface area is based on
A. Weight and age C. Age and height
B. Weight and height D. Width and height
139. Give amoxicillin caps 250 mg every 6 hour. Stock is Amoxil 500 mg/cap. Since there is no available stock
of 250 mg and the doctor acceded to the available stock, how many interval will you give the 500 mg
cap?
A. 8 hours C. 6 hours
B. 12 hours D. 10 hours
140. Investigational New Drug under Phase I clinical trial involves administering the drug:
A. To animals for toxicity studies
B. By select clinicians to patients suffering from disease
C. To animals to determine the effectiveness of the drug
D. By select clinicians to health volunteers
142. Most important factor that determines the success or failure of effervescent product during manufacture
A. Quantity and quality of ingredient C. Relative humidity
B. Substance that enhances dissolution D. All of these
143. When compounding Magnesium Citrate solution, which of the following substance is not included
A. Sodium bicarbonate C. Citric acid
B. Magnesium carbonate D. Magnesium citrate
146. Upon dispensing what vital information can the pharmacist give when a patient buys calcium
supplement
A. Calcium’s absorption is enhanced with Vit. D
B. Calcium’s absorption is enhanced with Vit. E
C. Calcium’s absorption is deterred with Vit. D
D. Calcium’s absorption is deterred with Vit. E
147. What would be the appropriate auxiliary label one can use in dispensing dispersion systems
A. For external use only C. Shake the bottle before using
B. Store in a cool dry place D. Both B and C
149. When dispersing Ferrous sulfate vitamin to a patient, which precaution should pharmacist give to the
patient
A. Avoid tea or coffee because it reduces iron absorption by as much as 50%
B. Avoid Vitamin A because it reduces iron absorption
C. Avoid tetracycline because it inactivates iron
D. Avoid aspirin because iron can cause elevated levels of aspirin in the blood
151. Which is the most appropriate cautionary and advisory label for Brufen?
A. Do not take anything containing aspirin while taking this medicine
B. This medicine may color the urine
C. Take with or after food
D. Do not take ingestion remedies at the same time of the day as this medicine
152. Pharmacist should counsel pregnant women not to take NSAIDs drugs during the entire period of
pregnancy because:
A. With regular use of closure of fetal ductus arteriosus in utero may occur
B. Risk of neutral tube defect may be increased
C. Placental perfusion may be reduced
D. Genital malformations cautionary and cardiac defect may be developed
153. Which is the most appropriate cautionary and advisory label for Nalidixic?
A. Avoid alcoholic drink
B. May cause drowsiness
C. Avoid exposure of skin to direct sunlight or sun lamps
D. Do not stop taking this medicine except on your doctor’s advice
154. Pharmacist should counsel pregnant women not to take Theophylline during the entire period of
pregnancy because
A. Possibility of premature separation of placenta in first 18 weeks may occur
B. Neonatal irritability and apnea may develop
C. Congenital anomalies may develop
D. Neonatal respiration may be depressed
155. In evaluating the appropriateness of the prescription and medication order, the pharmacist should
check the following, except:
A. The patient’s disease or condition requiring treatment
B. The prescribed route of administration
C. The biological action of the prescribed product
D. The auxiliary and cautionary labels
156. In selection of the proper package or container, the pharmacist choose the container that:
A. Ensure product stability C. Promote patient compliance
B. Comply with legal requirement D. All of the above
157. In order to ensure proper medication use, storage and compliance with applicable statutes, what
should be affixed on the label of the drug?
A. Expiration date of the medication C. Quantity of medication dispensed
B. Auxiliary and/or cautionary labels D. Product strength
158. It is part of patient profile which is necessary to assess the appropriateness of the dose
A. Birth date and weight C. Direction for use
B. Clinical condition D. Medication strength
159. In performing therapeutic intervention, the following information should be communicated to the
prescriber, except:
A. Brief description of the problem
B. Reference source that document the problem
C. Description of the clinical significance of the problem
D. Possible physical and chemical incompatibility
160. Detection of drug-related problem requires an assessment of the need for a change in drug therapy.
Possible problems include the following, except:
A. Inappropriate compliance C. Wrong drug
B. Dose too low D. Undiagnosed condition
161. In dispensing antifungal medication for skin, the patient should be instructed to
A. Exposure to air whenever possible
B. Use caution with Povidone – Iodine in anyone with allergies
C. Keep area covered at all time with cream and sterile dressing
D. Avoid combination of cholinergic medications
162. In dispensing Accutane for acne, which is the most appropriate instruction that should be given to the
patient?
A. Avoid multivitamins that contain Vitamin A
B. Keep area covered at all times with cream and sterile dressing
C. Avoid prolonged use
D. Discontinue and seek medical aid if irritation occurs
163. What is the most appropriate patient education for those undergoing ulcer therapies
A. Avoid cigarette smoking, this seems to decrease the effectiveness of medicine in the healing of
duodenal ulcers
B. Adequate fluid intake and bland diet
C. Avoid self-medication for longer than 48 hours
D. Regular exercise to develop muscle tone
166. The term applied to a reaction of organic compound with water resulting in fragmentation into its
component molecule is
A. Ionic hydrolysis C. Simple hydrolysis
B. Molecular hydrolysis D. None of the above
167. It is classified as one the human errors in dispensing, which are always physical in nature are defined as
unintended actions rather than as error in judgement
A. Slips C. Mistakes
B. Lapses D. None of the above
168. It is classified as an error in dispensing which usually either due to inattention or over attention
A. Knowledge based error C. Medication based error
B. Rule based error D. Skill based error
170. It is classified as one of the human errors in dispensing, which largely refers to failures in memory
A. Slip C. Mistakes
B. Lapses D. Rule based error
171. Before dispensing Theophylline prescription, Pharmacist must advise the patient to
A. Avoid acid foods like orange juice and tomato
B. Take foods rich in lipid
C. Take on an empty stomach with a glass of water
D. Take with food but avoid foods rich in protein and cabbages
172. It is classified as an error in dispensing which are usually due to either the misapplication of good
rules/procedure, or application of bad rules/procedure
A. Knowledge-based error C. Rule-based error
B. Medication-based error D. Skill-based error
173. It is and error in judgement and in most situation, it is made as either a failure or expertise or lack of
experience
A. Slip C. Mistakes
B. Lapses D. Skill based error
174. It is the most complex error type which is made within the realm of problem solving and judgement
A. Knowledge-based error C. Rule-based error
B. Medication-based error D. Skill-based error
175. The following drugs should be taken on an empty and with a glass full of water, except:
A. Tetracycline C. Rifampicin
B. Metronidazole D. Lincomycin
176. Situation(s) in which pharmacists are more likely to make mistakes in dispensing
A. Excessive reliance on memory and C. A only
lack of standardization D. Both A & B
B. Inadequate availability of information
177. The benefit of electronic prescribing and dispensing are the following, except
A. Rapid information exchange
B. Better clarity and communication of prescription information
C. Amplify fraud risks present in the paper-based process
D. Improved confidentiality and security of health information
183. In order for the Pharmacist to adhere to the standard dispensing procedure, he/she must not
A. Verify doubtful prescription from the patient
B. Check dose limits
C. Checked potential interaction between any medicine currently taken and the drug on the
prescription
D. Advise the patient on the proper use of the medications
184. In the dispensing of outpatient medication, which of the following must the pharmacist do
A. Review all appropriate information regarding prescription refills
B. Be personally present for supervision
C. None of the choices
D. All of the choices
185. According to the FDA, pharmacist can help in fixing the problem of drug name confusion through the
following means, except
A. Verifying with the doctors information that is not clear before filling a prescription
B. Keeping look-a-like, sound-a-like products separated from one another on pharmacy shelves
C. Avoiding stocking multiple product sizes together
D. Separate the product of the same packaging
188. It is defined as any preventable event occurring in the medication use process, including prescribing,
transcribing, dispensing, using and monitoring, which results in inappropriate medication use or patient
harm
A. Prescribing error C. Dispensing error
B. Medication error D. Rule-based error
189. A physical incompatibility wherein 2 or more liquids fail to dissolve or mix with one another
A. Insolubility C. Precipitation
B. Immiscibility D. Liquefaction
190. Forms of liquefaction where there is lowering of melting point of the substance lower than their individual
melting point
A. Eutexia C. Deliquescence
B. Hygroscopy D. All of the above
192. Bulk forming laxative are the choice for older adults or laxative dependent patient. Patients who are
using this laxative should be strictly instructed by the pharmacist to
A. Avoid its use longer than one week without medical supervision
B. Discontinue if any signs of diarrhea or abdominal pain occurs
C. Dissolve this laxative in one full glass of water and followed by another glass of fluid to prevent
obstruction
D. Avoid exposure to sunlight
193. The following are instructions necessary to patient who are taking decongestants, except
A. To be used only for few days to avoid rebound congestion
B. To be avoided when cardiac and thyroid conditions are present
C. To discontinue when its side effects sets in, such as nervousness, tremor, headache, etc
D. To avoid all estrogen and progestin product
195. A type of incompatibility which occurs when norepinephrine is added to sodium carbonate
A. Physical incompatibility C. Therapeutic incompatibility
B. Chemical incompatibility D. All of the above
197. Which of the following drugs must be protected from light during administration to maintain their stability
A. Na nitroprusside & Amphotericin B C. Penicillin & Chloramphenicol
B. Gentamicin & Heparin D. Erythromycin & Penicillin
198. A patient carrying prescription of Ampicillin, which of the following advises can the Pharmacist given to
the patients upon dispensing?
A. Take with food
B. Do not take with food
C. Avoid foods rich in proteins
D. Take on an empty stomach with a glass water and avoid acid foods
199. Factors commonly associated with chemical incompatibility for IV admixture, except
A. Drug concentration C. Color change
B. pH of the solution D. Temperature and light
201. The following factors can affect the compatibility of an IV drug or solution, except
A. Light C. Contact time
B. Pressure D. Cloudiness
202. Incompatibility which occurs when heparin solution is mixed with and intermittent Aminoglycoside
infusion is manifested with the following, except
A. Precipitation C. Gas bubbles
B. Color change D. Cloudiness
203. An incompatibility observed with Amiodarone HCl 12.5 mg/mL is mixed with Clavulanic acid 10mg/mL
A. Immediate lactescence (milk appearance)
B. Immediate yellow coloring
C. Initial heavy turbidity becoming red-orange liquid with red precipitate
D. Immediate opalescence
204. An incompatibility observed when Rifampicin 6 mg/mL is mixed with Tramadol 8.33 mg/mL
A. Immediate lactescence (milk appearance)
B. Immediate yellow coloring
C. Initial heavy turbidity becoming red-orange liquid with red precipitate
D. Immediate opalescence
205. An incompatibility observed when Pantoprazole 8 mg/mL is mixed Midazolam 0.1 mg/mL
A. Immediate lactescence (milk appearance)
B. Immediate yellow coloring
C. Initial heavy turbidity becoming red-orange liquid with red precipitate
D. Immediate opalescence
206. Patients who are going to take antihistamine must be advised by the Pharmacist to avoid its use for a
longer period of time because
A. It can increase bronchial or nasal congestion and dry cough
B. It can cause nervousness, tremor, dizziness and confusion
C. It can increase blood pressure
D. It can increase blood sugar
208. Patients who are using oral contraceptives should be informed by the Pharmacist that its frequent use
can result in a deficiency of which B vitamin
A. Vitamin B5 C. Vitamin B6
B. Vitamin B1 D. Vitamin B12
211. When the pharmacist is faced with a prescription they are not familiar with, they may read it as
something they are familiar with and this is called
A. Drug confusion C. Lapses
B. Confirmation bias D. Wrong dispensing
216. The most appropriate cautionary and advisory label for Diclofenac sodium
A. Take with or after food
B. Take an hour before food or on an empty stomach
C. Dissolved under the tongue
D. This medicine may color the urine
218. Before dispensing ferrous sulfate vitamin to patients, the Pharmacist must counsel the patient with the
following advises, except;
A. Do not take together with milk
B. Avoid foods rich in protein
C. Take with food if it causes gastric disturbance
D. Swallow the drug without chewing
219. Acid produces ulcer and patients with ulcers should not be given with ____ drug because it is acid-
producing
A. Antihistamine C. Diuretic
B. Antibacterial D. Antidepressant
220. General factors to be considered in judging the safety of a prescribed dose, except:
A. Physiological state of the patient C. Frequency of administration
B. Pathologic condition of the patient D. Site of absorption of the drug
221. When the combined effects of 2 drugs are equal to the sum of their individual effect, it is known as
A. Potentiation C. Summation
B. Synergism D. Additive
222. If the combined effects are equal to those expected for drugs acting by the same mechanism
A. Potentiation C. Summation
B. Synergism D. Additive
224. Kwell, used to treat lice infestation, is contraindicated in all of the following condition, except
A. Infancy C. Pregnancy
B. Obesity D. Older adult
225. The following precautions must be instructed to the patient with the use of antacid, except
A. Blurred vision C. Electrolyte imbalance
B. Constipation D. Osteoporosis
226. Mrs. Bella comes into the pharmacy to find drug use in the Philippines. Pharmacist may suggest all of the
following reference material, except:
A. Martindale Extra Pharmacopeia C. USP adopted name
B. Index Nominum D. All of the choices
229. One advise the pharmacist may explain upon dispensing Ferrous Sulfate supplement is that it may
discolor the stool
A. Black C. Red
B. Orange D. Cream
234. If a pharmacist compounds an emulsion following the 4:2:1 ratio, how many mL of water should be
added if the oil amounts to 30 mL
A. 10 mL C. 5 mL
B. 15 mL D. 7.5 mL
237. When the generic name is preceded by the brand name, the prescription is
A. Erroneous prescription C. Impossible prescription
B. Violative prescription D. Correct prescription
242. The prescription of an organic substance from a saturated solution when a highly soluble salt is added
is known as
A. Polymorphism C. Eutexia
B. Salting out D. Salting in
245. The prescription requires the combination of sodium bicarbonate, sodium salicylate and water. The
pharmacist fully understands, that sodium carbonate and sodium salicylate will lead to the oxidation of
the latter forming a brown quinoid derivative. This incompatibility can be remedied by
I. Adding color diluent
II. Adding antioxidant
III. Removal of water
A. I, II, III C. I, III
B. II, III D. I, II
246. Degradation of drugs or excipient molecules brought by light, either room light or sunlight is known as
A. Solvolysis C. Photolysis
B. Oxidation D. Deamination
248. A prescription implies the combination of Silver nitrate, water, made isotonic with sodium chloride. If the
incompatibility is not recognized it will lead to:
A. Change of color of the solution C. Formation of white precipitate
B. Evolution of carbon dioxide gas D. Formation of gel like substance
A. I, III C. I only
B. II, III D. I, II, III
253. Cough and cold remedies with decongestant are not recommended to patient s experiencing
A. Low RBC C. High blood pressure
B. GI ulcer D. Airway obstruction
255. This refers to a type of ADR that is usually dose dependent and predictable
A. Type A C. Type C
B. Type B D. Type D
257. The adverse effect that may be experienced when taking cough and cold remedies containing
Phenylpropanolamine
A. GI disturbance C. Increase in BP
B. Serum sickness D. Decrease in BP
258. Mrs. Romano experience skin rash, that re characteristically large, deep red in color which becomes
darker at the center. The said rashes appeared after taking Amoxicillin for 3 days. The ADR is suspected
to be
A. Photosensitivity reaction C. Erythema multiforme
B. Topical epidermal necrolysis D. Serum sickness
259. In a drug interaction, the drug whose action is affected by the combination with other drugs, food or
chemicals is called
A. Object drug C. Interactant drug
B. Precipitant drug D. Action drug
262. When two (2) interaction drugs results to alteration of ADME, it is classified as
A. Pharmacokinetic interaction C. Pharmaceutical interaction
B. Pharmacodynamic interaction D. Beneficial interaction
264. Smoking increased the activity of the drug metabolizing enzyme resulting in to
I. Rapid metabolism of drug
II. Decrease effect of drug
III. Increase effect of drug
A. I, II C. I only
B. I, III D. III only
266. Decrease in body clearance of a usual dose benzodiazepine causes extension sedation. This ADR is
classified as
A. Type A D. Type D
B. Type B E. Type E
C. Type C
267. ADRs may also be classified based on their onset of reaction. Which of the following classifications
appears two or more days after drug exposure
A. Acute C. Latent
B. Subacute D. Mild
268. Severity of the condition is another basis for classifying ADRs. Which class requires active treatment of
adverse reaction, or further testing or evaluation for assessment
A. Mild C. Severe
B. Moderate D. Significant
279. Practical strategies to reduce errors resulting from sound alike medication and verbal orders include
I. Read back verbal orders
II. Prohibit the use of all verbal orders
III. Establish policies on who receive verbal order
A. I, II D. III only
B. II,III E. I, II, III
C. I, III
282. Phenytoin precipitates and fall to the bottom of the IV bag as an insoluble salt when it is added to the
solution of
A. D10W D. NSS
B. D5W E. KCl
C. D50W
284. Newborns that are exposed repeatedly to benzyl alcohol (an injectable product preservative) may lead
to a potentially fatal condition known as
A. Cushing syndrome C. Steven Johnson’s syndrome
B. Gasping syndrome D. Reye’s syndrome
287. Concurrent use of alcohol with sedative and other depressant drugs result to
I. Hyperexcitability II. Hallucination III. Coma
A. I, II, III C. II, III
B. I, II D. III only
288. Milk and other dairy products _____________ the absorption of tetracycline and fluoroquinolones
A. Increase C. Does not affect
B. Decrease D. All of the given choices
289. Grapefruit juice increases the blood levels of drug concurrently taken with it by
A. Increasing the activity of drug metabolizing enzyme
B. Inhibiting the cytochrome P450 enzyme
C. Increasing the absorption of drugs
D. Blocking the urinary excretion of drug
292. Based on USP guidelines the stability of extemporaneously compounded aqueous liquids (reconstituted
formulation) is referred to as the
A. A beyond use date of not later than 25 % of the time remaining until the products’ expiration date
or 6 months, whichever is earlier
B. Not later than 14 days when stored at cold temperatures
C. Not less than the intended duration of therapy or 30 days, whichever is earlier
D. Maximum beyond use date is 6 months or 25 % of the remaining time between the compounding
date and the shortest expiration date of the ingredient, whichever is earlier
293. Administrative/ Professional function of the pharmacist that include training and supervising interns and
externs during their required work experience is
A. Supervisory C. Managerial
B. Community service D. Receptor
294. Which of the following is not a consulting service provided by the pharmacist in the community setting
A. Screening for cholesterol
B. Staff education and in service
C. Regulation and compliance
D. Patient profile monitoring
300. This incompatibility involves the degradation of the drug excipient through reaction with the solvent
present in the formulation
A. Solvolysis C. Photolysis
B. Oxidation D. Polymerization
—-END—-
MODULE 3
PRACTICE OF PHARMACY
INSTRUCTION: Select the correct answer for each of the following questions. Mark only one answer for each item
by shading the box corresponding to the letter of your choice on the answer sheet provided. STRICTLY NO
ERASURES ALLOWED. Use pencil no. 2 only.
gives protection
to light
as well
2. The most convenient and safest way for extemporaneous compounding is the preparation of
O
-
A. TPN C. Dermatological
-
special case
° ①
B. Philippine Heart Center
any case
D. The Medical City General -
8. Nutritional
-
solution prepared by hospital pharmacist for patients who has dysfunctional GiT
A. IV mixture added to TPN
-
C. NSS 0.9% Nacl
-
B. D5W -
Nutrition
9. All of the following are types of glass containers, O
except
A. Ear and bottle dropper C. Bottle
-
①
B. Active Medical Staff D. Residential Medical Staff Receiving training -
TPN
13. Additive in TPN are ( component of
D
PRACTICE SET – MODULE 3 Prepared by: Grace Marie A. Maclan, RPh, CPS, MSc
Course Audit 2019 – 2020
UNIVERSITY OF SANTO TOMAS
FACULTY OF PHARMACY
DEPARTMENT OF PHARMACY
.
cost
of goods sold D. Associated with multiple stock
OB. Small volume purchasing Atebeginninghending location
C. Dead inventory inventory
20. Responsible provision of drug therapy for the purpose of achieving definite outcome that improve the
patient’s quality of life Definition by Hepler a strand
-
g
A. Pharmaceutical Care C. Traditional Hospital Pharmacy
B. Clinical Pharmacy D. Contemporary Pharmacy
S2
23. Which of the following is a Regulated Drug
regular Rx w/
-
A. Codeine Sulfate -
O
C. Amytal Sodium Capsukle
B. Brown Mixture Tablet D. Tussinex Syrup
* the
rest are prohibited yellow Rx -
24. Use of drug that causes allergic reaction is what type of ADR
A. Type A C. Type C continuous
Augmented ( Bose related $
- -
Cattergic
Rxn ) D. Type D -
Delayed (teratogenic)
Type E -
End
of
Use (withdrawal)
25. Minimum number of beds to be considered a hospital
F Failure -
A. 5 C. 7
①
B. 6
Eutectic
D. 8
of
26. Camphor, menthol, acetophenetidine and phenol will form
O
A. Eutectic mixture
Liguifaction upon
-
mixing
C. Precipitation
B. Explosive combination D. Cementation
27. The type of health service for critically and seriously ill patient when drug and supplies are immediately
-
-
available
o
A. Intensive care C. Intermediate care
B. Home care D. All of the above
29. Milk and other dairy product _____ the absorption of tetracycline
A. Increase C. Does not affect
B. Decrease Due to chelation D. It depends
①
PRACTICE SET – MODULE 3 Prepared by: Grace Marie A. Maclan, RPh, CPS, MSc
Course Audit 2019 – 2020
UNIVERSITY OF SANTO TOMAS
FACULTY OF PHARMACY
DEPARTMENT OF PHARMACY
30. How many cod liver oil is necessary to make 2500 capsule each containing 0.33mL
OA. 825 mL C. 7.57 mL
B. 7,575 mL
0.gg/g-mtx2500ca/p--
825mL
D. 757 mL
31. These workbench provides abn environment of specifically filtered air that sweeps the work area and
-
Dispensing
33. Rifampicin can cause _____ coloration on plastic contact lens
A. Brown C. Yellow
B. Red O
D. Orange
38. Standard dosing schedule of “q 6h” dictates that dose are given at
A. 10 AM and 10 PM O
C. 9AM, 3PM, 9PM, 3AM
B. 9AM, 1PM, 5PM D. 6AM, 12 noon, 6PM
42. Type of service which provides care to patients with any type of illness
A. Special hospital C. Health center
B. General hospital D. Puericulture center
O
PRACTICE SET – MODULE 3 Prepared by: Grace Marie A. Maclan, RPh, CPS, MSc
Course Audit 2019 – 2020
UNIVERSITY OF SANTO TOMAS
FACULTY OF PHARMACY
DEPARTMENT OF PHARMACY
43. Container impervious to aair and other gases under ordinary conditions of handling, storage and
transport
-
A. Air tight O
C. Hermentically sealed
B. Security closed D. Child resistant
C)
B. 200 Tsm D. 2
47. A patient is administered desxtrose 5% in water, 1L to run for 10 hours. How many drops is used per minutes
A. 18.675 drops / minute 11=20000 gits C. 33.33 drops / minute
B. 49.8 drops / minute 10 hrs 600 min -
-
D. 24.9 drops / minute
20.0009113/600 min 33.33 qttsl min
-
-
48. White ointment has a label claim of 5g/1000g, when expressed in w/w, this is equal to
A. 5 % C. 0.05 %
①B. 0.5 % goto x 100 0.54 =
D. 50 %
.
54. If 5mL of a 20% w/v aqueous solution of furosemide is diluted to 10mL, what will be the final strength of
furosemide
( 5mL)(201 ) Comte) (x )
.
=
A. 5 C. 15
Tomi Tomi
B. 10 D. 30
O
f- 101 .
56. If two pints of guiafenesin elixir is present in the inventory, how many 4 fluid ounces prescriptioncan be
filled 2 pts 32ft OZ
34=8
-
A. 12 C. 5
B. 10 O
D. 8
57. What is the dose of Drug A for a 38.5 lbs. child if the average adult dose of a drug is 125 mg
A. 3.21 mg Clark 's = 38.5 lbs C. 0.321 mg
① TEX 125mg
32.0µg
-
B. 32.1 mg D. 321.0 mg
PRACTICE SET – MODULE 3 Prepared by: Grace Marie A. Maclan, RPh, CPS, MSc
Course Audit 2019 – 2020
UNIVERSITY OF SANTO TOMAS
FACULTY OF PHARMACY
DEPARTMENT OF PHARMACY
59. If 40 kg of green soap was bought at PhP 2550, how much will 15kg cost
SIE : Hyponatremia
A. P 952.65 C. P 926.25
40kg : 2550 : : 15kg :X
B. P 925.56
X X -956.25
-
O
D. P 956.25
O
61. Loop diuretic used as a -
rapid – acting intranvenous agent in reversing acute pulmonary edema
-
63. Convert 40 C to F
A. 40 C. -140
:
B. 144 D. 104
65. If 250 unit of an antibiotic weighs 1mg, how many unit are in 25mg
A. 6,256 250 : 1mg :X -25mg
'
O
C. 6,250
B. 6,255 4=6250 D. 6,500
WW
66. If 2 tablets contains 550 mg of paracetamol, how many should contain 3000mg
A. 10 Ltab C. 8
10.909 Ill
B. 12
55T
=
Foo i
gD. 11
O
C. Capsule
B. Medulla D. Nephron
69. Five pints of HCl weighs 2.79 Kg. Calculate its specific gravity
A. 0.810 Ipt 473mL
-
2790g C. 2.180
B. 1.180 1-18 D. 3.180
¥Im= =
70. How much water should be added to 2500 mL of 83% (v/v) alcohol to prepare 50% (v/v) alcohol?
O
Igo g
A. 1650 83 D. 1560
-
50
parts so : 2500 : : 33 : x
B. 1660 E. 1450
C. 1550 151650
O -33parts
71. A method of solving problem that involves
83 the mixing of solution or mixture of solid
A. Alligation method O C. A & B
B. Alligation alternate D. None of the above
PRACTICE SET – MODULE 3 Prepared by: Grace Marie A. Maclan, RPh, CPS, MSc
Course Audit 2019 – 2020
UNIVERSITY OF SANTO TOMAS
FACULTY OF PHARMACY
DEPARTMENT OF PHARMACY
480 min
✓
s-oomhxaom-gtts.30.gg#tEaa.sO-012tabx7
73. Calculate the IV flow rate for 500mL D5W to run for 8 hours. The drop factor is 60gtts/mL
A. 70 gtts/min C. 63 gtts/min
B. 65 gtts/min D. 54 gtts/min
74. Calculate the number of tablets required for one week, if 12 tablets used each day
A. 12 days i 4g tab 0
C. 84
B. 86 D. 4
day
75. Change to percent the number 1/300
A. 3% C. 3.3%
B. 33% D. 13%
E 0.337
-
-
76. The dose of a drug for children based on body surface area is based on
.
A. Anxiety
o
78. Which of the following is not a subjective parameter
C. Insomia
B. Indigestion O
D. Respiratory rate -
objective
79. The emergency treatment of patients with heart failure
O
-
80. Thrombolytic agent such as streptokinase are useful for patient with
T Fon freely R
A. Myocardial infarction C. Heart failure
B. Hypertension D. Angina pectoris
not cause thrombosis
81. Glucosuria usually indicates the presence of
-0 A. Diabetes mellitus C. Multiple myeloma
(
B. Diabetes myeloma D. Acute hepatitis
urine
@
82. Above normal level of Uric acid indicate
A. Urinary Tract Infection C. Acute hepatitis
B. Urethral Contamination OD. Gout
①D. BUN
85. Involves measuring direct and indirect cost attributred to a specific disease
o
- -
86. A type of angina which can be induced by exertion, emotional stress or heavy metal and can be relieved
-
by rest, nitroglycerin or both
- -
coronary spasm -
B. Angina decubitus -
recumbent position ①
D. Stable angina
87. If a prescription order requires 25 g of concentrated HCl (density = 1.18 g/mL), what volume should the
pharmacist measure
A. 29.50 mL D= F V IIIs C. 23.0 mL
- -
- -
PRACTICE SET – MODULE 3 Prepared by: Grace Marie A. Maclan, RPh, CPS, MSc
Course Audit 2019 – 2020
UNIVERSITY OF SANTO TOMAS
FACULTY OF PHARMACY
DEPARTMENT OF PHARMACY
88. Force exerted on the ventricular muscle at the end of diastole that determines the degree of muscle fiber
stretch
A. Afterload tension in the ventricular muscle
-
OC. Preload
B. Cardiac output co HRXSV =
during D. Decompensation
contraction
Grand mat
90. Which of the following diuretics should be used as initial therapy for most patients with hypertension
o
-
92. Which of the following statement is/are true regarding the potential unfavorable effect of
antihypertensive t uric acid
I. (Thiazide diuretic should be used cautiously in patients with gout
II. Beta-blocker should generally be avoided in individuals with asthma
(
-
III. Aldosterone antagonist and potassium sparing diuretic can cause hypokalemia
D Ba receptors heart -
Farid hyperkalemia
A. I, II & III C. I & III
-
g
B. I & II D. II & III
A. Exercise
O
93. Which of the following is not a cause of Intrinsic Asthma
=
C. Emotional upset
①
B. Atopy Extrinsic
-
D. Viral infection
94. The usual initial dose of chlormbucil is 150mcg/kg body weight once a day. How many milligrams should
be administered to a person with a weight of 154 lbs
0
A. 10.5 mg C. 15 mg
B. 18 mg
150,792×70--10.5%-79 to D. 8 mg
-
-5mg
95. Patients with nephrotic syndrome wouyld exhibit the following except,
-
.
A. Edema C. Proteinuria
B. Hematuria D. Hyperalbuminemia
* Patient has proteinuria thus Iv protein (albumin) in blood
96. Patient EF was rushed to the ER due to difficulty of breathing
A. Medication history O
C. Chief complaint
B. History of present illness D. Past medical history
statins
"
-
98. Small part of a clot that breaksoff and travels to other parts of trhe vascular system
O
-0
A. Embolous C. Infarct
B. Thrombus D. Ischaemia
99. Inhibits degranulation of mast cells in the asmathic patients, thereby preventing the release of chemical
mediator of anaphylaxis
A. Theophylline C. Cromolyn sodium
B. Albuterol D. Catecholamine
—-END—-
PRACTICE SET – MODULE 3 Prepared by: Grace Marie A. Maclan, RPh, CPS, MSc
Course Audit 2019 – 2020
Type of angina is usually caused by lying down in a Which of the following drug distribution system is
recumbent position commonly used in government hospitals?
- Angina decubitus - Combination of floor stock and individual
prescription
Which of the following is not a regulated drug?
- Deka syrup Which of the following drug combination causes
physical incompatibility?
Calculate the volume of a 30g liquid with a specific - Alcohol + Acacia
gravity of 1.90 - Strychnine + Aromatic spirit of ammonia
- 15.78
A newly licensed physician took his taking his
Which preparation exhibits physical incompatibility specialized training in a government hospital. How
- Camphor + Menthol would you classify him as a medical staff?
- Resident staff
The process of wetting and smoothing zinc oxide with
mineral oil in preparation for incorporation into an Which of the following drugs causes SLE like
ointment base symptoms?
- Levigation - Hydralazine
In which part of the formulary is the list of dialyzable Which corticosteroid is used for the treatment of acute
poison included severe asthma
- Special information - Methyl prednisone
Which of the following type of questions should be Which of the following is written for the intended
avoided while interviewing the patient unusual dose
- Multiple questions, Leading question, - Putting quotation marks to the amount
Excessive yes/no questions
How many mL of the syrup has a specific gravity of
If a patient has a pain grade of 5/10 which of the 1.350 should be mixed with 3000mL of syrup having
following analgesic is best given? a specific gravity of 1.250 to obtain a product having
- Oxycodone a specific gravity of 1.310.
- 4500 mL
To prepare an irrigation solution containing 0.25%
wt/vol acetic acid, how much glacial acetic acid must Brewer's yeast is an example of which type of
be added to 1 gallon of water? dispensing
- 9g - Mechanical dispensing
The following is part of the drug product listing, Which of the following is not a risk factor for diabetes
except mellitus?
- Categories of drug - Low LDL
Which of the following is not a fundamental function A patient states that he smokes 8 stick of cigarette per
of a hospital day for 25 years. Compute for the number of pack
- Social service years
- 10
Advice for female adult taking Etretinate for psoriasis?
- Do not get pregnant
A pharmacist is to prepare #5 capsule of drug A with - Physical
a weight of 300 mg. The following capsule size could
be used except Which of the following is an antiviral preparation?
- 4 - Zovirax
Identify the best therapeutic plan for giving Phenytoin Which of the following agents is/are available in
to epileptic patients. sublingual dosage form?
- 15 mg/kg loading dose at nmt 50 mg/min - Isordil
rate
What is the difference, in percent, between a 1:4
Patient NW has a BMI of 27kg/m2, patient is solution and a 10% solution?
considered to be - 15%
- Overweight
The following are treatment goals for STEMI, except
It is a genetically determined abnormal response to an - Prevent total occlusion of the infarcted
ordinary dose of a drug area
- Idiosyncrasy
Procaine HCl (MW = 273) is a 2 ion electrolyte,
Calculate the specific gravity of a 5-pints HCl dissociating 80% in a certain concentration. Calculate
weighing 2.79 kg its NaCl equivalent (E)
- 1.180 - 0.21
Use of a drug that causes an allergic reaction is what Which supporting service of the hospital provides data
type of ADR? which can be used for research and education?
- Type B - Medical record
Direct patient care includes Four quartz is equivalent to how many mL?
- Counseling of self-administered - 3786 mL
medication
A tuberculin syringe can measure up to
"PAtient ABC, a 45-year-old male, married with 4 - 1 mL
children. He was born in Ilocos Norte and is currently
living in Manila." To which part of the patient chart is The process of preparing old-time recipe or special
the information located. medicine for a given medical supply?
- Patient demographics - Compounding
A 49-year-old woman presents to the clinic with an In preparing a solution which of the following is not
acute onset of dysuria with gross hematuria, true?
frequency, and urgency. What is the best management
for the condition?
- Start empiric therapy with nitrofurantoin Each solid ingredient must be dissolved in a solvent
they are most soluble
A type of hospital that provides specific service to the - Alcoholic solution is added to the aqueous
specific and selected type of patient or cases solution
- Special hospital
Which of the following statement are true regarding Which of the following is an activity of a pharmacist
compounding of suspension? in the dispensing area
I. Shake well label is to be placed on the final - Provides proper record keeping of patient
packaging medication record
II. Should always be added with flavoring to mask the
unpleasant taste Exforge is an anti-hypertensive agent which is a
III. Tight container must be used to ensure stability combination of:
IV. Should be vis - Amlodipine + Valsartan
- I, II, and IV
The following are factors in identifying turnover rate
What is the % w/v of iodine present if a pharmacist except
adds 76 mL of Strong Iodine Solution USP (5% w/v) - Date of purchase
to 1 L of sterile water for irrigation?
- 0.38% If a patient was prescribed Cefadroxil 30mg/kg/day (1
tbsp. bid) of a suspension with a strength of
Carcinogenicity is what type of ADR? 125mg/5mL, what is the weight of the patient?
- Type D correction: 1tsp bid daw dapat
- 18.3 lbs
Which of the following is not true regarding
medication order Carcinogenicity is under what type of ADR?
- Used in the community setting - Type D
"To start Metformin 500mg tablet PO BID, Applies to an instance when the physician did not
Amlodipine 5 mg/tablet PO OD and hook in IVF NSS specify the exact number of doses or duration of
1L". This statement is information gathered from what therapy?
part of the medical chart? - Automatic stop medication
- Physician’s order sheet
A neonatal patient with a weight of 6 lbs. was Night blindness is reported due to dietary deficiency
prescribed theophylline at an infusion rate of 0.06 of
mg/kg/hr. In a 12-hour infusion, how many mg of - Vitamin A
theophylline is to be used for this prescription?
- 1.7 mg All are criteria for the classification of systemic lupus
erythematosus (SLE); except:
What is the density of a 250 mL alcohol weighing - GI ulceration
203g?
- 0.812 g/mL M. ft. sol stands for
- mix to make a solution
A tertiary hospital would have how many bed capacity
- 300 - 399 Patient EG's BP measurement is 135/85 mmHg at the
first check-up. Which of the following is correct?
Which department of the hospital pharmacy is - ESC High Normal
responsible for the control and dispensing of IV fluids
- Departmental service Patient EF is a 63 years old male who has the
following laboratory results
30 minims is equivalent to ____ mL HDL-C 75 mg/dL
- 1.8 LDL-C 190 mg/dL
TG - 150 mg/dL
What is the standard size of a hospital bed? TC 226 mg/dL
- 36" x 84" Her Framingham Risk Score is Calculated to be 4%
and the LDL-C goal is 130mg. Which statin would you
Smoking stimulates drug-metabolizing hepatic recommend to the patient?
enzymes results in - Pravastatin 20mg 32% LDL-C reduction
- Decrease in the pharmacologic effect of the
drug Patient MDD was prescribed with MDI for her asthma
- Meter Dose Inhaler
In compounding parenteral chemotherapeutic agent
which of the following should be used? A pharmacist's clinical function include the following
- Vertical Laminar Flow Hood except:
- Drug administration
Which of the following is not a white blood cell?
- Reticulocyte Which of the following is a contraindication for
pregnant patients with hypertension?
Camphor, menthol, acetophenetidine, and phenol will - ACE inhibitor
form
- Eutectic mixture Inflammation of the tongue
- Glossitis
Secretary of the Pharmacy and Therapeutics
Committee Which of the following factors does not affect
- Chief pharmacist pharmaceutical stability
- Air
A prescription calls for 1 tablet ac, how many tablets
are needed for a 1-week treatment? A prescription calls for 1/2 tsp of Solmux 100mg/5mL
- 21 every 8 hours. For how many days would a 60mL
bottle last?
- 8 days
Express 0.5 mg / 500g capsule in % w/w. The following are the main components of TPN,
- 0.1% except
- Vitamin
Myocardial oxygen demand is increased by the
following, except: Which of the following drugs is to be administered to
- Metoprolol a patient who will be undergoing a pharmaceutical
stress test?
Which of the following chemical mediator causes - Dobutamine
chest pain
- Adenosine Given the prescription, which is an adjuvant
A physician requested 1 av. lb. Bacitracin ointment Potassium Iodide .................. 2.4%
containing 200 U of Bacitracin per gram. How many
grams of Bacitracin ointment (500 U/g) must be used Alcohol, qs ............................30.0 mL
to make this ointment? - Potassium iodide
- 182 g